НаТмитС "Enter", Ρ‡Ρ‚ΠΎΠ±Ρ‹ ΠΏΠ΅Ρ€Π΅ΠΉΡ‚ΠΈ ΠΊ ΡΠΎΠ΄Π΅Ρ€ΠΆΠ°Π½ΠΈΡŽ

ΠŸΠΎΡΡ‚Ρ€ΠΎΠ΅Π½ΠΈΠ΅ Π³Ρ€Π°Ρ„ΠΈΠΊΠ° Π² полярной систСмС ΠΊΠΎΠΎΡ€Π΄ΠΈΠ½Π°Ρ‚ ΠΎΠ½Π»Π°ΠΉΠ½: ΠŸΠΎΡΡ‚Ρ€ΠΎΠΈΡ‚ΡŒ Π³Ρ€Π°Ρ„ΠΈΠΊ Π² полярных ΠΊΠΎΠΎΡ€Π΄ΠΈΠ½Π°Ρ‚Π°Ρ… Π½Π° плоскости

Π‘ΠΎΠ΄Π΅Ρ€ΠΆΠ°Π½ΠΈΠ΅

ΠŸΠΎΡΡ‚Ρ€ΠΎΠ΅Π½ΠΈΠ΅ Π³Ρ€Π°Ρ„ΠΈΠΊΠ° Π² полярных ΠΊΠΎΠΎΡ€Π΄ΠΈΠ½Π°Ρ‚Π°Ρ…. ΠšΠΎΠ½Ρ‚Ρ€ΠΎΠ»ΡŒΠ½Ρ‹Π΅ ΠΎΠ½Π»Π°ΠΉΠ½

ΠŸΠΎΡΡ‚Ρ€ΠΎΠ΅Π½ΠΈΠ΅ Π³Ρ€Π°Ρ„ΠΈΠΊΠ° Π² полярных ΠΊΠΎΠΎΡ€Π΄ΠΈΠ½Π°Ρ‚Π°Ρ…

Π”Π°Π½ΠΎ ΡƒΡ€Π°Π²Π½Π΅Π½ΠΈΠ΅ ΠΊΡ€ΠΈΠ²ΠΎΠΉ Π² полярной систСмС ΠΊΠΎΠΎΡ€Π΄ΠΈΠ½Π°Ρ‚ .
ВрСбуСтся:
Β Β  Π°) ΠΏΠΎΡΡ‚Ρ€ΠΎΠΈΡ‚ΡŒ ΠΊΡ€ΠΈΠ²ΡƒΡŽ ΠΏΠΎ Ρ‚ΠΎΡ‡ΠΊΠ°ΠΌ, придавая j значСния ΠΈΠ· ΠΏΡ€ΠΎΠΌΠ΅ΠΆΡƒΡ‚ΠΊΠ°  с шагом ;
Β Β  Π±) Π·Π°ΠΏΠΈΡΠ°Ρ‚ΡŒ ΡƒΡ€Π°Π²Π½Π΅Π½ΠΈΠ΅ этой ΠΊΡ€ΠΈΠ²ΠΎΠΉ Π² Π΄Π΅ΠΊΠ°Ρ€Ρ‚ΠΎΠ²ΠΎΠΉ ΠΏΡ€ΡΠΌΠΎΡƒΠ³ΠΎΠ»ΡŒΠ½ΠΎΠΉ систСмС ΠΊΠΎΠΎΡ€Π΄ΠΈΠ½Π°Ρ‚, согласованной с полярной, ΠΈ ΠΎΠΏΡ€Π΅Π΄Π΅Π»ΠΈΡ‚ΡŒ Ρ‚ΠΈΠΏ этой ΠΊΡ€ΠΈΠ²ΠΎΠΉ.
РСшСниС
Β Β  Π°) Боставим Ρ‚Π°Π±Π»ΠΈΡ†Ρƒ Π·Π½Π°Ρ‡Π΅Π½ΠΈΠΉ Ρ„ΡƒΠ½ΠΊΡ†ΠΈΠΈ.

j

0

p/8

p/4

3p/8

p/2

5p/8

3p/4

7p/8

p

9p/8

5p/8

11p/8

3p/2

13p/8

7p/4

15p/8

r

3

2,8

2,32

1,72

1,5

1,26

1,11

1,02

1

1,02

1,11

1,26

1,5

1,72

2,32

2,8

Β Β  По этим Π΄Π°Π½Π½Ρ‹ΠΌ ΠΎΡ‚ΠΌΠ΅Ρ‚ΠΈΠΌ Ρ‚ΠΎΡ‡ΠΊΠΈ Π½Π° плоскости ΠΈ, ΠΏΠ»Π°Π²Π½ΠΎ соСдиняя сосСдниС Ρ‚ΠΎΡ‡ΠΊΠΈ, построим линию.

Π±) ΠŸΠ΅Ρ€Π΅ΠΉΠ΄Ρ‘ΠΌ ΠΊ Π΄Π΅ΠΊΠ°Ρ€Ρ‚ΠΎΠ²ΠΎΠΉ ΠΏΡ€ΡΠΌΠΎΡƒΠ³ΠΎΠ»ΡŒΠ½ΠΎΠΉ систСмС ΠΊΠΎΠΎΡ€Π΄ΠΈΠ½Π°Ρ‚, ΠΏΠΎΠ»ΡŒΠ·ΡƒΡΡΡŒ Ρ„ΠΎΡ€ΠΌΡƒΠ»Π°ΠΌΠΈ , .
Π—Π°Π΄Π°Π½Π½ΠΎΠ΅ ΡƒΡ€Π°Π²Π½Π΅Π½ΠΈΠ΅ ΠΏΡ€ΠΈΠΌΠ΅Ρ‚ Π²ΠΈΠ΄ .
ΠŸΡ€Π΅ΠΎΠ±Ρ€Π°Π·ΡƒΠ΅ΠΌ это ΡƒΡ€Π°Π²Π½Π΅Π½ΠΈΠ΅: ,

, , , .
Π’Ρ‹Π΄Π΅Π»ΠΈΠ² ΠΏΠΎΠ»Π½Ρ‹Π΅ ΠΊΠ²Π°Π΄Ρ€Π°Ρ‚Ρ‹ ΠΏΠ΅Ρ€Π΅ΠΌΠ΅Π½Π½Ρ‹Ρ… Β ΠΈ , ΠΏΠΎΠ»ΡƒΡ‡ΠΈΠΌ Β ΠΈΠ»ΠΈΒ  .

Β Β  Π­Ρ‚ΠΎ ΡƒΡ€Π°Π²Π½Π΅Π½ΠΈΠ΅ эллипса с Ρ†Π΅Π½Ρ‚Ρ€ΠΎΠΌ Π² Ρ‚ΠΎΡ‡ΠΊΠ΅ Β ΠΈ полуосями Β ΠΈ .

ΠŸΠΎΡΡ‚Ρ€ΠΎΠ΅Π½ΠΈΠ΅ Π³Ρ€Π°Ρ„ΠΈΠΊΠΎΠ² ΠΎΠ½Π»Π°ΠΉΠ½. Π“Ρ€Π°Ρ„ΠΈΠΊ Ρ„ΡƒΠ½ΠΊΡ†ΠΈΠΈ y=sin x Π“Ρ€Π°Ρ„ΠΈΠΊ sin 2x

Как ΠΏΠΎΡΡ‚Ρ€ΠΎΠΈΡ‚ΡŒ Π³Ρ€Π°Ρ„ΠΈΠΊ Ρ„ΡƒΠ½ΠΊΡ†ΠΈΠΈ y=sin x? Для Π½Π°Ρ‡Π°Π»Π° рассмотрим Π³Ρ€Π°Ρ„ΠΈΠΊ синуса Π½Π° ΠΏΡ€ΠΎΠΌΠ΅ΠΆΡƒΡ‚ΠΊΠ΅ .

Π•Π΄ΠΈΠ½ΠΈΡ‡Π½Ρ‹ΠΉ ΠΎΡ‚Ρ€Π΅Π·ΠΎΠΊ Π±Π΅Ρ€Ρ‘ΠΌ Π΄Π»ΠΈΠ½ΠΎΠΉ 2 ΠΊΠ»Π΅Ρ‚ΠΎΡ‡ΠΊΠΈ Ρ‚Π΅Ρ‚Ρ€Π°Π΄ΠΈ. На оси Oy ΠΎΡ‚ΠΌΠ΅Ρ‡Π°Π΅ΠΌ Π΅Π΄ΠΈΠ½ΠΈΡ†Ρƒ.

Для удобства число Ο€/2 округляСм Π΄ΠΎ 1,5 (Π° Π½Π΅ Π΄ΠΎ 1,6, ΠΊΠ°ΠΊ трСбуСтся ΠΏΠΎ ΠΏΡ€Π°Π²ΠΈΠ»Π°ΠΌ округлСния). Π’ этом случаС ΠΎΡ‚Ρ€Π΅Π·ΠΊΡƒ Π΄Π»ΠΈΠ½ΠΎΠΉ Ο€/2 ΡΠΎΠΎΡ‚Π²Π΅Ρ‚ΡΡ‚Π²ΡƒΡŽΡ‚ 3 ΠΊΠ»Π΅Ρ‚ΠΎΡ‡ΠΊΠΈ.

На оси Ox ΠΎΡ‚ΠΌΠ΅Ρ‡Π°Π΅ΠΌ Π½Π΅ Π΅Π΄ΠΈΠ½ΠΈΡ‡Π½Ρ‹Π΅ ΠΎΡ‚Ρ€Π΅Π·ΠΊΠΈ, Π° ΠΎΡ‚Ρ€Π΅Π·ΠΊΠΈ Π΄Π»ΠΈΠ½ΠΎΠΉ Ο€/2 (Ρ‡Π΅Ρ€Π΅Π· ΠΊΠ°ΠΆΠ΄Ρ‹Π΅ 3 ΠΊΠ»Π΅Ρ‚ΠΎΡ‡ΠΊΠΈ). БоотвСтствСнно, ΠΎΡ‚Ρ€Π΅Π·ΠΊΡƒ Π΄Π»ΠΈΠ½ΠΎΠΉ Ο€ соотвСтствуСт 6 ΠΊΠ»Π΅Ρ‚ΠΎΡ‡Π΅ΠΊ, ΠΎΡ‚Ρ€Π΅Π·ΠΊΡƒ Π΄Π»ΠΈΠ½ΠΎΠΉ Ο€/6 β€” 1 ΠΊΠ»Π΅Ρ‚ΠΎΡ‡ΠΊΠ°.

ΠŸΡ€ΠΈ Ρ‚Π°ΠΊΠΎΠΌ Π²Ρ‹Π±ΠΎΡ€Π΅ Π΅Π΄ΠΈΠ½ΠΈΡ‡Π½ΠΎΠ³ΠΎ ΠΎΡ‚Ρ€Π΅Π·ΠΊΠ° Π³Ρ€Π°Ρ„ΠΈΠΊ, ΠΈΠ·ΠΎΠ±Ρ€Π°ΠΆΡ‘Π½Π½Ρ‹ΠΉ Π½Π° листС Ρ‚Π΅Ρ‚Ρ€Π°Π΄ΠΈ Π² ΠΊΠ»Π΅Ρ‚ΠΎΡ‡ΠΊΡƒ, максимально соотвСтствуСт Π³Ρ€Π°Ρ„ΠΈΠΊΡƒ Ρ„ΡƒΠ½ΠΊΡ†ΠΈΠΈ y=sin x.

Боставим Ρ‚Π°Π±Π»ΠΈΡ†Ρƒ Π·Π½Π°Ρ‡Π΅Π½ΠΈΠΉ синуса Π½Π° ΠΏΡ€ΠΎΠΌΠ΅ΠΆΡƒΡ‚ΠΊΠ΅ :

ΠŸΠΎΠ»ΡƒΡ‡Π΅Π½Π½Ρ‹Π΅ Ρ‚ΠΎΡ‡ΠΊΠΈ ΠΎΡ‚ΠΌΠ΅Ρ‚ΠΈΠΌ Π½Π° ΠΊΠΎΠΎΡ€Π΄ΠΈΠ½Π°Ρ‚Π½ΠΎΠΉ плоскости:

Π’Π°ΠΊ ΠΊΠ°ΠΊ y=sin x β€” нСчётная функция, Π³Ρ€Π°Ρ„ΠΈΠΊ синуса симмСтричСн ΠΎΡ‚Π½ΠΎΡΠΈΡ‚Π΅Π»ΡŒΠ½ΠΎ Π½Π°Ρ‡Π°Π»Π° отсчёта β€” Ρ‚ΠΎΡ‡ΠΊΠΈ O(0;0). Π‘ ΡƒΡ‡Ρ‘Ρ‚ΠΎΠΌ этого Ρ„Π°ΠΊΡ‚Π° ΠΏΡ€ΠΎΠ΄ΠΎΠ»ΠΆΠΈΠΌ построСниС Π³Ρ€Π°Ρ„ΠΈΠΊΠ° Π²Π»Π΅Π²ΠΎ, Ρ‚ΠΎ Ρ‚ΠΎΡ‡ΠΊΠΈ -Ο€:

Ѐункция y=sin x β€” пСриодичСская с ΠΏΠ΅Ρ€ΠΈΠΎΠ΄ΠΎΠΌ T=2Ο€. ΠŸΠΎΡΡ‚ΠΎΠΌΡƒ Π³Ρ€Π°Ρ„ΠΈΠΊ Ρ„ΡƒΠ½ΠΊΡ†ΠΈΠΈ, взятый Π½Π° Π½Π° ΠΏΡ€ΠΎΠΌΠ΅ΠΆΡƒΡ‚ΠΊΠ΅ [-Ο€;Ο€], повторяСтся бСсконСчноС число Ρ€Π°Π· Π²ΠΏΡ€Π°Π²ΠΎ ΠΈ Π²Π»Π΅Π²ΠΎ.

Π£Ρ€ΠΎΠΊ ΠΈ прСзСнтация Π½Π° Ρ‚Π΅ΠΌΡƒ: «Π€ΡƒΠ½ΠΊΡ†ΠΈΡ y=sin(x). ΠžΠΏΡ€Π΅Π΄Π΅Π»Π΅Π½ΠΈΡ ΠΈ свойства»

Π”ΠΎΠΏΠΎΠ»Π½ΠΈΡ‚Π΅Π»ΡŒΠ½Ρ‹Π΅ ΠΌΠ°Ρ‚Π΅Ρ€ΠΈΠ°Π»Ρ‹
Π£Π²Π°ΠΆΠ°Π΅ΠΌΡ‹Π΅ ΠΏΠΎΠ»ΡŒΠ·ΠΎΠ²Π°Ρ‚Π΅Π»ΠΈ, Π½Π΅ Π·Π°Π±Ρ‹Π²Π°ΠΉΡ‚Π΅ ΠΎΡΡ‚Π°Π²Π»ΡΡ‚ΡŒ свои ΠΊΠΎΠΌΠΌΠ΅Π½Ρ‚Π°Ρ€ΠΈΠΈ, ΠΎΡ‚Π·Ρ‹Π²Ρ‹, поТСлания! ВсС ΠΌΠ°Ρ‚Π΅Ρ€ΠΈΠ°Π»Ρ‹ ΠΏΡ€ΠΎΠ²Π΅Ρ€Π΅Π½Ρ‹ антивирусной ΠΏΡ€ΠΎΠ³Ρ€Π°ΠΌΠΌΠΎΠΉ.

Пособия ΠΈ Ρ‚Ρ€Π΅Π½Π°ΠΆΠ΅Ρ€Ρ‹ Π² ΠΈΠ½Ρ‚Π΅Ρ€Π½Π΅Ρ‚-ΠΌΠ°Π³Π°Π·ΠΈΠ½Π΅ «Π˜Π½Ρ‚Π΅Π³Ρ€Π°Π»» для 10 класса ΠΎΡ‚ 1Π‘
РСшаСм Π·Π°Π΄Π°Ρ‡ΠΈ ΠΏΠΎ Π³Π΅ΠΎΠΌΠ΅Ρ‚Ρ€ΠΈΠΈ. Π˜Π½Ρ‚Π΅Ρ€Π°ΠΊΡ‚ΠΈΠ²Π½Ρ‹Π΅ задания Π½Π° построСниС для 7-10 классов
ΠŸΡ€ΠΎΠ³Ρ€Π°ΠΌΠΌΠ½Π°Ρ срСда «1Π‘: ΠœΠ°Ρ‚Π΅ΠΌΠ°Ρ‚ΠΈΡ‡Π΅ΡΠΊΠΈΠΉ конструктор 6.1»

Π§Ρ‚ΠΎ Π±ΡƒΠ΄Π΅ΠΌ ΠΈΠ·ΡƒΡ‡Π°Ρ‚ΡŒ:

  • Бвойства Ρ„ΡƒΠ½ΠΊΡ†ΠΈΠΈ Y=sin(X).
  • Π“Ρ€Π°Ρ„ΠΈΠΊ Ρ„ΡƒΠ½ΠΊΡ†ΠΈΠΈ.
  • Как ΡΡ‚Ρ€ΠΎΠΈΡ‚ΡŒ Π³Ρ€Π°Ρ„ΠΈΠΊ ΠΈ Π΅Π³ΠΎ ΠΌΠ°ΡΡˆΡ‚Π°Π±.
  • ΠŸΡ€ΠΈΠΌΠ΅Ρ€Ρ‹.

Бвойства синуса. Y=sin(X)

РСбята, ΠΌΡ‹ ΡƒΠΆΠ΅ познакомились с тригономСтричСскими функциями числового Π°Ρ€Π³ΡƒΠΌΠ΅Π½Ρ‚Π°. Π’Ρ‹ ΠΏΠΎΠΌΠ½ΠΈΡ‚Π΅ ΠΈΡ…?

Π”Π°Π²Π°ΠΉΡ‚Π΅ познакомимся ΠΏΠΎΠ±Π»ΠΈΠΆΠ΅ с Ρ„ΡƒΠ½ΠΊΡ†ΠΈΠ΅ΠΉ Y=sin(X)

Π—Π°ΠΏΠΈΡˆΠ΅ΠΌ Π½Π΅ΠΊΠΎΡ‚ΠΎΡ€Ρ‹Π΅ свойства этой Ρ„ΡƒΠ½ΠΊΡ†ΠΈΠΈ:
1) ΠžΠ±Π»Π°ΡΡ‚ΡŒ опрСдСлСния – мноТСство Π΄Π΅ΠΉΡΡ‚Π²ΠΈΡ‚Π΅Π»ΡŒΠ½Ρ‹Ρ… чисСл.
2) Ѐункция нСчСтная. Π”Π°Π²Π°ΠΉΡ‚Π΅ вспомним ΠΎΠΏΡ€Π΅Π΄Π΅Π»Π΅Π½ΠΈΠ΅ Π½Π΅Ρ‡Π΅Ρ‚Π½ΠΎΠΉ Ρ„ΡƒΠ½ΠΊΡ†ΠΈΠΈ. Ѐункция называСтся Π½Π΅Ρ‡Π΅Ρ‚Π½ΠΎΠΉ Ссли выполняСтся равСнство: y(-x)=-y(x). Как ΠΌΡ‹ ΠΏΠΎΠΌΠ½ΠΈΠΌ ΠΈΠ· Ρ„ΠΎΡ€ΠΌΡƒΠ» привидСния: sin(-x)=-sin(x). ΠžΠΏΡ€Π΅Π΄Π΅Π»Π΅Π½ΠΈΠ΅ Π²Ρ‹ΠΏΠΎΠ»Π½ΠΈΠ»ΠΎΡΡŒ, Π·Π½Π°Ρ‡ΠΈΡ‚ Y=sin(X) – нСчСтная функция.
3) Ѐункция Y=sin(X) возрастаСт Π½Π° ΠΎΡ‚Ρ€Π΅Π·ΠΊΠ΅ ΠΈ ΡƒΠ±Ρ‹Π²Π°Π΅Ρ‚ Π½Π° ΠΎΡ‚Ρ€Π΅Π·ΠΊΠ΅ [Ο€/2; Ο€]. Когда ΠΌΡ‹ двиТСмся ΠΏΠΎ ΠΏΠ΅Ρ€Π²ΠΎΠΉ Ρ‡Π΅Ρ‚Π²Π΅Ρ€Ρ‚ΠΈ (ΠΏΡ€ΠΎΡ‚ΠΈΠ² часовой стрСлки), ΠΎΡ€Π΄ΠΈΠ½Π°Ρ‚Π° увСличиваСтся, Π° ΠΏΡ€ΠΈ Π΄Π²ΠΈΠΆΠ΅Π½ΠΈΠΈ ΠΏΠΎ Π²Ρ‚ΠΎΡ€ΠΎΠΉ Ρ‡Π΅Ρ‚Π²Π΅Ρ€Ρ‚ΠΈ ΠΎΠ½Π° ΡƒΠΌΠ΅Π½ΡŒΡˆΠ°Π΅Ρ‚ΡΡ.

4) Ѐункция Y=sin(X) ΠΎΠ³Ρ€Π°Π½ΠΈΡ‡Π΅Π½Π° снизу ΠΈ свСрху. Π”Π°Π½Π½ΠΎΠ΅ свойство слСдуСт ΠΈΠ· Ρ‚ΠΎΠ³ΠΎ, Ρ‡Ρ‚ΠΎ

-1 ≀ sin(X) ≀ 1
5) НаимСньшСС Π·Π½Π°Ρ‡Π΅Π½ΠΈΠ΅ Ρ„ΡƒΠ½ΠΊΡ†ΠΈΠΈ Ρ€Π°Π²Π½ΠΎ -1 (ΠΏΡ€ΠΈ Ρ… = — Ο€/2+ Ο€k). НаибольшСС Π·Π½Π°Ρ‡Π΅Π½ΠΈΠ΅ Ρ„ΡƒΠ½ΠΊΡ†ΠΈΠΈ Ρ€Π°Π²Π½ΠΎ 1 (ΠΏΡ€ΠΈ Ρ… = Ο€/2+ Ο€k).

Π”Π°Π²Π°ΠΉΡ‚Π΅, воспользовавшись свойствами 1-5, построим Π³Ρ€Π°Ρ„ΠΈΠΊ Ρ„ΡƒΠ½ΠΊΡ†ΠΈΠΈ Y=sin(X). Π‘ΡƒΠ΄Π΅ΠΌ ΡΡ‚Ρ€ΠΎΠΈΡ‚ΡŒ наш Π³Ρ€Π°Ρ„ΠΈΠΊ ΠΏΠΎΡΠ»Π΅Π΄ΠΎΠ²Π°Ρ‚Π΅Π»ΡŒΠ½ΠΎ, примСняя наши свойства. НачнСм ΡΡ‚Ρ€ΠΎΠΈΡ‚ΡŒ Π³Ρ€Π°Ρ„ΠΈΠΊ Π½Π° ΠΎΡ‚Ρ€Π΅Π·ΠΊΠ΅ .

ОсобоС Π²Π½ΠΈΠΌΠ°Π½ΠΈΠ΅ стоит ΠΎΠ±Ρ€Π°Ρ‚ΠΈΡ‚ΡŒ Π½Π° ΠΌΠ°ΡΡˆΡ‚Π°Π±. На оси ΠΎΡ€Π΄ΠΈΠ½Π°Ρ‚ ΡƒΠ΄ΠΎΠ±Π½Π΅Π΅ ΠΏΡ€ΠΈΠ½ΡΡ‚ΡŒ Π΅Π΄ΠΈΠ½ΠΈΡ‡Π½Ρ‹ΠΉ ΠΎΡ‚Ρ€Π΅Π·ΠΎΠΊ Ρ€Π°Π²Π½Ρ‹ΠΉ 2 ΠΊΠ»Π΅Ρ‚ΠΎΡ‡ΠΊΠ°ΠΌ, Π° Π½Π° оси абсцисс — Π΅Π΄ΠΈΠ½ΠΈΡ‡Π½Ρ‹ΠΉ ΠΎΡ‚Ρ€Π΅Π·ΠΎΠΊ (Π΄Π²Π΅ ΠΊΠ»Π΅Ρ‚ΠΎΡ‡ΠΊΠΈ) ΠΏΡ€ΠΈΠ½ΡΡ‚ΡŒ Ρ€Π°Π²Π½Ρ‹ΠΌ Ο€/3 (смотритС рисунок).


ΠŸΠΎΡΡ‚Ρ€ΠΎΠ΅Π½ΠΈΠ΅ Π³Ρ€Π°Ρ„ΠΈΠΊΠ° Ρ„ΡƒΠ½ΠΊΡ†ΠΈΠΈ синус Ρ…, y=sin(x)

ΠŸΠΎΡΡ‡ΠΈΡ‚Π°Π΅ΠΌ значСния Ρ„ΡƒΠ½ΠΊΡ†ΠΈΠΈ Π½Π° нашСм ΠΎΡ‚Ρ€Π΅Π·ΠΊΠ΅:


ΠŸΠΎΡΡ‚Ρ€ΠΎΠΈΠΌ Π³Ρ€Π°Ρ„ΠΈΠΊ ΠΏΠΎ нашим Ρ‚ΠΎΡ‡ΠΊΠ°ΠΌ, с ΡƒΡ‡Π΅Ρ‚ΠΎΠΌ Ρ‚Ρ€Π΅Ρ‚ΡŒΠ΅Π³ΠΎ свойства.

Π’Π°Π±Π»ΠΈΡ†Π° ΠΏΡ€Π΅ΠΎΠ±Ρ€Π°Π·ΠΎΠ²Π°Π½ΠΈΠΉ для Ρ„ΠΎΡ€ΠΌΡƒΠ» привидСния

Π’ΠΎΡΠΏΠΎΠ»ΡŒΠ·ΡƒΠ΅ΠΌΡΡ Π²Ρ‚ΠΎΡ€Ρ‹ΠΌ свойством, ΠΊΠΎΡ‚ΠΎΡ€ΠΎΠ΅ Π³ΠΎΠ²ΠΎΡ€ΠΈΡ‚, Ρ‡Ρ‚ΠΎ наша функция нСчСтная, Π° это Π·Π½Π°Ρ‡ΠΈΡ‚, Ρ‡Ρ‚ΠΎ Π΅Π΅ ΠΌΠΎΠΆΠ½ΠΎ ΠΎΡ‚Ρ€Π°Π·ΠΈΡ‚ΡŒ симмСтрично ΠΎΡ‚Π½ΠΎΡΠΈΡ‚Π΅Π»ΡŒΠ½ΠΎ Π½Π°Ρ‡Π°Π»ΠΎ ΠΊΠΎΠΎΡ€Π΄ΠΈΠ½Π°Ρ‚:


ΠœΡ‹ Π·Π½Π°Π΅ΠΌ, Ρ‡Ρ‚ΠΎ sin(x+ 2Ο€) = sin(x). Π­Ρ‚ΠΎ Π·Π½Π°Ρ‡ΠΈΡ‚, Ρ‡Ρ‚ΠΎ Π½Π° ΠΎΡ‚Ρ€Π΅Π·ΠΊΠ΅ [- Ο€; Ο€] Π³Ρ€Π°Ρ„ΠΈΠΊ выглядит Ρ‚Π°ΠΊ ΠΆΠ΅, ΠΊΠ°ΠΊ Π½Π° ΠΎΡ‚Ρ€Π΅Π·ΠΊΠ΅ [Ο€; 3Ο€] ΠΈΠ»ΠΈ ΠΈΠ»ΠΈ [-3Ο€; — Ο€] ΠΈ Ρ‚Π°ΠΊ Π΄Π°Π»Π΅Π΅. Нам остаСтся Π°ΠΊΠΊΡƒΡ€Π°Ρ‚Π½ΠΎ ΠΏΠ΅Ρ€Π΅Ρ€ΠΈΡΠΎΠ²Π°Ρ‚ΡŒ Π³Ρ€Π°Ρ„ΠΈΠΊ Π½Π° ΠΏΡ€Π΅Π΄Ρ‹Π΄ΡƒΡ‰Π΅ΠΌ рисункС Π½Π° всю ось абсцисс.

Π“Ρ€Π°Ρ„ΠΈΠΊ Ρ„ΡƒΠ½ΠΊΡ†ΠΈΠΈ Y=sin(X) Π½Π°Π·Ρ‹Π²Π°ΡŽΡ‚ — синусоидой.

НапишСм Π΅Ρ‰Π΅ нСсколько свойств согласно построСнному Π³Ρ€Π°Ρ„ΠΈΠΊΡƒ:
6) Ѐункция Y=sin(X) возрастаСт Π½Π° любом ΠΎΡ‚Ρ€Π΅Π·ΠΊΠ΅ Π²ΠΈΠ΄Π°: [- Ο€/2+ 2Ο€k; Ο€/2+ 2Ο€k], k – Ρ†Π΅Π»ΠΎΠ΅ число ΠΈ ΡƒΠ±Ρ‹Π²Π°Π΅Ρ‚ Π½Π° любом ΠΎΡ‚Ρ€Π΅Π·ΠΊΠ΅ Π²ΠΈΠ΄Π°: [Ο€/2+ 2Ο€k; 3Ο€/2+ 2Ο€k], k – Ρ†Π΅Π»ΠΎΠ΅ число.
7) Ѐункция Y=sin(X) – нСпрСрывная функция. ΠŸΠΎΡΠΌΠΎΡ‚Ρ€ΠΈΠΌ Π½Π° Π³Ρ€Π°Ρ„ΠΈΠΊ Ρ„ΡƒΠ½ΠΊΡ†ΠΈΠΈ ΠΈ убСдимся Ρ‡Ρ‚ΠΎ Ρƒ нашСй Ρ„ΡƒΠ½ΠΊΡ†ΠΈΠΈ Π½Π΅Ρ‚ Ρ€Π°Π·Ρ€Ρ‹Π²ΠΎΠ², это ΠΈ ΠΎΠ·Π½Π°Ρ‡Π°Π΅Ρ‚ Π½Π΅ΠΏΡ€Π΅Ρ€Ρ‹Π²Π½ΠΎΡΡ‚ΡŒ.
8) ΠžΠ±Π»Π°ΡΡ‚ΡŒ Π·Π½Π°Ρ‡Π΅Π½ΠΈΠΉ: ΠΎΡ‚Ρ€Π΅Π·ΠΎΠΊ [- 1; 1]. Π­Ρ‚ΠΎ Ρ‚Π°ΠΊΠΆΠ΅ Ρ…ΠΎΡ€ΠΎΡˆΠΎ Π²ΠΈΠ΄Π½ΠΎ ΠΈΠ· Π³Ρ€Π°Ρ„ΠΈΠΊΠ° Ρ„ΡƒΠ½ΠΊΡ†ΠΈΠΈ.
9) Ѐункция Y=sin(X) — пСриодичСская функция. ΠŸΠΎΡΠΌΠΎΡ‚Ρ€ΠΈΠΌ ΠΎΠΏΡΡ‚ΡŒ Π½Π° Π³Ρ€Π°Ρ„ΠΈΠΊ ΠΈ ΡƒΠ²ΠΈΠ΄ΠΈΠΌ, Ρ‡Ρ‚ΠΎ функция ΠΏΡ€ΠΈΠ½ΠΈΠΌΠ°Π΅Ρ‚ ΠΎΠ΄Π½ΠΈ ΠΈ Ρ‚Π΅ ΠΆΠ΅ значСния, Ρ‡Π΅Ρ€Π΅Π· Π½Π΅ΠΊΠΎΡ‚ΠΎΡ€Ρ‹Π΅ ΠΏΡ€ΠΎΠΌΠ΅ΠΆΡƒΡ‚ΠΊΠΈ.

ΠŸΡ€ΠΈΠΌΠ΅Ρ€Ρ‹ Π·Π°Π΄Π°Ρ‡ с синусом

1. Π Π΅ΡˆΠΈΡ‚ΡŒ ΡƒΡ€Π°Π²Π½Π΅Π½ΠΈΠ΅ sin(x)= x-Ο€

РСшСниС: ΠŸΠΎΡΡ‚Ρ€ΠΎΠΈΠΌ 2 Π³Ρ€Π°Ρ„ΠΈΠΊΠ° Ρ„ΡƒΠ½ΠΊΡ†ΠΈΠΈ: y=sin(x) ΠΈ y=x-Ο€ (см. рисунок).
Наши Π³Ρ€Π°Ρ„ΠΈΠΊΠΈ ΠΏΠ΅Ρ€Π΅ΡΠ΅ΠΊΠ°ΡŽΡ‚ΡΡ Π² ΠΎΠ΄Π½ΠΎΠΉ Ρ‚ΠΎΡ‡ΠΊΠ΅ А(Ο€;0), это ΠΈ Π΅ΡΡ‚ΡŒ ΠΎΡ‚Π²Π΅Ρ‚: x = Ο€


2. ΠŸΠΎΡΡ‚Ρ€ΠΎΠΈΡ‚ΡŒ Π³Ρ€Π°Ρ„ΠΈΠΊ Ρ„ΡƒΠ½ΠΊΡ†ΠΈΠΈ y=sin(Ο€/6+x)-1

РСшСниС: Π˜ΡΠΊΠΎΠΌΡ‹ΠΉ Π³Ρ€Π°Ρ„ΠΈΠΊ получится ΠΏΡƒΡ‚Π΅ΠΌ пСрСноса Π³Ρ€Π°Ρ„ΠΈΠΊΠ° Ρ„ΡƒΠ½ΠΊΡ†ΠΈΠΈ y=sin(x) Π½Π° Ο€/6 Π΅Π΄ΠΈΠ½ΠΈΡ† Π²Π»Π΅Π²ΠΎ ΠΈ 1 Π΅Π΄ΠΈΠ½ΠΈΡ†Ρƒ Π²Π½ΠΈΠ·.


РСшСниС: ΠŸΠΎΡΡ‚Ρ€ΠΎΠΈΠΌ Π³Ρ€Π°Ρ„ΠΈΠΊ Ρ„ΡƒΠ½ΠΊΡ†ΠΈΠΈ ΠΈ рассмотрим наш ΠΎΡ‚Ρ€Π΅Π·ΠΎΠΊ [Ο€/2; 5Ο€/4].
На Π³Ρ€Π°Ρ„ΠΈΠΊΠ΅ Ρ„ΡƒΠ½ΠΊΡ†ΠΈΠΈ Π²ΠΈΠ΄Π½ΠΎ, Ρ‡Ρ‚ΠΎ наибольшиС ΠΈ наимСньшиС значСния Π΄ΠΎΡΡ‚ΠΈΠ³Π°ΡŽΡ‚ΡΡ Π½Π° ΠΊΠΎΠ½Ρ†Π°Ρ… ΠΎΡ‚Ρ€Π΅Π·ΠΊΠ°, Π² Ρ‚ΠΎΡ‡ΠΊΠ°Ρ… Ο€/2 ΠΈ 5Ο€/4 соотвСтствСнно.
ΠžΡ‚Π²Π΅Ρ‚: sin(Ο€/2) = 1 – наибольшСС Π·Π½Π°Ρ‡Π΅Π½ΠΈΠ΅, sin(5Ο€/4) = наимСньшСС Π·Π½Π°Ρ‡Π΅Π½ΠΈΠ΅.

Π—Π°Π΄Π°Ρ‡ΠΈ Π½Π° синус для ΡΠ°ΠΌΠΎΡΡ‚ΠΎΡΡ‚Π΅Π»ΡŒΠ½ΠΎΠ³ΠΎ Ρ€Π΅ΡˆΠ΅Π½ΠΈΡ


  • Π Π΅ΡˆΠΈΡ‚Π΅ ΡƒΡ€Π°Π²Π½Π΅Π½ΠΈΠ΅: sin(x)= x+3Ο€, sin(x)= x-5Ο€
  • ΠŸΠΎΡΡ‚Ρ€ΠΎΠΈΡ‚ΡŒ Π³Ρ€Π°Ρ„ΠΈΠΊ Ρ„ΡƒΠ½ΠΊΡ†ΠΈΠΈ y=sin(Ο€/3+x)-2
  • ΠŸΠΎΡΡ‚Ρ€ΠΎΠΈΡ‚ΡŒ Π³Ρ€Π°Ρ„ΠΈΠΊ Ρ„ΡƒΠ½ΠΊΡ†ΠΈΠΈ y=sin(-2Ο€/3+x)+1
  • Найти наибольшСС ΠΈ наимСньшСС Π·Π½Π°Ρ‡Π΅Π½ΠΈΠ΅ Ρ„ΡƒΠ½ΠΊΡ†ΠΈΠΈ y=sin(x) Π½Π° ΠΎΡ‚Ρ€Π΅Π·ΠΊΠ΅
  • Найти наибольшСС ΠΈ наимСньшСС Π·Π½Π°Ρ‡Π΅Π½ΠΈΠ΅ Ρ„ΡƒΠ½ΠΊΡ†ΠΈΠΈ y=sin(x) Π½Π° ΠΎΡ‚Ρ€Π΅Π·ΠΊΠ΅ [- Ο€/3; 5Ο€/6]

Β«ΠŸΠΎΡΡ‚Ρ€ΠΎΠ΅Π½ΠΈΠ΅ Π³Ρ€Π°Ρ„ΠΈΠΊΠ° Ρ„ΡƒΠ½ΠΊΡ†ΠΈΠΈ с ΠΌΠΎΠ΄ΡƒΠ»Π΅ΠΌΒ» — Y = lnx. Π—Π°ΠΊΡ€Π΅ΠΏΠΈΠ»ΠΈ знания Π½Π° Ρ€Π°Π½Π΅Π΅ ΠΈΠ·ΡƒΡ‡Π΅Π½Π½Ρ‹Ρ… функциях. ΠŸΠΎΡΡ‚Ρ€ΠΎΠ΅Π½ΠΈΠ΅ Π³Ρ€Π°Ρ„ΠΈΠΊΠΎΠ² Ρ„ΡƒΠ½ΠΊΡ†ΠΈΠΉ. Вопрос классу. Y = x2 – 2x – 3. ΠŸΡ€ΠΎΠ΅ΠΊΡ‚Π½Π°Ρ Π΄Π΅ΡΡ‚Π΅Π»ΡŒΠ½ΠΎΡΡ‚ΡŒ. Π£Ρ€ΠΎΠΊ обобщСния ΠΈ систСматизации Π·Π½Π°Π½ΠΈΠΉ. Π“Ρ€Π°Ρ„ΠΈΠΊ Ρ„ΡƒΠ½ΠΊΡ†ΠΈΠΈ. Актуализация Π·Π½Π°Π½ΠΈΠΉ ΠΎ Π³Ρ€Π°Ρ„ΠΈΠΊΠ°Ρ… Ρ„ΡƒΠ½ΠΊΡ†ΠΈΠΉ. ΠžΠ±ΠΎΠ±Ρ‰Π΅Π½ΠΈΠ΅. ΠŸΠΎΠΏΡ€ΠΎΠ±ΡƒΠΉΡ‚Π΅ ΡΠ°ΠΌΠΎΡΡ‚ΠΎΡΡ‚Π΅Π»ΡŒΠ½ΠΎ ΠΏΠΎΡΡ‚Ρ€ΠΎΠΈΡ‚ΡŒ Π³Ρ€Π°Ρ„ΠΈΠΊΠΈ. Y = f(x).

««Графики Ρ„ΡƒΠ½ΠΊΡ†ΠΈΠΉΒ» 9 класс» — Π¦Π΅Π»ΠΈ ΡƒΡ€ΠΎΠΊΠ°. Π‘ΠΎΠ»ΡŒΡˆΠ΅ΠΌΡƒ Π·Π½Π°Ρ‡Π΅Π½ΠΈΡŽ Π°Ρ€Π³ΡƒΠΌΠ΅Π½Ρ‚Π° соотвСтствуСт большСС Π·Π½Π°Ρ‡Π΅Π½ΠΈΠ΅ Ρ„ΡƒΠ½ΠΊΡ†ΠΈΠΈ. Нули Ρ„ΡƒΠ½ΠΊΡ†ΠΈΠΈ. ΠžΠΏΡ€Π΅Π΄Π΅Π»Π΅Π½ΠΈΠ΅. Π—Π°ΠΏΠΎΠ»Π½ΠΈΡ‚Π΅ пропуски. УстановитС соотвСтствиС ΠΌΠ΅ΠΆΠ΄Ρƒ Ρ„ΡƒΠ½ΠΊΡ†ΠΈΠ΅ΠΉ ΠΈ Π²Π΅Ρ€ΡˆΠΈΠ½ΠΎΠΉ. Π’Ρ€Π΅Π½Π°ΠΆΠ΅Ρ€. Π’Ρ‹Π±Π΅Ρ€ΠΈΡ‚Π΅ ΡƒΡ€Π°Π²Π½Π΅Π½ΠΈΠ΅, с ΠΏΠΎΠΌΠΎΡ‰ΡŒΡŽ ΠΊΠΎΡ‚ΠΎΡ€ΠΎΠ³ΠΎ Π·Π°Π΄Π°Π½Π° линСйная функция. УстановитС соотвСтствиС. Π’Ρ‹Π±Π΅Ρ€ΠΈΡ‚Π΅ ΡƒΡ€Π°Π²Π½Π΅Π½ΠΈΠ΅. ΠžΠ±Ρ€Π°Ρ‚Π½Π°Ρ ΠΏΡ€ΠΎΠΏΠΎΡ€Ρ†ΠΈΠΎΠ½Π°Π»ΡŒΠ½ΠΎΡΡ‚ΡŒ.

Β«Π“Ρ€Π°Ρ„ΠΈΠΊΠΈ Ρ„ΡƒΠ½ΠΊΡ†ΠΈΠΉ с модулями» — Найдём Π²Π΅Ρ€ΡˆΠΈΠ½Ρƒ Ρ„ΡƒΠ½ΠΊΡ†ΠΈΠΈ. ΠšΡƒΠ±ΠΈΡ‡Π΅ΡΠΊΠ°Ρ функция. ΠžΡ‚Ρ€ΠΈΡ†Π°Ρ‚Π΅Π»ΡŒΠ½Π°Ρ сторона. Π“Ρ€Π°Ρ„ΠΈΠΊΠΈ Ρ„ΡƒΠ½ΠΊΡ†ΠΈΠΉ. ΠšΠ²Π°Π΄Ρ€Π°Ρ‚ΠΈΡ‡Π½Π°Ρ функция. БлоТная функция. Ѐункция с ΠΌΠΎΠ΄ΡƒΠ»Π΅ΠΌ. Π“Ρ€Π°Ρ„ΠΈΠΊΠΈ Ρ„ΡƒΠ½ΠΊΡ†ΠΈΠΉ Π½Π°Π΄ΠΎ ΠΎΠ±ΡΠ·Π°Ρ‚Π΅Π»ΡŒΠ½ΠΎ ΡƒΠΌΠ΅Ρ‚ΡŒ ΡΡ‚Ρ€ΠΎΠΈΡ‚ΡŒ. ΠŸΠΎΠ΄Π³ΠΎΡ‚ΠΎΠ²ΠΊΠ° ΠΊ Π•Π“Π­. Π“Ρ€Π°Ρ„ΠΈΠΊΠΈ Ρ„ΡƒΠ½ΠΊΡ†ΠΈΠΉ с модулями. ΠŸΠ°Ρ€Π°Π±ΠΎΠ»Π°. Π“Ρ€Π°Ρ„ΠΈΠΊ Ρ„ΡƒΠ½ΠΊΡ†ΠΈΠΈ.

Β«Π£Ρ€Π°Π²Π½Π΅Π½ΠΈΠ΅ ΠΊΠ°ΡΠ°Ρ‚Π΅Π»ΡŒΠ½ΠΎΠΉ ΠΊ Π³Ρ€Π°Ρ„ΠΈΠΊΡƒ Ρ„ΡƒΠ½ΠΊΡ†ΠΈΠΈΒ» — ΠŸΡ€ΠΎΠΈΠ·Π²ΠΎΠ΄Π½Π°Ρ Π² Ρ‚ΠΎΡ‡ΠΊΠ΅. ΠŸΡ€Π°Π²ΠΈΠ»Π° диффСрСнцирования. Π“Ρ€Π°Ρ„ΠΈΠΊ Ρ„ΡƒΠ½ΠΊΡ†ΠΈΠΈ. Алгоритм нахоТдСния уравнСния. ΠžΡ‚Π²Π΅Ρ‚ΡŒΡ‚Π΅ Π½Π° вопросы. ГСомСтричСский смысл ΠΏΡ€ΠΎΠΈΠ·Π²ΠΎΠ΄Π½ΠΎΠΉ. НомСра ΠΈΠ· ΡƒΡ‡Π΅Π±Π½ΠΈΠΊΠ°. Π£Ρ€Π°Π²Π½Π΅Π½ΠΈΠ΅ ΠΊΠ°ΡΠ°Ρ‚Π΅Π»ΡŒΠ½ΠΎΠΉ ΠΊ Π³Ρ€Π°Ρ„ΠΈΠΊΡƒ Ρ„ΡƒΠ½ΠΊΡ†ΠΈΠΈ. ΠžΠΏΡ€Π΅Π΄Π΅Π»Π΅Π½ΠΈΠ΅. ΠšΠ°ΡΠ°Ρ‚Π΅Π»ΡŒΠ½Π°Ρ ΠΊ Π³Ρ€Π°Ρ„ΠΈΠΊΡƒ Ρ„ΡƒΠ½ΠΊΡ†ΠΈΠΈ. ΠžΡΠ½ΠΎΠ²Π½Ρ‹Π΅ Ρ„ΠΎΡ€ΠΌΡƒΠ»Ρ‹ диффСрСнцирования. ΠŸΡ€ΠΎΠ²Π΅ΡΡ‚ΠΈ ΠΊΠ°ΡΠ°Ρ‚Π΅Π»ΡŒΠ½ΡƒΡŽ.

Β«ΠŸΠΎΡΡ‚Ρ€ΠΎΠ΅Π½ΠΈΠ΅ Π³Ρ€Π°Ρ„ΠΈΠΊΠΎΠ² Ρ„ΡƒΠ½ΠΊΡ†ΠΈΠΉΒ» — ΠŸΠΎΡΡ‚Ρ€ΠΎΠ΅Π½ΠΈΠ΅ Π³Ρ€Π°Ρ„ΠΈΠΊΠ° Ρ„ΡƒΠ½ΠΊΡ†ΠΈΠΈ y = sinx. Линия тангСнсов. АлгСбра. Π’Π΅ΠΌΠ°: ΠŸΠΎΡΡ‚Ρ€ΠΎΠ΅Π½ΠΈΠ΅ Π³Ρ€Π°Ρ„ΠΈΠΊΠΎΠ² Ρ„ΡƒΠ½ΠΊΡ†ΠΈΠΉ. Π“Ρ€Π°Ρ„ΠΈΠΊ Ρ„ΡƒΠ½ΠΊΡ†ΠΈΠΈ y = sinx. Π’Ρ‹ΠΏΠΎΠ»Π½ΠΈΠ»Π°: Π€ΠΈΠ»ΠΈΠΏΠΏΠΎΠ²Π° ΠΠ°Ρ‚Π°Π»ΡŒΡ Π’Π°ΡΠΈΠ»ΡŒΠ΅Π²Π½Π° ΡƒΡ‡ΠΈΡ‚Π΅Π»ΡŒ ΠΌΠ°Ρ‚Π΅ΠΌΠ°Ρ‚ΠΈΠΊΠΈ БСлоярская срСдняя ΠΎΠ±Ρ‰Π΅ΠΎΠ±Ρ€Π°Π·ΠΎΠ²Π°Ρ‚Π΅Π»ΡŒΠ½Π°Ρ школа β„–1. ΠŸΠΎΡΡ‚Ρ€ΠΎΠΈΡ‚ΡŒ Π³Ρ€Π°Ρ„ΠΈΠΊ Ρ„ΡƒΠ½ΠΊΡ†ΠΈΠΈ y=sin(x) +cos(x).

Β«Π“Ρ€Π°Ρ„ΠΈΠΊ ΠΎΠ±Ρ€Π°Ρ‚Π½ΠΎΠΉ ΠΏΡ€ΠΎΠΏΠΎΡ€Ρ†ΠΈΠΎΠ½Π°Π»ΡŒΠ½ΠΎΡΡ‚ΠΈΒ» — ΠŸΡ€ΠΈΠΌΠ΅Π½Π΅Π½ΠΈΠ΅ Π³ΠΈΠΏΠ΅Ρ€Π±ΠΎΠ»Ρ‹. Π“ΠΈΠΏΠ΅Ρ€Π±ΠΎΠ»Π°. ΠœΠΎΠ½ΠΎΡ‚ΠΎΠ½Π½ΠΎΡΡ‚ΡŒ Ρ„ΡƒΠ½ΠΊΡ†ΠΈΠΈ. Π§Ρ‘Ρ‚Π½ΠΎΡΡ‚ΡŒ, Π½Π΅Ρ‡Ρ‘Ρ‚Π½ΠΎΡΡ‚ΡŒ. Ѐункция Β«ΠžΠ±Ρ€Π°Ρ‚Π½Π°Ρ ΠΏΡ€ΠΎΠΏΠΎΡ€Ρ†ΠΈΠΎΠ½Π°Π»ΡŒΠ½ΠΎΡΡ‚ΡŒΒ». Π“Ρ€Π°Ρ„ΠΈΠΊ. ΠŸΠΎΡΡ‚Ρ€ΠΎΠ΅Π½ΠΈΠ΅ Π³Ρ€Π°Ρ„ΠΈΠΊΠ° ΠΎΠ±Ρ€Π°Ρ‚Π½ΠΎΠΉ ΠΏΡ€ΠΎΠΏΠΎΡ€Ρ†ΠΈΠΎΠ½Π°Π»ΡŒΠ½ΠΎΡΡ‚ΠΈ. Π“ΠΈΠΏΠ΅Ρ€Π±ΠΎΠ»Π° ΠΈ космичСскиС спутники. ΠžΠ΄Π½ΠΎΠΏΠΎΠ»ΠΎΡΡ‚Π½ΠΎΠΉ Π³ΠΈΠΏΠ΅Ρ€Π±ΠΎΠ»ΠΎΠΈΠ΄. Асимптота. ΠŸΡ€ΠΈΠΌΠ΅Π½Π΅Π½ΠΈΠ΅ Π³ΠΈΠΏΠ΅Ρ€Π±ΠΎΠ»ΠΎΠΈΠ΄ΠΎΠ². ΠžΠΏΡ€Π΅Π΄Π΅Π»Π΅Π½ΠΈΠ΅ ΠΎΠ±Ρ€Π°Ρ‚Π½ΠΎΠΉ ΠΏΡ€ΠΎΠΏΠΎΡ€Ρ†ΠΈΠΎΠ½Π°Π»ΡŒΠ½ΠΎΡΡ‚ΠΈ.

ВсСго Π² Ρ‚Π΅ΠΌΠ΅ 25 ΠΏΡ€Π΅Π·Π΅Π½Ρ‚Π°Ρ†ΠΈΠΉ

ΠŸΠΎΡΡ‚Ρ€ΠΎΠΈΡ‚ΡŒ Ρ„ΡƒΠ½ΠΊΡ†ΠΈΡŽ

ΠœΡ‹ ΠΏΡ€Π΅Π΄Π»Π°Π³Π°Π΅ΠΌ Π²Π°ΡˆΠ΅ΠΌΡƒ вниманию сСрвис ΠΏΠΎ ΠΏΠΎΡ‚Ρ€ΠΎΠ΅Π½ΠΈΡŽ Π³Ρ€Π°Ρ„ΠΈΠΊΠΎΠ² Ρ„ΡƒΠ½ΠΊΡ†ΠΈΠΉ ΠΎΠ½Π»Π°ΠΉΠ½, всС ΠΏΡ€Π°Π²Π° Π½Π° ΠΊΠΎΡ‚ΠΎΡ€Ρ‹ΠΉ ΠΏΡ€ΠΈΠ½Π°Π΄Π»Π΅ΠΆΠ°Ρ‚ ΠΊΠΎΠΌΠΏΠ°Π½ΠΈΠΈ

Desmos .2/16=1)
  • Π’ΠΎΠ·ΠΌΠΎΠΆΠ½ΠΎΡΡ‚ΡŒ ΡΠΎΡ…Ρ€Π°Π½ΡΡ‚ΡŒ Π³Ρ€Π°Ρ„ΠΈΠΊΠΈ ΠΈ ΠΏΠΎΠ»ΡƒΡ‡Π°Ρ‚ΡŒ Π½Π° Π½ΠΈΡ… ссылку, которая становится доступной для всСх Π² ΠΈΠ½Ρ‚Π΅Ρ€Π½Π΅Ρ‚Π΅
  • Π£ΠΏΡ€Π°Π²Π»Π΅Π½ΠΈΠ΅ ΠΌΠ°ΡΡˆΡ‚Π°Π±ΠΎΠΌ, Ρ†Π²Π΅Ρ‚ΠΎΠΌ Π»ΠΈΠ½ΠΈΠΉ
  • Π’ΠΎΠ·ΠΌΠΎΠΆΠ½ΠΎΡΡ‚ΡŒ построСния Π³Ρ€Π°Ρ„ΠΈΠΊΠΎΠ² ΠΏΠΎ Ρ‚ΠΎΡ‡ΠΊΠ°ΠΌ, использованиС констант
  • ΠŸΠΎΡΡ‚Ρ€ΠΎΠ΅Π½ΠΈΠ΅ ΠΎΠ΄Π½ΠΎΠ²Ρ€Π΅ΠΌΠ΅Π½Π½ΠΎ Π½Π΅ΡΠΊΠΎΠ»ΡŒΠΊΠΈΡ… Π³Ρ€Π°Ρ„ΠΈΠΊΠΎΠ² Ρ„ΡƒΠ½ΠΊΡ†ΠΈΠΉ
  • ΠŸΠΎΡΡ‚Ρ€ΠΎΠ΅Π½ΠΈΠ΅ Π³Ρ€Π°Ρ„ΠΈΠΊΠΎΠ² Π² полярной систСмС ΠΊΠΎΠΎΡ€Π΄ΠΈΠ½Π°Ρ‚ (ΠΈΡΠΏΠΎΠ»ΡŒΠ·ΡƒΠΉΡ‚Π΅ r ΠΈ ΞΈ(\theta))
  • Π‘ Π½Π°ΠΌΠΈ Π»Π΅Π³ΠΊΠΎ Π² Ρ€Π΅ΠΆΠΈΠΌΠ΅ ΠΎΠ½Π»Π°ΠΉΠ½ ΡΡ‚Ρ€ΠΎΠΈΡ‚ΡŒ Π³Ρ€Π°Ρ„ΠΈΠΊΠΈ Ρ€Π°Π·Π»ΠΈΡ‡Π½ΠΎΠΉ слоТности. ΠŸΠΎΡΡ‚Ρ€ΠΎΠ΅Π½ΠΈΠ΅ производится ΠΌΠ³Π½ΠΎΠ²Π΅Π½Π½ΠΎ. БСрвис вострСбован для нахоТдСния Ρ‚ΠΎΡ‡Π΅ΠΊ пСрСсСчСния Ρ„ΡƒΠ½ΠΊΡ†ΠΈΠΉ, для изобраТСния Π³Ρ€Π°Ρ„ΠΈΠΊΠΎΠ² для дальнСйшСго ΠΈΡ… пСрСмСщСния Π² Word Π΄ΠΎΠΊΡƒΠΌΠ΅Π½Ρ‚ Π² качСствС ΠΈΠ»Π»ΡŽΡΡ‚Ρ€Π°Ρ†ΠΈΠΉ ΠΏΡ€ΠΈ Ρ€Π΅ΡˆΠ΅Π½ΠΈΠΈ Π·Π°Π΄Π°Ρ‡, для Π°Π½Π°Π»ΠΈΠ·Π° повСдСнчСских особСнностСй Π³Ρ€Π°Ρ„ΠΈΠΊΠΎΠ² Ρ„ΡƒΠ½ΠΊΡ†ΠΈΠΉ. ΠžΠΏΡ‚ΠΈΠΌΠ°Π»ΡŒΠ½Ρ‹ΠΌ Π±Ρ€Π°ΡƒΠ·Π΅Ρ€ΠΎΠΌ для Ρ€Π°Π±ΠΎΡ‚Ρ‹ с Π³Ρ€Π°Ρ„ΠΈΠΊΠ°ΠΌΠΈ Π½Π° Π΄Π°Π½Π½ΠΎΠΉ страницС сайта являСтся Google Chrome. ΠŸΡ€ΠΈ использовании Π΄Ρ€ΡƒΠ³ΠΈΡ… Π±Ρ€Π°ΡƒΠ·Π΅Ρ€ΠΎΠ² ΠΊΠΎΡ€Ρ€Π΅ΠΊΡ‚Π½ΠΎΡΡ‚ΡŒ Ρ€Π°Π±ΠΎΡ‚Ρ‹ Π½Π΅ гарантируСтся.

    ΠŸΠΎΡΡ‚Ρ€ΠΎΠΈΡ‚ΡŒ Π³Ρ€Π°Ρ„ΠΈΠΊ y 2×2

    ΠŸΠΎΡΡ‚Ρ€ΠΎΠ΅Π½ΠΈΠ΅ Π³Ρ€Π°Ρ„ΠΈΠΊΠΎΠ² ΠΎΠ½Π»Π°ΠΉΠ½ с ΠΏΠΎΠΌΠΎΡ‰ΡŒΡŽ нашСго сСрвиса являСтся простой Π·Π°Π΄Π°Ρ‡Π΅ΠΉ.2/16=1)

  • Π’ΠΎΠ·ΠΌΠΎΠΆΠ½ΠΎΡΡ‚ΡŒ ΡΠΎΡ…Ρ€Π°Π½ΡΡ‚ΡŒ Π³Ρ€Π°Ρ„ΠΈΠΊΠΈ ΠΈ ΠΏΠΎΠ»ΡƒΡ‡Π°Ρ‚ΡŒ Π½Π° Π½ΠΈΡ… ссылку, которая становится доступной для всСх Π² ΠΈΠ½Ρ‚Π΅Ρ€Π½Π΅Ρ‚Π΅
  • Π£ΠΏΡ€Π°Π²Π»Π΅Π½ΠΈΠ΅ ΠΌΠ°ΡΡˆΡ‚Π°Π±ΠΎΠΌ, Ρ†Π²Π΅Ρ‚ΠΎΠΌ Π»ΠΈΠ½ΠΈΠΉ
  • Π’ΠΎΠ·ΠΌΠΎΠΆΠ½ΠΎΡΡ‚ΡŒ построСния Π³Ρ€Π°Ρ„ΠΈΠΊΠΎΠ² ΠΏΠΎ Ρ‚ΠΎΡ‡ΠΊΠ°ΠΌ, использованиС констант
  • ΠŸΠΎΡΡ‚Ρ€ΠΎΠ΅Π½ΠΈΠ΅ ΠΎΠ΄Π½ΠΎΠ²Ρ€Π΅ΠΌΠ΅Π½Π½ΠΎ Π½Π΅ΡΠΊΠΎΠ»ΡŒΠΊΠΈΡ… Π³Ρ€Π°Ρ„ΠΈΠΊΠΎΠ² Ρ„ΡƒΠ½ΠΊΡ†ΠΈΠΉ
  • ΠŸΠΎΡΡ‚Ρ€ΠΎΠ΅Π½ΠΈΠ΅ Π³Ρ€Π°Ρ„ΠΈΠΊΠΎΠ² Π² полярной систСмС ΠΊΠΎΠΎΡ€Π΄ΠΈΠ½Π°Ρ‚ (ΠΈΡΠΏΠΎΠ»ΡŒΠ·ΡƒΠΉΡ‚Π΅ r ΠΈ ΞΈ( heta) )
  • Π‘ Π½Π°ΠΌΠΈ Π»Π΅Π³ΠΊΠΎ Π² Ρ€Π΅ΠΆΠΈΠΌΠ΅ ΠΎΠ½Π»Π°ΠΉΠ½ ΡΡ‚Ρ€ΠΎΠΈΡ‚ΡŒ Π³Ρ€Π°Ρ„ΠΈΠΊΠΈ Ρ€Π°Π·Π»ΠΈΡ‡Π½ΠΎΠΉ слоТности. ΠŸΠΎΡΡ‚Ρ€ΠΎΠ΅Π½ΠΈΠ΅ производится ΠΌΠ³Π½ΠΎΠ²Π΅Π½Π½ΠΎ. БСрвис вострСбован для нахоТдСния Ρ‚ΠΎΡ‡Π΅ΠΊ пСрСсСчСния Ρ„ΡƒΠ½ΠΊΡ†ΠΈΠΉ, для изобраТСния Π³Ρ€Π°Ρ„ΠΈΠΊΠΎΠ² для дальнСйшСго ΠΈΡ… пСрСмСщСния Π² Word Π΄ΠΎΠΊΡƒΠΌΠ΅Π½Ρ‚ Π² качСствС ΠΈΠ»Π»ΡŽΡΡ‚Ρ€Π°Ρ†ΠΈΠΉ ΠΏΡ€ΠΈ Ρ€Π΅ΡˆΠ΅Π½ΠΈΠΈ Π·Π°Π΄Π°Ρ‡, для Π°Π½Π°Π»ΠΈΠ·Π° повСдСнчСских особСнностСй Π³Ρ€Π°Ρ„ΠΈΠΊΠΎΠ² Ρ„ΡƒΠ½ΠΊΡ†ΠΈΠΉ. ΠžΠΏΡ‚ΠΈΠΌΠ°Π»ΡŒΠ½Ρ‹ΠΌ Π±Ρ€Π°ΡƒΠ·Π΅Ρ€ΠΎΠΌ для Ρ€Π°Π±ΠΎΡ‚Ρ‹ с Π³Ρ€Π°Ρ„ΠΈΠΊΠ°ΠΌΠΈ Π½Π° Π΄Π°Π½Π½ΠΎΠΉ страницС сайта являСтся Google Chrome. ΠŸΡ€ΠΈ использовании Π΄Ρ€ΡƒΠ³ΠΈΡ… Π±Ρ€Π°ΡƒΠ·Π΅Ρ€ΠΎΠ² ΠΊΠΎΡ€Ρ€Π΅ΠΊΡ‚Π½ΠΎΡΡ‚ΡŒ Ρ€Π°Π±ΠΎΡ‚Ρ‹ Π½Π΅ гарантируСтся.

    Π§Ρ‚ΠΎ Ρ‚Ρ‹ Ρ…ΠΎΡ‡Π΅ΡˆΡŒ ΡƒΠ·Π½Π°Ρ‚ΡŒ?

    ΠžΡ‚Π²Π΅Ρ‚

    ΠŸΡ€ΠΎΠ²Π΅Ρ€Π΅Π½ΠΎ экспСртом

    Π“Ρ€Π°Ρ„ΠΈΠΊ Ρ„ΡƒΠ½ΠΊΡ†ΠΈΠΈ прСдставляСт собой ΠΏΠ°Ρ€Π°Π±ΠΎΠ»Ρƒ.
    Учитывая, Ρ‡Ρ‚ΠΎ Π°=-2 Π΅Ρ‘ Π²Π΅Ρ‚Π²ΠΈ Π½Π°ΠΏΡ€Π°Π²Π»Π΅Π½Ρ‹ Π²Π½ΠΈΠ·.

    Π’Π΅Ρ€ΡˆΠΈΠ½Π° ΠΏΠ°Ρ€Π°Π±ΠΎΠ»Ρ‹ находится Π² Ρ‚ΠΎΡ‡ΠΊΠ΅ (0; 0)

    ΠŸΡ€ΠΈΠΌΠ΅Ρ€ поиска ΠΏΡ€ΠΎΠΌΠ΅ΠΆΡƒΡ‚ΠΎΡ‡Π½Ρ‹Ρ… Ρ‚ΠΎΡ‡Π΅ΠΊ.
    ΠŸΡƒΡΡ‚ΡŒ Ρ… = 1, Ρ‚ΠΎΠ³Π΄Π° Ρƒ = -2 * 1 Β² = -2
    Ρ… = -1, Ρ‚ΠΎΠ³Π΄Π° Ρƒ = -2 * (-1)Β² = -2 ΠΈ Ρ‚.Π΄.

    Π‘Π°ΠΌ Π³Ρ€Π°Ρ„ΠΈΠΊ ΠΈ Ρ‚Π°Π±Π»ΠΈΡ†Π° сданными для построСниС, прСдставлСно Π½ΠΈΠΆΠ΅.

    Рассмотрим Ρ„ΡƒΠ½ΠΊΡ†ΠΈΡŽ Π·Π°Π΄Π°Π½Π½ΡƒΡŽ Ρ„ΠΎΡ€ΠΌΡƒΠ»ΠΎΠΉ y = x 2 .

    На основании опрСдСлСния Ρ„ΡƒΠ½ΠΊΡ†ΠΈΠΈ ΠΊΠ°ΠΆΠ΄ΠΎΠΌΡƒ Π·Π½Π°Ρ‡Π΅Π½ΠΈΡŽ Π°Ρ€Π³ΡƒΠΌΠ΅Π½Ρ‚Π° Ρ…
    ΠΈΠ· области опрСдСлСния R ( всС Π΄Π΅ΠΉΡΡ‚Π²ΠΈΡ‚Π΅Π»ΡŒΠ½Ρ‹Π΅ числа )
    соотвСтствуСт СдинствСнноС Π·Π½Π°Ρ‡Π΅Π½ΠΈΠ΅ Ρ„ΡƒΠ½ΠΊΡ†ΠΈΠΈ y , Ρ€Π°Π²Π½ΠΎΠ΅ x 2 .

    НапримСр, ΠΏΡ€ΠΈ Ρ… = 3 Π·Π½Π°Ρ‡Π΅Π½ΠΈΠ΅ Ρ„ΡƒΠ½ΠΊΡ†ΠΈΠΈ y = 3 2 = 9 ,
    Π° ΠΏΡ€ΠΈ Ρ… = –2 Π·Π½Π°Ρ‡Π΅Π½ΠΈΠ΅ Ρ„ΡƒΠ½ΠΊΡ†ΠΈΠΈ y = ( –2 ) 2 = 4 .

    Π˜Π·ΠΎΠ±Ρ€Π°Π·ΠΈΠΌ Π³Ρ€Π°Ρ„ΠΈΠΊ Ρ„ΡƒΠ½ΠΊΡ†ΠΈΠΈ y = x 2 . Для этого присвоим
    Π°Ρ€Π³ΡƒΠΌΠ΅Π½Ρ‚Ρƒ Ρ… нСсколько Π·Π½Π°Ρ‡Π΅Π½ΠΈΠΉ, вычислим ΡΠΎΠΎΡ‚Π²Π΅Ρ‚ΡΡ‚Π²ΡƒΡŽΡ‰ΠΈΠ΅ значСния
    Ρ„ΡƒΠ½ΠΊΡ†ΠΈΠΈ ΠΈ внСсСм ΠΈΡ… Π² Ρ‚Π°Π±Π»ΠΈΡ†Ρƒ.

    Если: x = –3 , x = –2 , x = –1 , x = 0 , x = 1 , x = 2 , x = 3 ,

    Ρ‚ΠΎ: y = 9 , y = 4 , y = 1 , y = 0 , y = 1 , y = 4 , y = 9 .

    НанСсСм Ρ‚ΠΎΡ‡ΠΊΠΈ с вычислСнными ΠΊΠΎΠΎΡ€Π΄ΠΈΠ½Π°Ρ‚Π°ΠΌΠΈ (x ; y) Π½Π° ΠΏΠ»ΠΎΡΠΊΠΎΡΡ‚ΡŒ ΠΈ
    соСдиним ΠΈΡ… ΠΏΠ»Π°Π²Π½ΠΎΠΉ Π½Π΅ΠΏΡ€Π΅Ρ€Ρ‹Π²Π½ΠΎΠΉ ΠΊΡ€ΠΈΠ²ΠΎΠΉ. Π­Ρ‚Π° кривая, Π½Π°Π·Ρ‹Π²Π°ΡŽΡ‰Π°ΡΡΡ
    ΠΏΠ°Ρ€Π°Π±ΠΎΠ»ΠΎΠΉ, ΠΈ Π΅ΡΡ‚ΡŒ Π³Ρ€Π°Ρ„ΠΈΠΊ исслСдуСмой Π½Π°ΠΌΠΈ Ρ„ΡƒΠ½ΠΊΡ†ΠΈΠΈ.

    На Π³Ρ€Π°Ρ„ΠΈΠΊΠ΅ Π²ΠΈΠ΄Π½ΠΎ, Ρ‡Ρ‚ΠΎ ось OY Π΄Π΅Π»ΠΈΡ‚ ΠΏΠ°Ρ€Π°Π±ΠΎΠ»Ρƒ Π½Π° симмСтричныС
    Π»Π΅Π²ΡƒΡŽ ΠΈ ΠΏΡ€Π°Π²ΡƒΡŽ части (Π²Π΅Ρ‚Π²ΠΈ ΠΏΠ°Ρ€Π°Π±ΠΎΠ»Ρ‹), Π² Ρ‚ΠΎΡ‡ΠΊΠ΅ с ΠΊΠΎΠΎΡ€Π΄ΠΈΠ½Π°Ρ‚Π°ΠΌΠΈ (0; 0)
    (Π²Π΅Ρ€ΡˆΠΈΠ½Π΅ ΠΏΠ°Ρ€Π°Π±ΠΎΠ»Ρ‹) Π·Π½Π°Ρ‡Π΅Π½ΠΈΠ΅ Ρ„ΡƒΠ½ΠΊΡ†ΠΈΠΈ x 2 β€” наимСньшСС.
    НаибольшСго значСния функция Π½Π΅ ΠΈΠΌΠ΅Π΅Ρ‚. Π’Π΅Ρ€ΡˆΠΈΠ½Π° ΠΏΠ°Ρ€Π°Π±ΠΎΠ»Ρ‹ β€” это
    Ρ‚ΠΎΡ‡ΠΊΠ° пСрСсСчСния Π³Ρ€Π°Ρ„ΠΈΠΊΠ° с осью симмСтрии OY .

    На участкС Π³Ρ€Π°Ρ„ΠΈΠΊΠ° ΠΏΡ€ΠΈ x ∈ (– ∞ ; 0 ] функция ΡƒΠ±Ρ‹Π²Π°Π΅Ρ‚,
    Π° ΠΏΡ€ΠΈ x ∈ [ 0; + ∞ ) возрастаСт.

    Ѐункция y = x 2 являСтся частным случаСм ΠΊΠ²Π°Π΄Ρ€Π°Ρ‚ΠΈΡ‡Π½ΠΎΠΉ Ρ„ΡƒΠ½ΠΊΡ†ΠΈΠΈ.

    Рассмотрим Π΅Ρ‰Ρ‘ нСсколько Π΅Ρ‘ Π²Π°Ρ€ΠΈΠ°Π½Ρ‚ΠΎΠ². НапримСр, y = – x 2 .

    Π“Ρ€Π°Ρ„ΠΈΠΊΠΎΠΌ Ρ„ΡƒΠ½ΠΊΡ†ΠΈΠΈ y = – x 2 Ρ‚Π°ΠΊΠΆΠ΅ являСтся ΠΏΠ°Ρ€Π°Π±ΠΎΠ»Π°,
    Π½ΠΎ Π΅Ρ‘ Π²Π΅Ρ‚Π²ΠΈ Π½Π°ΠΏΡ€Π°Π²Π»Π΅Π½Ρ‹ Π²Π½ΠΈΠ·.

    Эскиз Π³Ρ€Π°Ρ„ΠΈΠΊΠ° Ρ„ΡƒΠ½ΠΊΡ†ΠΈΠΈ (Π½Π° ΠΏΡ€ΠΈΠΌΠ΅Ρ€Π΅ Π΄Ρ€ΠΎΠ±Π½ΠΎ-ΠΊΠ²Π°Π΄Ρ€Π°Ρ‚ΠΈΡ‡Π½ΠΎΠΉ Ρ„ΡƒΠ½ΠΊΡ†ΠΈΠΈ). ΠŸΠΎΡΡ‚Ρ€ΠΎΠ΅Π½ΠΈΠ΅ Π³Ρ€Π°Ρ„ΠΈΠΊΠΎΠ² ΠΎΠ½Π»Π°ΠΉΠ½

    Π’ Π΄Π°Π½Π½ΠΎΠΌ ΡƒΡ€ΠΎΠΊΠ΅ ΠΌΡ‹ рассмотрим ΠΌΠ΅Ρ‚ΠΎΠ΄ΠΈΠΊΡƒ построСния эскиза Π³Ρ€Π°Ρ„ΠΈΠΊΠ° Ρ„ΡƒΠ½ΠΊΡ†ΠΈΠΈ, ΠΏΡ€ΠΈΠ²Π΅Π΄Π΅ΠΌ Ρ€Π°Π·ΡŠΡΡΠ½ΡΡŽΡ‰ΠΈΠ΅ ΠΏΡ€ΠΈΠΌΠ΅Ρ€Ρ‹.

    Π’Π΅ΠΌΠ°: ΠŸΠΎΠ²Ρ‚ΠΎΡ€Π΅Π½ΠΈΠ΅

    Π£Ρ€ΠΎΠΊ: Эскиз Π³Ρ€Π°Ρ„ΠΈΠΊΠ° Ρ„ΡƒΠ½ΠΊΡ†ΠΈΠΈ (Π½Π° ΠΏΡ€ΠΈΠΌΠ΅Ρ€Π΅ Π΄Ρ€ΠΎΠ±Π½ΠΎ-ΠΊΠ²Π°Π΄Ρ€Π°Ρ‚ΠΈΡ‡Π½ΠΎΠΉ Ρ„ΡƒΠ½ΠΊΡ†ΠΈΠΈ)

    Наша Ρ†Π΅Π»ΡŒ — ΠΏΠΎΡΡ‚Ρ€ΠΎΠΈΡ‚ΡŒ эскиз Π³Ρ€Π°Ρ„ΠΈΠΊΠ° Π΄Ρ€ΠΎΠ±Π½ΠΎ-ΠΊΠ²Π°Π΄Ρ€Π°Ρ‚ΠΈΡ‡Π½ΠΎΠΉ Ρ„ΡƒΠ½ΠΊΡ†ΠΈΠΈ. Для ΠΏΡ€ΠΈΠΌΠ΅Ρ€Π° возьмСм ΡƒΠΆΠ΅ Π·Π½Π°ΠΊΠΎΠΌΡƒΡŽ Π½Π°ΠΌ Ρ„ΡƒΠ½ΠΊΡ†ΠΈΡŽ:

    Π—Π°Π΄Π°Π½Π° дробная функция, Π² числитСлС ΠΈ Π·Π½Π°ΠΌΠ΅Π½Π°Ρ‚Π΅Π»Π΅ ΠΊΠΎΡ‚ΠΎΡ€ΠΎΠΉ стоят ΠΊΠ²Π°Π΄Ρ€Π°Ρ‚ΠΈΡ‡Π½Ρ‹Π΅ Ρ„ΡƒΠ½ΠΊΡ†ΠΈΠΈ.

    ΠœΠ΅Ρ‚ΠΎΠ΄ΠΈΠΊΠ° построСния эскиза Ρ‚Π°ΠΊΠΎΠ²Π°:

    1. Π’Ρ‹Π΄Π΅Π»ΠΈΠΌ ΠΈΠ½Ρ‚Π΅Ρ€Π²Π°Π»Ρ‹ знакопостоянства ΠΈ ΠΎΠΏΡ€Π΅Π΄Π΅Π»ΠΈΠΌ Π½Π° ΠΊΠ°ΠΆΠ΄ΠΎΠΌ Π·Π½Π°ΠΊ Ρ„ΡƒΠ½ΠΊΡ†ΠΈΠΈ (рисунок 1)

    ΠœΡ‹ ΠΏΠΎΠ΄Ρ€ΠΎΠ±Π½ΠΎ рассматривали ΠΈ выяснили, Ρ‡Ρ‚ΠΎ функция, нСпрСрывная Π² ΠžΠ”Π—, ΠΌΠΎΠΆΠ΅Ρ‚ ΡΠΌΠ΅Π½ΠΈΡ‚ΡŒ Π·Π½Π°ΠΊ Ρ‚ΠΎΠ»ΡŒΠΊΠΎ ΠΏΡ€ΠΈ ΠΏΠ΅Ρ€Π΅Ρ…ΠΎΠ΄Π΅ Π°Ρ€Π³ΡƒΠΌΠ΅Π½Ρ‚Π° Ρ‡Π΅Ρ€Π΅Π· ΠΊΠΎΡ€Π½ΠΈ ΠΈ Ρ‚ΠΎΡ‡ΠΊΠΈ Ρ€Π°Π·Ρ€Ρ‹Π²Π° ΠžΠ”Π—.

    Заданная функция Ρƒ Π½Π΅ΠΏΡ€Π΅Ρ€Ρ‹Π²Π½Π° Π² своСй ΠžΠ”Π—, ΡƒΠΊΠ°ΠΆΠ΅ΠΌ ΠžΠ”Π—:

    НайдСм ΠΊΠΎΡ€Π½ΠΈ:

    Π’Ρ‹Π΄Π΅Π»ΠΈΠΌ ΠΈΠ½Ρ‚Π΅Ρ€Π²Π°Π»Ρ‹ знакопостоянства. ΠœΡ‹ нашли ΠΊΠΎΡ€Π½ΠΈ Ρ„ΡƒΠ½ΠΊΡ†ΠΈΠΈ ΠΈ Ρ‚ΠΎΡ‡ΠΊΠΈ Ρ€Π°Π·Ρ€Ρ‹Π²Π° области опрСдСлСния — ΠΊΠΎΡ€Π½ΠΈ знамСнатСля. Π’Π°ΠΆΠ½ΠΎ ΠΎΡ‚ΠΌΠ΅Ρ‚ΠΈΡ‚ΡŒ, Ρ‡Ρ‚ΠΎ Π²Π½ΡƒΡ‚Ρ€ΠΈ ΠΊΠ°ΠΆΠ΄ΠΎΠ³ΠΎ ΠΈΠ½Ρ‚Π΅Ρ€Π²Π°Π»Π° функция сохраняСт Π·Π½Π°ΠΊ.

    Рис. 1. Π˜Π½Ρ‚Π΅Ρ€Π²Π°Π»Ρ‹ знакопостоянства Ρ„ΡƒΠ½ΠΊΡ†ΠΈΠΈ

    Π§Ρ‚ΠΎΠ±Ρ‹ ΠΎΠΏΡ€Π΅Π΄Π΅Π»ΠΈΡ‚ΡŒ Π·Π½Π°ΠΊ Ρ„ΡƒΠ½ΠΊΡ†ΠΈΠΈ Π½Π° ΠΊΠ°ΠΆΠ΄ΠΎΠΌ ΠΈΠ½Ρ‚Π΅Ρ€Π²Π°Π»Π΅, ΠΌΠΎΠΆΠ½ΠΎ Π²Π·ΡΡ‚ΡŒ Π»ΡŽΠ±ΡƒΡŽ Ρ‚ΠΎΡ‡ΠΊΡƒ, ΠΏΡ€ΠΈΠ½Π°Π΄Π»Π΅ΠΆΠ°Ρ‰ΡƒΡŽ ΠΈΠ½Ρ‚Π΅Ρ€Π²Π°Π»Ρƒ, ΠΏΠΎΠ΄ΡΡ‚Π°Π²ΠΈΡ‚ΡŒ Π΅Π΅ Π² Ρ„ΡƒΠ½ΠΊΡ†ΠΈΡŽ ΠΈ ΠΎΠΏΡ€Π΅Π΄Π΅Π»ΠΈΡ‚ΡŒ Π΅Π΅ Π·Π½Π°ΠΊ. НапримСр:

    На ΠΈΠ½Ρ‚Π΅Ρ€Π²Π°Π»Π΅ функция ΠΈΠΌΠ΅Π΅Ρ‚ Π·Π½Π°ΠΊ плюс

    На ΠΈΠ½Ρ‚Π΅Ρ€Π²Π°Π»Π΅ функция ΠΈΠΌΠ΅Π΅Ρ‚ Π·Π½Π°ΠΊ минус.

    Π’ этом прСимущСство ΠΌΠ΅Ρ‚ΠΎΠ΄Π° ΠΈΠ½Ρ‚Π΅Ρ€Π²Π°Π»ΠΎΠ²: ΠΌΡ‹ опрСдСляСм Π·Π½Π°ΠΊ Π² СдинствСнной ΠΏΡ€ΠΎΠ±Π½ΠΎΠΉ Ρ‚ΠΎΡ‡ΠΊΠ΅ ΠΈ Π·Π°ΠΊΠ»ΡŽΡ‡Π°Π΅ΠΌ, Ρ‡Ρ‚ΠΎ функция Π±ΡƒΠ΄Π΅Ρ‚ ΠΈΠΌΠ΅Ρ‚ΡŒ Ρ‚Π°ΠΊΠΎΠΉ ΠΆΠ΅ Π·Π½Π°ΠΊ Π½Π° всСм Π²Ρ‹Π±Ρ€Π°Π½Π½ΠΎΠΌ ΠΈΠ½Ρ‚Π΅Ρ€Π²Π°Π»Π΅.

    Однако ΠΌΠΎΠΆΠ½ΠΎ Π²Ρ‹ΡΡ‚Π°Π²Π»ΡΡ‚ΡŒ Π·Π½Π°ΠΊΠΈ автоматичСски, Π½Π΅ высчитывая Π·Π½Π°Ρ‡Π΅Π½ΠΈΠΉ Ρ„ΡƒΠ½ΠΊΡ†ΠΈΠΈ, для этого ΠΎΠΏΡ€Π΅Π΄Π΅Π»ΠΈΡ‚ΡŒ Π·Π½Π°ΠΊ Π½Π° ΠΊΡ€Π°ΠΉΠ½Π΅ΠΌ ΠΈΠ½Ρ‚Π΅Ρ€Π²Π°Π»Π΅, Π° Π΄Π°Π»Π΅Π΅ Ρ‡Π΅Ρ€Π΅Π΄ΠΎΠ²Π°Ρ‚ΡŒ Π·Π½Π°ΠΊΠΈ.

    1. ΠŸΠΎΡΡ‚Ρ€ΠΎΠΈΠΌ Π³Ρ€Π°Ρ„ΠΈΠΊ Π² окрСстности ΠΊΠ°ΠΆΠ΄ΠΎΠ³ΠΎ корня. Напомним, Ρ‡Ρ‚ΠΎ ΠΊΠΎΡ€Π½ΠΈ Π΄Π°Π½Π½ΠΎΠΉ Ρ„ΡƒΠ½ΠΊΡ†ΠΈΠΈ ΠΈ :

    Рис. 2. Π“Ρ€Π°Ρ„ΠΈΠΊ Π² окрСстностях ΠΊΠΎΡ€Π½Π΅ΠΉ

    ΠŸΠΎΡΠΊΠΎΠ»ΡŒΠΊΡƒ Π² Ρ‚ΠΎΡ‡ΠΊΠ΅ Π·Π½Π°ΠΊ Ρ„ΡƒΠ½ΠΊΡ†ΠΈΠΈ мСняСтся с плюса Π½Π° минус, Ρ‚ΠΎ кривая сначала находится Π½Π°Π΄ осью, ΠΏΠΎΡ‚ΠΎΠΌ ΠΏΡ€ΠΎΡ…ΠΎΠ΄ΠΈΡ‚ Ρ‡Π΅Ρ€Π΅Π· ноль ΠΈ Π΄Π°Π»Π΅Π΅ располоТСна ΠΏΠΎΠ΄ осью Ρ…. Π’ Ρ‚ΠΎΡ‡ΠΊΠ΅ Π½Π°ΠΎΠ±ΠΎΡ€ΠΎΡ‚.

    2. ΠŸΠΎΡΡ‚Ρ€ΠΎΠΈΠΌ Π³Ρ€Π°Ρ„ΠΈΠΊ Π² окрСстности ΠΊΠ°ΠΆΠ΄ΠΎΠ³ΠΎ Ρ€Π°Π·Ρ€Ρ‹Π²Π° ΠžΠ”Π—. Напомним, Ρ‡Ρ‚ΠΎ ΠΊΠΎΡ€Π½ΠΈ знамСнатСля Π΄Π°Π½Π½ΠΎΠΉ Ρ„ΡƒΠ½ΠΊΡ†ΠΈΠΈ ΠΈ :

    Рис. 3. Π“Ρ€Π°Ρ„ΠΈΠΊ Ρ„ΡƒΠ½ΠΊΡ†ΠΈΠΈ Π² окрСстностях Ρ‚ΠΎΡ‡Π΅ΠΊ Ρ€Π°Π·Ρ€Ρ‹Π²Π° ΠžΠ”Π—

    Когда ΠΈΠ»ΠΈ Π·Π½Π°ΠΌΠ΅Π½Π°Ρ‚Π΅Π»ΡŒ Π΄Ρ€ΠΎΠ±ΠΈ практичСски Ρ€Π°Π²Π΅Π½ Π½ΡƒΠ»ΡŽ, Π·Π½Π°Ρ‡ΠΈΡ‚, ΠΊΠΎΠ³Π΄Π° Π·Π½Π°Ρ‡Π΅Π½ΠΈΠ΅ Π°Ρ€Π³ΡƒΠΌΠ΅Π½Ρ‚Π° стрСмится ΠΊ этим числам, Π·Π½Π°Ρ‡Π΅Π½ΠΈΠ΅ Π΄Ρ€ΠΎΠ±ΠΈ стрСмится ΠΊ бСсконСчности. Π’ Π΄Π°Π½Π½ΠΎΠΌ случаС, ΠΊΠΎΠ³Π΄Π° Π°Ρ€Π³ΡƒΠΌΠ΅Π½Ρ‚ ΠΏΠΎΠ΄Ρ…ΠΎΠ΄ΠΈΡ‚ ΠΊ Ρ‚Ρ€ΠΎΠΉΠΊΠ΅ слСва функция ΠΏΠΎΠ»ΠΎΠΆΠΈΡ‚Π΅Π»ΡŒΠ½Π° ΠΈ стрСмится ΠΊ плюс бСсконСчности, справа функция ΠΎΡ‚Ρ€ΠΈΡ†Π°Ρ‚Π΅Π»ΡŒΠ½Π° ΠΈ Π²Ρ‹Ρ…ΠΎΠ΄ΠΈΡ‚ ΠΈΠ· минус бСсконСчности. Около Ρ‡Π΅Ρ‚Π²Π΅Ρ€ΠΊΠΈ Π½Π°ΠΎΠ±ΠΎΡ€ΠΎΡ‚, слСва функция стрСмится ΠΊ минус бСсконСчности, Π° справа Π²Ρ‹Ρ…ΠΎΠ΄ΠΈΡ‚ ΠΈΠ· плюс бСсконСчности.

    Богласно построСнному эскизу ΠΌΡ‹ ΠΌΠΎΠΆΠ΅ΠΌ Π² Π½Π΅ΠΊΠΎΡ‚ΠΎΡ€Ρ‹Ρ… ΠΏΡ€ΠΎΠΌΠ΅ΠΆΡƒΡ‚ΠΊΠ°Ρ… ΡƒΠ³Π°Π΄Π°Ρ‚ΡŒ Ρ…Π°Ρ€Π°ΠΊΡ‚Π΅Ρ€ повСдСния Ρ„ΡƒΠ½ΠΊΡ†ΠΈΠΈ.

    Рис. 4. Эскиз Π³Ρ€Π°Ρ„ΠΈΠΊΠ° Ρ„ΡƒΠ½ΠΊΡ†ΠΈΠΈ

    Рассмотрим ΡΠ»Π΅Π΄ΡƒΡŽΡ‰ΡƒΡŽ Π²Π°ΠΆΠ½ΡƒΡŽ Π·Π°Π΄Π°Ρ‡Ρƒ — ΠΏΠΎΡΡ‚Ρ€ΠΎΠΈΡ‚ΡŒ эскиз Π³Ρ€Π°Ρ„ΠΈΠΊΠ° Ρ„ΡƒΠ½ΠΊΡ†ΠΈΠΈ Π² окрСстностях бСсконСчно ΡƒΠ΄Π°Π»Π΅Π½Π½Ρ‹Ρ… Ρ‚ΠΎΡ‡Π΅ΠΊ, Ρ‚.Π΅. ΠΊΠΎΠ³Π΄Π° Π°Ρ€Π³ΡƒΠΌΠ΅Π½Ρ‚ стрСмится ΠΊ плюс ΠΈΠ»ΠΈ минус бСсконСчности. ΠŸΠΎΡΡ‚ΠΎΡΠ½Π½Ρ‹ΠΌΠΈ слагаСмыми ΠΏΡ€ΠΈ этом ΠΌΠΎΠΆΠ½ΠΎ ΠΏΡ€Π΅Π½Π΅Π±Ρ€Π΅Ρ‡ΡŒ. ИмССм:

    Иногда ΠΌΠΎΠΆΠ½ΠΎ Π²ΡΡ‚Ρ€Π΅Ρ‚ΠΈΡ‚ΡŒ Ρ‚Π°ΠΊΡƒΡŽ запись Π΄Π°Π½Π½ΠΎΠ³ΠΎ Ρ„Π°ΠΊΡ‚Π°:

    Рис. 5. Эскиз Π³Ρ€Π°Ρ„ΠΈΠΊΠ° Ρ„ΡƒΠ½ΠΊΡ†ΠΈΠΈ Π² окрСстностях бСсконСчно ΡƒΠ΄Π°Π»Π΅Π½Π½Ρ‹Ρ… Ρ‚ΠΎΡ‡Π΅ΠΊ

    ΠœΡ‹ ΠΏΠΎΠ»ΡƒΡ‡ΠΈΠ»ΠΈ ΠΏΡ€ΠΈΠ±Π»ΠΈΠ·ΠΈΡ‚Π΅Π»ΡŒΠ½Ρ‹ΠΉ Ρ…Π°Ρ€Π°ΠΊΡ‚Π΅Ρ€ повСдСния Ρ„ΡƒΠ½ΠΊΡ†ΠΈΠΈ Π½Π° всСй Π΅Π΅ области опрСдСлСния, Π΄Π°Π»Π΅Π΅ Π½ΡƒΠΆΠ½ΠΎ ΡƒΡ‚ΠΎΡ‡Π½ΡΡ‚ΡŒ построСния с ΠΏΡ€ΠΈΠΌΠ΅Π½Π΅Π½ΠΈΠ΅ΠΌ ΠΏΡ€ΠΎΠΈΠ·Π²ΠΎΠ΄Π½ΠΎΠΉ.

    ΠŸΡ€ΠΈΠΌΠ΅Ρ€ 1 — ΠΏΠΎΡΡ‚Ρ€ΠΎΠΈΡ‚ΡŒ эскиз Π³Ρ€Π°Ρ„ΠΈΠΊΠ° Ρ„ΡƒΠ½ΠΊΡ†ΠΈΠΈ:

    ИмССм Ρ‚Ρ€ΠΈ Ρ‚ΠΎΡ‡ΠΊΠΈ, ΠΏΡ€ΠΈ ΠΏΠ΅Ρ€Π΅Ρ…ΠΎΠ΄Π΅ Π°Ρ€Π³ΡƒΠΌΠ΅Π½Ρ‚Π° Ρ‡Π΅Ρ€Π΅Π· ΠΊΠΎΡ‚ΠΎΡ€Ρ‹Π΅ функция ΠΌΠΎΠΆΠ΅Ρ‚ ΠΌΠ΅Π½ΡΡ‚ΡŒ Π·Π½Π°ΠΊ.

    ΠžΠΏΡ€Π΅Π΄Π΅Π»ΡΠ΅ΠΌ Π·Π½Π°ΠΊΠΈ Ρ„ΡƒΠ½ΠΊΡ†ΠΈΠΈ Π½Π° ΠΊΠ°ΠΆΠ΄ΠΎΠΌ ΠΈΠ½Ρ‚Π΅Ρ€Π²Π°Π»Π΅. ИмССм плюс Π½Π° ΠΊΡ€Π°ΠΉΠ½Π΅ΠΌ ΠΏΡ€Π°Π²ΠΎΠΌ ΠΈΠ½Ρ‚Π΅Ρ€Π²Π°Π»Π΅, Π΄Π°Π»Π΅Π΅ Π·Π½Π°ΠΊΠΈ Ρ‡Π΅Ρ€Π΅Π΄ΡƒΡŽΡ‚ΡΡ, Ρ‚Π°ΠΊ ΠΊΠ°ΠΊ всС ΠΊΠΎΡ€Π½ΠΈ ΠΈΠΌΠ΅ΡŽΡ‚ ΠΏΠ΅Ρ€Π²ΡƒΡŽ ΡΡ‚Π΅ΠΏΠ΅Π½ΡŒ.

    Π‘Ρ‚Ρ€ΠΎΠΈΠΌ эскиз Π³Ρ€Π°Ρ„ΠΈΠΊΠ° Π² окрСстностях ΠΊΠΎΡ€Π½Π΅ΠΉ ΠΈ Ρ‚ΠΎΡ‡Π΅ΠΊ Ρ€Π°Π·Ρ€Ρ‹Π²Π° ΠžΠ”Π—. ИмССм: ΠΏΠΎΡΠΊΠΎΠ»ΡŒΠΊΡƒ Π² Ρ‚ΠΎΡ‡ΠΊΠ΅ Π·Π½Π°ΠΊ Ρ„ΡƒΠ½ΠΊΡ†ΠΈΠΈ мСняСтся с плюса Π½Π° минус, Ρ‚ΠΎ кривая сначала находится Π½Π°Π΄ осью, ΠΏΠΎΡ‚ΠΎΠΌ ΠΏΡ€ΠΎΡ…ΠΎΠ΄ΠΈΡ‚ Ρ‡Π΅Ρ€Π΅Π· ноль ΠΈ Π΄Π°Π»Π΅Π΅ располоТСна ΠΏΠΎΠ΄ осью Ρ…. Когда ΠΈΠ»ΠΈ Π·Π½Π°ΠΌΠ΅Π½Π°Ρ‚Π΅Π»ΡŒ Π΄Ρ€ΠΎΠ±ΠΈ практичСски Ρ€Π°Π²Π΅Π½ Π½ΡƒΠ»ΡŽ, Π·Π½Π°Ρ‡ΠΈΡ‚, ΠΊΠΎΠ³Π΄Π° Π·Π½Π°Ρ‡Π΅Π½ΠΈΠ΅ Π°Ρ€Π³ΡƒΠΌΠ΅Π½Ρ‚Π° стрСмится ΠΊ этим числам, Π·Π½Π°Ρ‡Π΅Π½ΠΈΠ΅ Π΄Ρ€ΠΎΠ±ΠΈ стрСмится ΠΊ бСсконСчности. Π’ Π΄Π°Π½Π½ΠΎΠΌ случаС, ΠΊΠΎΠ³Π΄Π° Π°Ρ€Π³ΡƒΠΌΠ΅Π½Ρ‚ ΠΏΠΎΠ΄Ρ…ΠΎΠ΄ΠΈΡ‚ ΠΊ минус Π΄Π²ΡƒΠΌ слСва функция ΠΎΡ‚Ρ€ΠΈΡ†Π°Ρ‚Π΅Π»ΡŒΠ½Π° ΠΈ стрСмится ΠΊ минус бСсконСчности, справа функция ΠΏΠΎΠ»ΠΎΠΆΠΈΡ‚Π΅Π»ΡŒΠ½Π° ΠΈ Π²Ρ‹Ρ…ΠΎΠ΄ΠΈΡ‚ ΠΈΠ· плюс бСсконСчности. Около Π΄Π²ΠΎΠΉΠΊΠΈ Π°Π½Π°Π»ΠΎΠ³ΠΈΡ‡Π½ΠΎ.

    НайдСм ΠΏΡ€ΠΎΠΈΠ·Π²ΠΎΠ΄Π½ΡƒΡŽ Ρ„ΡƒΠ½ΠΊΡ†ΠΈΠΈ:

    ΠžΡ‡Π΅Π²ΠΈΠ΄Π½ΠΎ, Ρ‡Ρ‚ΠΎ производная всСгда мСньшС нуля, ΡΠ»Π΅Π΄ΠΎΠ²Π°Ρ‚Π΅Π»ΡŒΠ½ΠΎ, функция ΡƒΠ±Ρ‹Π²Π°Π΅Ρ‚ Π½Π° всСх участках. Π’Π°ΠΊ, Π½Π° участкС ΠΎΡ‚ минус бСсконСчности Π΄ΠΎ минус Π΄Π²ΡƒΡ… функция ΡƒΠ±Ρ‹Π²Π°Π΅Ρ‚ ΠΎΡ‚ нуля Π΄ΠΎ минус бСсконСчности; Π½Π° участкС ΠΎΡ‚ минус Π΄Π²ΡƒΡ… Π΄ΠΎ нуля функция ΡƒΠ±Ρ‹Π²Π°Π΅Ρ‚ ΠΎΡ‚ плюс бСсконСчности Π΄ΠΎ нуля; Π½Π° участкС ΠΎΡ‚ нуля Π΄ΠΎ Π΄Π²ΡƒΡ… функция ΡƒΠ±Ρ‹Π²Π°Π΅Ρ‚ ΠΎΡ‚ нуля Π΄ΠΎ минус бСсконСчности; Π½Π° участкС ΠΎΡ‚ Π΄Π²ΡƒΡ… Π΄ΠΎ плюс бСсконСчности функция ΡƒΠ±Ρ‹Π²Π°Π΅Ρ‚ ΠΎΡ‚ плюс бСсконСчности Π΄ΠΎ нуля.

    ΠŸΡ€ΠΎΠΈΠ»Π»ΡŽΡΡ‚Ρ€ΠΈΡ€ΡƒΠ΅ΠΌ:

    Рис. 6. Эскиз Π³Ρ€Π°Ρ„ΠΈΠΊΠ° Ρ„ΡƒΠ½ΠΊΡ†ΠΈΠΈ ΠΊ ΠΏΡ€ΠΈΠΌΠ΅Ρ€Ρƒ 1

    ΠŸΡ€ΠΈΠΌΠ΅Ρ€ 2 — ΠΏΠΎΡΡ‚Ρ€ΠΎΠΈΡ‚ΡŒ эскиз Π³Ρ€Π°Ρ„ΠΈΠΊΠ° Ρ„ΡƒΠ½ΠΊΡ†ΠΈΠΈ:

    Π‘Ρ‚Ρ€ΠΎΠΈΠΌ эскиз Π³Ρ€Π°Ρ„ΠΈΠΊΠ° Ρ„ΡƒΠ½ΠΊΡ†ΠΈΠΈ Π±Π΅Π· использования ΠΏΡ€ΠΎΠΈΠ·Π²ΠΎΠ΄Π½ΠΎΠΉ.

    Π‘Π½Π°Ρ‡Π°Π»Π° исслСдуСм Π·Π°Π΄Π°Π½Π½ΡƒΡŽ Ρ„ΡƒΠ½ΠΊΡ†ΠΈΡŽ:

    ИмССм Π΅Π΄ΠΈΠ½ΡΡ‚Π²Π΅Π½Π½ΡƒΡŽ Ρ‚ΠΎΡ‡ΠΊΡƒ, ΠΏΡ€ΠΈ ΠΏΠ΅Ρ€Π΅Ρ…ΠΎΠ΄Π΅ Π°Ρ€Π³ΡƒΠΌΠ΅Π½Ρ‚Π° Ρ‡Π΅Ρ€Π΅Π· ΠΊΠΎΡ‚ΠΎΡ€ΡƒΡŽ функция ΠΌΠΎΠΆΠ΅Ρ‚ ΠΌΠ΅Π½ΡΡ‚ΡŒ Π·Π½Π°ΠΊ.

    ΠžΡ‚ΠΌΠ΅Ρ‚ΠΈΠΌ, Ρ‡Ρ‚ΠΎ заданная функция нСчСтная.

    ΠžΠΏΡ€Π΅Π΄Π΅Π»ΡΠ΅ΠΌ Π·Π½Π°ΠΊΠΈ Ρ„ΡƒΠ½ΠΊΡ†ΠΈΠΈ Π½Π° ΠΊΠ°ΠΆΠ΄ΠΎΠΌ ΠΈΠ½Ρ‚Π΅Ρ€Π²Π°Π»Π΅. ИмССм плюс Π½Π° ΠΊΡ€Π°ΠΉΠ½Π΅ΠΌ ΠΏΡ€Π°Π²ΠΎΠΌ ΠΈΠ½Ρ‚Π΅Ρ€Π²Π°Π»Π΅, Π΄Π°Π»Π΅Π΅ Π·Π½Π°ΠΊ мСняСтся, Ρ‚Π°ΠΊ ΠΊΠ°ΠΊ ΠΊΠΎΡ€Π΅Π½ΡŒ ΠΈΠΌΠ΅Π΅Ρ‚ ΠΏΠ΅Ρ€Π²ΡƒΡŽ ΡΡ‚Π΅ΠΏΠ΅Π½ΡŒ.

    Π‘Ρ‚Ρ€ΠΎΠΈΠΌ эскиз Π³Ρ€Π°Ρ„ΠΈΠΊΠ° Π² окрСстностях корня. ИмССм: ΠΏΠΎΡΠΊΠΎΠ»ΡŒΠΊΡƒ Π² Ρ‚ΠΎΡ‡ΠΊΠ΅ Π·Π½Π°ΠΊ Ρ„ΡƒΠ½ΠΊΡ†ΠΈΠΈ мСняСтся с минуса Π½Π° плюс, Ρ‚ΠΎ кривая сначала находится ΠΏΠΎΠ΄ осью, ΠΏΠΎΡ‚ΠΎΠΌ ΠΏΡ€ΠΎΡ…ΠΎΠ΄ΠΈΡ‚ Ρ‡Π΅Ρ€Π΅Π· ноль ΠΈ Π΄Π°Π»Π΅Π΅ располоТСна Π½Π°Π΄ осью Ρ….

    Π’Π΅ΠΏΠ΅Ρ€ΡŒ строим эскиз Π³Ρ€Π°Ρ„ΠΈΠΊΠ° Ρ„ΡƒΠ½ΠΊΡ†ΠΈΠΈ Π² окрСстностях бСсконСчно ΡƒΠ΄Π°Π»Π΅Π½Π½Ρ‹Ρ… Ρ‚ΠΎΡ‡Π΅ΠΊ, Ρ‚.Π΅. ΠΊΠΎΠ³Π΄Π° Π°Ρ€Π³ΡƒΠΌΠ΅Π½Ρ‚ стрСмится ΠΊ плюс ΠΈΠ»ΠΈ минус бСсконСчности. ΠŸΠΎΡΡ‚ΠΎΡΠ½Π½Ρ‹ΠΌΠΈ слагаСмыми ΠΏΡ€ΠΈ этом ΠΌΠΎΠΆΠ½ΠΎ ΠΏΡ€Π΅Π½Π΅Π±Ρ€Π΅Ρ‡ΡŒ. ИмССм:

    ПослС выполнСния Π²Ρ‹ΡˆΠ΅ΠΏΠ΅Ρ€Π΅Ρ‡ΠΈΡΠ»Π΅Π½Π½Ρ‹Ρ… дСйствий ΠΌΡ‹ ΡƒΠΆΠ΅ прСдставляСм сСбС Π³Ρ€Π°Ρ„ΠΈΠΊ Ρ„ΡƒΠ½ΠΊΡ†ΠΈΠΈ, Π½ΠΎ трСбуСтся ΡƒΡ‚ΠΎΡ‡Π½ΠΈΡ‚ΡŒ Π΅Π³ΠΎ с ΠΏΠΎΠΌΠΎΡ‰ΡŒΡŽ ΠΏΡ€ΠΎΠΈΠ·Π²ΠΎΠ΄Π½ΠΎΠΉ.

    НайдСм ΠΏΡ€ΠΎΠΈΠ·Π²ΠΎΠ΄Π½ΡƒΡŽ Ρ„ΡƒΠ½ΠΊΡ†ΠΈΠΈ:

    ВыдСляСм ΠΈΠ½Ρ‚Π΅Ρ€Π²Π°Π»Ρ‹ знакопостоянства ΠΏΡ€ΠΎΠΈΠ·Π²ΠΎΠ΄Π½ΠΎΠΉ: ΠΏΡ€ΠΈ . ΠžΠ”Π— здСсь . Π’Π°ΠΊΠΈΠΌ ΠΎΠ±Ρ€Π°Π·ΠΎΠΌ, ΠΈΠΌΠ΅Π΅ΠΌ Ρ‚Ρ€ΠΈ ΠΈΠ½Ρ‚Π΅Ρ€Π²Π°Π»Π° знакопостоянства ΠΏΡ€ΠΎΠΈΠ·Π²ΠΎΠ΄Π½ΠΎΠΉ ΠΈ Ρ‚Ρ€ΠΈ участка монотонности исходной Ρ„ΡƒΠ½ΠΊΡ†ΠΈΠΈ. ΠžΠΏΡ€Π΅Π΄Π΅Π»ΠΈΠΌ Π·Π½Π°ΠΊΠΈ ΠΏΡ€ΠΎΠΈΠ·Π²ΠΎΠ΄Π½ΠΎΠΉ Π½Π° ΠΊΠ°ΠΆΠ΄ΠΎΠΌ ΠΈΠ½Ρ‚Π΅Ρ€Π²Π°Π»Π΅. Когда производная ΠΏΠΎΠ»ΠΎΠΆΠΈΡ‚Π΅Π»ΡŒΠ½Π°, функция возрастаСт; ΠΊΠΎΠ³Π΄Π° производная ΠΎΡ‚Ρ€ΠΈΡ†Π°Ρ‚Π΅Π»ΡŒΠ½Π°, функция ΡƒΠ±Ρ‹Π²Π°Π΅Ρ‚. ΠŸΡ€ΠΈ этом — Ρ‚ΠΎΡ‡ΠΊΠ° ΠΌΠΈΠ½ΠΈΠΌΡƒΠΌ, Ρ‚.ΠΊ. производная мСняСт Π·Π½Π°ΠΊ с минуса Π½Π° плюс; Π½Π°ΠΎΠ±ΠΎΡ€ΠΎΡ‚, Ρ‚ΠΎΡ‡ΠΊΠ° максимума.

    ΠŸΠΎΡΡ‚Ρ€ΠΎΠΈΡ‚ΡŒ Ρ„ΡƒΠ½ΠΊΡ†ΠΈΡŽ

    ΠœΡ‹ ΠΏΡ€Π΅Π΄Π»Π°Π³Π°Π΅ΠΌ Π²Π°ΡˆΠ΅ΠΌΡƒ вниманию сСрвис ΠΏΠΎ ΠΏΠΎΡ‚Ρ€ΠΎΠ΅Π½ΠΈΡŽ Π³Ρ€Π°Ρ„ΠΈΠΊΠΎΠ² Ρ„ΡƒΠ½ΠΊΡ†ΠΈΠΉ ΠΎΠ½Π»Π°ΠΉΠ½, всС ΠΏΡ€Π°Π²Π° Π½Π° ΠΊΠΎΡ‚ΠΎΡ€Ρ‹ΠΉ ΠΏΡ€ΠΈΠ½Π°Π΄Π»Π΅ΠΆΠ°Ρ‚ ΠΊΠΎΠΌΠΏΠ°Π½ΠΈΠΈ Desmos . Для Π²Π²ΠΎΠ΄Π° Ρ„ΡƒΠ½ΠΊΡ†ΠΈΠΉ Π²ΠΎΡΠΏΠΎΠ»ΡŒΠ·ΡƒΠΉΡ‚Π΅ΡΡŒ Π»Π΅Π²ΠΎΠΉ ΠΊΠΎΠ»ΠΎΠ½ΠΊΠΎΠΉ. Π’Π²ΠΎΠ΄ΠΈΡ‚ΡŒ ΠΌΠΎΠΆΠ½ΠΎ Π²Ρ€ΡƒΡ‡Π½ΡƒΡŽ Π»ΠΈΠ±ΠΎ с ΠΏΠΎΠΌΠΎΡ‰ΡŒΡŽ Π²ΠΈΡ€Ρ‚ΡƒΠ°Π»ΡŒΠ½ΠΎΠΉ ΠΊΠ»Π°Π²ΠΈΠ°Ρ‚ΡƒΡ€Ρ‹ Π²Π½ΠΈΠ·Ρƒ ΠΎΠΊΠ½Π°. Для увСличСния ΠΎΠΊΠ½Π° с Π³Ρ€Π°Ρ„ΠΈΠΊΠΎΠΌ ΠΌΠΎΠΆΠ½ΠΎ ΡΠΊΡ€Ρ‹Ρ‚ΡŒ ΠΊΠ°ΠΊ Π»Π΅Π²ΡƒΡŽ ΠΊΠΎΠ»ΠΎΠ½ΠΊΡƒ, Ρ‚Π°ΠΊ ΠΈ Π²ΠΈΡ€Ρ‚ΡƒΠ°Π»ΡŒΠ½ΡƒΡŽ ΠΊΠ»Π°Π²ΠΈΠ°Ρ‚ΡƒΡ€Ρƒ.

    ΠŸΡ€Π΅ΠΈΠΌΡƒΡ‰Π΅ΡΡ‚Π²Π° построСния Π³Ρ€Π°Ρ„ΠΈΠΊΠΎΠ² ΠΎΠ½Π»Π°ΠΉΠ½
    • Π’ΠΈΠ·ΡƒΠ°Π»ΡŒΠ½ΠΎΠ΅ ΠΎΡ‚ΠΎΠ±Ρ€Π°ΠΆΠ΅Π½ΠΈΠ΅ Π²Π²ΠΎΠ΄ΠΈΠΌΡ‹Ρ… Ρ„ΡƒΠ½ΠΊΡ†ΠΈΠΉ
    • ΠŸΠΎΡΡ‚Ρ€ΠΎΠ΅Π½ΠΈΠ΅ ΠΎΡ‡Π΅Π½ΡŒ слоТных Π³Ρ€Π°Ρ„ΠΈΠΊΠΎΠ²
    • ΠŸΠΎΡΡ‚Ρ€ΠΎΠ΅Π½ΠΈΠ΅ Π³Ρ€Π°Ρ„ΠΈΠΊΠΎΠ², Π·Π°Π΄Π°Π½Π½Ρ‹Ρ… нСявно (Π½Π°ΠΏΡ€ΠΈΠΌΠ΅Ρ€ эллипс x^2/9+y^2/16=1)
    • Π’ΠΎΠ·ΠΌΠΎΠΆΠ½ΠΎΡΡ‚ΡŒ ΡΠΎΡ…Ρ€Π°Π½ΡΡ‚ΡŒ Π³Ρ€Π°Ρ„ΠΈΠΊΠΈ ΠΈ ΠΏΠΎΠ»ΡƒΡ‡Π°Ρ‚ΡŒ Π½Π° Π½ΠΈΡ… ссылку, которая становится доступной для всСх Π² ΠΈΠ½Ρ‚Π΅Ρ€Π½Π΅Ρ‚Π΅
    • Π£ΠΏΡ€Π°Π²Π»Π΅Π½ΠΈΠ΅ ΠΌΠ°ΡΡˆΡ‚Π°Π±ΠΎΠΌ, Ρ†Π²Π΅Ρ‚ΠΎΠΌ Π»ΠΈΠ½ΠΈΠΉ
    • Π’ΠΎΠ·ΠΌΠΎΠΆΠ½ΠΎΡΡ‚ΡŒ построСния Π³Ρ€Π°Ρ„ΠΈΠΊΠΎΠ² ΠΏΠΎ Ρ‚ΠΎΡ‡ΠΊΠ°ΠΌ, использованиС констант
    • ΠŸΠΎΡΡ‚Ρ€ΠΎΠ΅Π½ΠΈΠ΅ ΠΎΠ΄Π½ΠΎΠ²Ρ€Π΅ΠΌΠ΅Π½Π½ΠΎ Π½Π΅ΡΠΊΠΎΠ»ΡŒΠΊΠΈΡ… Π³Ρ€Π°Ρ„ΠΈΠΊΠΎΠ² Ρ„ΡƒΠ½ΠΊΡ†ΠΈΠΉ
    • ΠŸΠΎΡΡ‚Ρ€ΠΎΠ΅Π½ΠΈΠ΅ Π³Ρ€Π°Ρ„ΠΈΠΊΠΎΠ² Π² полярной систСмС ΠΊΠΎΠΎΡ€Π΄ΠΈΠ½Π°Ρ‚ (ΠΈΡΠΏΠΎΠ»ΡŒΠ·ΡƒΠΉΡ‚Π΅ r ΠΈ ΞΈ(\theta))

    Π‘ Π½Π°ΠΌΠΈ Π»Π΅Π³ΠΊΠΎ Π² Ρ€Π΅ΠΆΠΈΠΌΠ΅ ΠΎΠ½Π»Π°ΠΉΠ½ ΡΡ‚Ρ€ΠΎΠΈΡ‚ΡŒ Π³Ρ€Π°Ρ„ΠΈΠΊΠΈ Ρ€Π°Π·Π»ΠΈΡ‡Π½ΠΎΠΉ слоТности. ΠŸΠΎΡΡ‚Ρ€ΠΎΠ΅Π½ΠΈΠ΅ производится ΠΌΠ³Π½ΠΎΠ²Π΅Π½Π½ΠΎ. БСрвис вострСбован для нахоТдСния Ρ‚ΠΎΡ‡Π΅ΠΊ пСрСсСчСния Ρ„ΡƒΠ½ΠΊΡ†ΠΈΠΉ, для изобраТСния Π³Ρ€Π°Ρ„ΠΈΠΊΠΎΠ² для дальнСйшСго ΠΈΡ… пСрСмСщСния Π² Word Π΄ΠΎΠΊΡƒΠΌΠ΅Π½Ρ‚ Π² качСствС ΠΈΠ»Π»ΡŽΡΡ‚Ρ€Π°Ρ†ΠΈΠΉ ΠΏΡ€ΠΈ Ρ€Π΅ΡˆΠ΅Π½ΠΈΠΈ Π·Π°Π΄Π°Ρ‡, для Π°Π½Π°Π»ΠΈΠ·Π° повСдСнчСских особСнностСй Π³Ρ€Π°Ρ„ΠΈΠΊΠΎΠ² Ρ„ΡƒΠ½ΠΊΡ†ΠΈΠΉ. ΠžΠΏΡ‚ΠΈΠΌΠ°Π»ΡŒΠ½Ρ‹ΠΌ Π±Ρ€Π°ΡƒΠ·Π΅Ρ€ΠΎΠΌ для Ρ€Π°Π±ΠΎΡ‚Ρ‹ с Π³Ρ€Π°Ρ„ΠΈΠΊΠ°ΠΌΠΈ Π½Π° Π΄Π°Π½Π½ΠΎΠΉ страницС сайта являСтся Google Chrome. ΠŸΡ€ΠΈ использовании Π΄Ρ€ΡƒΠ³ΠΈΡ… Π±Ρ€Π°ΡƒΠ·Π΅Ρ€ΠΎΠ² ΠΊΠΎΡ€Ρ€Π΅ΠΊΡ‚Π½ΠΎΡΡ‚ΡŒ Ρ€Π°Π±ΠΎΡ‚Ρ‹ Π½Π΅ гарантируСтся.

    Π’Ρ‹Π±Π΅Ρ€Π΅ΠΌ Π½Π° плоскости ΠΏΡ€ΡΠΌΠΎΡƒΠ³ΠΎΠ»ΡŒΠ½ΡƒΡŽ систСму ΠΊΠΎΠΎΡ€Π΄ΠΈΠ½Π°Ρ‚ ΠΈ Π±ΡƒΠ΄Π΅ΠΌ ΠΎΡ‚ΠΊΠ»Π°Π΄Ρ‹Π²Π°Ρ‚ΡŒ Π½Π° оси абсцисс значСния Π°Ρ€Π³ΡƒΠΌΠ΅Π½Ρ‚Π° Ρ… , Π° Π½Π° оси ΠΎΡ€Π΄ΠΈΠ½Π°Ρ‚ — значСния Ρ„ΡƒΠ½ΠΊΡ†ΠΈΠΈ Ρƒ = f (Ρ…) .

    Π“Ρ€Π°Ρ„ΠΈΠΊΠΎΠΌ Ρ„ΡƒΠ½ΠΊΡ†ΠΈΠΈ y = f(x) называСтся мноТСство всСх Ρ‚ΠΎΡ‡Π΅ΠΊ, Ρƒ ΠΊΠΎΡ‚ΠΎΡ€Ρ‹Ρ… абсциссы ΠΏΡ€ΠΈΠ½Π°Π΄Π»Π΅ΠΆΠ°Ρ‚ области опрСдСлСния Ρ„ΡƒΠ½ΠΊΡ†ΠΈΠΈ, Π° ΠΎΡ€Π΄ΠΈΠ½Π°Ρ‚Ρ‹ Ρ€Π°Π²Π½Ρ‹ ΡΠΎΠΎΡ‚Π²Π΅Ρ‚ΡΡ‚Π²ΡƒΡŽΡ‰ΠΈΠΌ значСниям Ρ„ΡƒΠ½ΠΊΡ†ΠΈΠΈ.

    Π”Ρ€ΡƒΠ³ΠΈΠΌΠΈ словами, Π³Ρ€Π°Ρ„ΠΈΠΊ Ρ„ΡƒΠ½ΠΊΡ†ΠΈΠΈ y = f (Ρ…) — это мноТСство всСх Ρ‚ΠΎΡ‡Π΅ΠΊ плоскости, ΠΊΠΎΠΎΡ€Π΄ΠΈΠ½Π°Ρ‚Ρ‹ Ρ…, Ρƒ ΠΊΠΎΡ‚ΠΎΡ€Ρ‹Ρ… ΡƒΠ΄ΠΎΠ²Π»Π΅Ρ‚Π²ΠΎΡ€ΡΡŽΡ‚ ΡΠΎΠΎΡ‚Π½ΠΎΡˆΠ΅Π½ΠΈΡŽ y = f(x) .

    На рис. 45 ΠΈ 46 ΠΏΡ€ΠΈΠ²Π΅Π΄Π΅Π½Ρ‹ Π³Ρ€Π°Ρ„ΠΈΠΊΠΈ Ρ„ΡƒΠ½ΠΊΡ†ΠΈΠΉ Ρƒ = 2Ρ… + 1 ΠΈ Ρƒ = Ρ… 2 — 2Ρ… .

    Π‘Ρ‚Ρ€ΠΎΠ³ΠΎ говоря, слСдуСт Ρ€Π°Π·Π»ΠΈΡ‡Π°Ρ‚ΡŒ Π³Ρ€Π°Ρ„ΠΈΠΊ Ρ„ΡƒΠ½ΠΊΡ†ΠΈΠΈ (Ρ‚ΠΎΡ‡Π½ΠΎΠ΅ матСматичСскоС ΠΎΠΏΡ€Π΅Π΄Π΅Π»Π΅Π½ΠΈΠ΅ ΠΊΠΎΡ‚ΠΎΡ€ΠΎΠ³ΠΎ Π±Ρ‹Π»ΠΎ Π΄Π°Π½ΠΎ Π²Ρ‹ΡˆΠ΅) ΠΈ Π½Π°Ρ‡Π΅Ρ€Ρ‡Π΅Π½Π½ΡƒΡŽ ΠΊΡ€ΠΈΠ²ΡƒΡŽ, которая всСгда Π΄Π°Π΅Ρ‚ лишь Π±ΠΎΠ»Π΅Π΅ ΠΈΠ»ΠΈ ΠΌΠ΅Π½Π΅Π΅ Ρ‚ΠΎΡ‡Π½Ρ‹ΠΉ эскиз Π³Ρ€Π°Ρ„ΠΈΠΊΠ° (Π΄Π° ΠΈ Ρ‚ΠΎ, ΠΊΠ°ΠΊ ΠΏΡ€Π°Π²ΠΈΠ»ΠΎ, Π½Π΅ всСго Π³Ρ€Π°Ρ„ΠΈΠΊΠ°, Π° лишь Π΅Π³ΠΎ части, располоТСнного Π² ΠΊΠΎΠ½Π΅Ρ‡Π½ΠΎΠΉ части плоскости). Π’ дальнСйшСм, ΠΎΠ΄Π½Π°ΠΊΠΎ, ΠΌΡ‹ ΠΎΠ±Ρ‹Ρ‡Π½ΠΎ Π±ΡƒΠ΄Π΅ΠΌ Π³ΠΎΠ²ΠΎΡ€ΠΈΡ‚ΡŒ Β«Π³Ρ€Π°Ρ„ΠΈΠΊΒ», Π° Π½Π΅ «эскиз Π³Ρ€Π°Ρ„ΠΈΠΊΠ°Β».

    Π‘ ΠΏΠΎΠΌΠΎΡ‰ΡŒΡŽ Π³Ρ€Π°Ρ„ΠΈΠΊΠ° ΠΌΠΎΠΆΠ½ΠΎ Π½Π°Ρ…ΠΎΠ΄ΠΈΡ‚ΡŒ Π·Π½Π°Ρ‡Π΅Π½ΠΈΠ΅ Ρ„ΡƒΠ½ΠΊΡ†ΠΈΠΈ Π² Ρ‚ΠΎΡ‡ΠΊΠ΅. ИмСнно, Ссли Ρ‚ΠΎΡ‡ΠΊΠ° Ρ… = Π° ΠΏΡ€ΠΈΠ½Π°Π΄Π»Π΅ΠΆΠΈΡ‚ области опрСдСлСния Ρ„ΡƒΠ½ΠΊΡ†ΠΈΠΈ y = f(x) , Ρ‚ΠΎ для нахоТдСния числа f(Π°) (Ρ‚. Π΅. значСния Ρ„ΡƒΠ½ΠΊΡ†ΠΈΠΈ Π² Ρ‚ΠΎΡ‡ΠΊΠ΅ Ρ… = Π° ) слСдуСт ΠΏΠΎΡΡ‚ΡƒΠΏΠΈΡ‚ΡŒ Ρ‚Π°ΠΊ. НуТно Ρ‡Π΅Ρ€Π΅Π· Ρ‚ΠΎΡ‡ΠΊΡƒ с абсциссой Ρ… = Π° провСсти ΠΏΡ€ΡΠΌΡƒΡŽ, ΠΏΠ°Ρ€Π°Π»Π»Π΅Π»ΡŒΠ½ΡƒΡŽ оси ΠΎΡ€Π΄ΠΈΠ½Π°Ρ‚; эта прямая пСрСсСчСт Π³Ρ€Π°Ρ„ΠΈΠΊ Ρ„ΡƒΠ½ΠΊΡ†ΠΈΠΈ y = f(x) Π² ΠΎΠ΄Π½ΠΎΠΉ Ρ‚ΠΎΡ‡ΠΊΠ΅; ΠΎΡ€Π΄ΠΈΠ½Π°Ρ‚Π° этой Ρ‚ΠΎΡ‡ΠΊΠΈ ΠΈ Π±ΡƒΠ΄Π΅Ρ‚, Π² силу опрСдСлСния Π³Ρ€Π°Ρ„ΠΈΠΊΠ°, Ρ€Π°Π²Π½Π° f(Π°) (рис. 47).

    НапримСр, для Ρ„ΡƒΠ½ΠΊΡ†ΠΈΠΈ f(Ρ…) = Ρ… 2 — 2x с ΠΏΠΎΠΌΠΎΡ‰ΡŒΡŽ Π³Ρ€Π°Ρ„ΠΈΠΊΠ° (рис. 46) Π½Π°Ρ…ΠΎΠ΄ΠΈΠΌ f(-1) = 3, f(0) = 0, f(1) = -l, f(2) = 0 ΠΈ Ρ‚. Π΄.

    Π“Ρ€Π°Ρ„ΠΈΠΊ Ρ„ΡƒΠ½ΠΊΡ†ΠΈΠΈ наглядно ΠΈΠ»Π»ΡŽΡΡ‚Ρ€ΠΈΡ€ΡƒΠ΅Ρ‚ ΠΏΠΎΠ²Π΅Π΄Π΅Π½ΠΈΠ΅ ΠΈ свойства Ρ„ΡƒΠ½ΠΊΡ†ΠΈΠΈ. НапримСр, ΠΈΠ· рассмотрСния рис. 46 ясно, Ρ‡Ρ‚ΠΎ функция Ρƒ = Ρ… 2 — 2Ρ… ΠΏΡ€ΠΈΠ½ΠΈΠΌΠ°Π΅Ρ‚ ΠΏΠΎΠ»ΠΎΠΆΠΈΡ‚Π΅Π»ΡŒΠ½Ρ‹Π΅ значСния ΠΏΡ€ΠΈ Ρ… ΠΈ ΠΏΡ€ΠΈ Ρ… > 2 , ΠΎΡ‚Ρ€ΠΈΡ†Π°Ρ‚Π΅Π»ΡŒΠ½Ρ‹Π΅ — ΠΏΡ€ΠΈ 0 Ρƒ = Ρ… 2 — 2Ρ… ΠΏΡ€ΠΈΠ½ΠΈΠΌΠ°Π΅Ρ‚ ΠΏΡ€ΠΈ Ρ… = 1 .

    Для построСния Π³Ρ€Π°Ρ„ΠΈΠΊΠ° Ρ„ΡƒΠ½ΠΊΡ†ΠΈΠΈ f(x) Π½ΡƒΠΆΠ½ΠΎ Π½Π°ΠΉΡ‚ΠΈ всС Ρ‚ΠΎΡ‡ΠΊΠΈ плоскости, ΠΊΠΎΠΎΡ€Π΄ΠΈΠ½Π°Ρ‚Ρ‹ Ρ… , Ρƒ ΠΊΠΎΡ‚ΠΎΡ€Ρ‹Ρ… ΡƒΠ΄ΠΎΠ²Π»Π΅Ρ‚Π²ΠΎΡ€ΡΡŽΡ‚ ΡƒΡ€Π°Π²Π½Π΅Π½ΠΈΡŽ y = f(x) . Π’ Π±ΠΎΠ»ΡŒΡˆΠΈΠ½ΡΡ‚Π²Π΅ случаСв это ΡΠ΄Π΅Π»Π°Ρ‚ΡŒ Π½Π΅Π²ΠΎΠ·ΠΌΠΎΠΆΠ½ΠΎ, Ρ‚Π°ΠΊ ΠΊΠ°ΠΊ Ρ‚Π°ΠΊΠΈΡ… Ρ‚ΠΎΡ‡Π΅ΠΊ бСсконСчно ΠΌΠ½ΠΎΠ³ΠΎ. ΠŸΠΎΡΡ‚ΠΎΠΌΡƒ Π³Ρ€Π°Ρ„ΠΈΠΊ Ρ„ΡƒΠ½ΠΊΡ†ΠΈΠΈ ΠΈΠ·ΠΎΠ±Ρ€Π°ΠΆΠ°ΡŽΡ‚ ΠΏΡ€ΠΈΠ±Π»ΠΈΠ·ΠΈΡ‚Π΅Π»ΡŒΠ½ΠΎ — с большСй ΠΈΠ»ΠΈ мСньшСй Ρ‚ΠΎΡ‡Π½ΠΎΡΡ‚ΡŒΡŽ. Π‘Π°ΠΌΡ‹ΠΌ простым являСтся ΠΌΠ΅Ρ‚ΠΎΠ΄ построСния Π³Ρ€Π°Ρ„ΠΈΠΊΠ° ΠΏΠΎ нСскольким Ρ‚ΠΎΡ‡ΠΊΠ°ΠΌ. Он состоит Π² Ρ‚ΠΎΠΌ, Ρ‡Ρ‚ΠΎ Π°Ρ€Π³ΡƒΠΌΠ΅Π½Ρ‚Ρƒ Ρ… ΠΏΡ€ΠΈΠ΄Π°ΡŽΡ‚ ΠΊΠΎΠ½Π΅Ρ‡Π½ΠΎΠ΅ число Π·Π½Π°Ρ‡Π΅Π½ΠΈΠΉ — скаТСм, Ρ… 1 , Ρ… 2 , x 3 ,…, Ρ… k ΠΈ ΡΠΎΡΡ‚Π°Π²Π»ΡΡŽΡ‚ Ρ‚Π°Π±Π»ΠΈΡ†Ρƒ, Π² ΠΊΠΎΡ‚ΠΎΡ€ΡƒΡŽ входят Π²Ρ‹Π±Ρ€Π°Π½Π½Ρ‹Π΅ значСния Ρ„ΡƒΠ½ΠΊΡ†ΠΈΠΈ.

    Π’Π°Π±Π»ΠΈΡ†Π° выглядит ΡΠ»Π΅Π΄ΡƒΡŽΡ‰ΠΈΠΌ ΠΎΠ±Ρ€Π°Π·ΠΎΠΌ:


    Боставив Ρ‚Π°ΠΊΡƒΡŽ Ρ‚Π°Π±Π»ΠΈΡ†Ρƒ, ΠΌΡ‹ ΠΌΠΎΠΆΠ΅ΠΌ Π½Π°ΠΌΠ΅Ρ‚ΠΈΡ‚ΡŒ нСсколько Ρ‚ΠΎΡ‡Π΅ΠΊ Π³Ρ€Π°Ρ„ΠΈΠΊΠ° Ρ„ΡƒΠ½ΠΊΡ†ΠΈΠΈ y = f(x) . Π—Π°Ρ‚Π΅ΠΌ, соСдиняя эти Ρ‚ΠΎΡ‡ΠΊΠΈ ΠΏΠ»Π°Π²Π½ΠΎΠΉ Π»ΠΈΠ½ΠΈΠ΅ΠΉ, ΠΌΡ‹ ΠΈ ΠΏΠΎΠ»ΡƒΡ‡Π°Π΅ΠΌ ΠΏΡ€ΠΈΠ±Π»ΠΈΠ·ΠΈΡ‚Π΅Π»ΡŒΠ½Ρ‹ΠΉ Π²ΠΈΠ΄ Π³Ρ€Π°Ρ„ΠΈΠΊΠ° Ρ„ΡƒΠ½ΠΊΡ†ΠΈΠΈ y = f(x).

    Π‘Π»Π΅Π΄ΡƒΠ΅Ρ‚, ΠΎΠ΄Π½Π°ΠΊΠΎ, Π·Π°ΠΌΠ΅Ρ‚ΠΈΡ‚ΡŒ, Ρ‡Ρ‚ΠΎ ΠΌΠ΅Ρ‚ΠΎΠ΄ построСния Π³Ρ€Π°Ρ„ΠΈΠΊΠ° ΠΏΠΎ нСскольким Ρ‚ΠΎΡ‡ΠΊΠ°ΠΌ ΠΎΡ‡Π΅Π½ΡŒ Π½Π΅Π½Π°Π΄Π΅ΠΆΠ΅Π½. Π’ самом Π΄Π΅Π»Π΅ ΠΏΠΎΠ²Π΅Π΄Π΅Π½ΠΈΠ΅ Π³Ρ€Π°Ρ„ΠΈΠΊΠ° ΠΌΠ΅ΠΆΠ΄Ρƒ Π½Π°ΠΌΠ΅Ρ‡Π΅Π½Π½Ρ‹ΠΌΠΈ Ρ‚ΠΎΡ‡ΠΊΠ°ΠΌΠΈ ΠΈ ΠΏΠΎΠ²Π΅Π΄Π΅Π½ΠΈΠ΅ Π΅Π³ΠΎ Π²Π½Π΅ ΠΎΡ‚Ρ€Π΅Π·ΠΊΠ° ΠΌΠ΅ΠΆΠ΄Ρƒ ΠΊΡ€Π°ΠΉΠ½ΠΈΠΌΠΈ ΠΈΠ· взятых Ρ‚ΠΎΡ‡Π΅ΠΊ остаСтся нСизвСстным.

    ΠŸΡ€ΠΈΠΌΠ΅Ρ€ 1 . Для построСния Π³Ρ€Π°Ρ„ΠΈΠΊΠ° Ρ„ΡƒΠ½ΠΊΡ†ΠΈΠΈ y = f(x) Π½Π΅ΠΊΡ‚ΠΎ составил Ρ‚Π°Π±Π»ΠΈΡ†Ρƒ Π·Π½Π°Ρ‡Π΅Π½ΠΈΠΉ Π°Ρ€Π³ΡƒΠΌΠ΅Π½Ρ‚Π° ΠΈ Ρ„ΡƒΠ½ΠΊΡ†ΠΈΠΈ:


    Π‘ΠΎΠΎΡ‚Π²Π΅Ρ‚ΡΡ‚Π²ΡƒΡŽΡ‰ΠΈΠ΅ ΠΏΡΡ‚ΡŒ Ρ‚ΠΎΡ‡Π΅ΠΊ ΠΏΠΎΠΊΠ°Π·Π°Π½Ρ‹ Π½Π° рис. 48.

    На основании располоТСния этих Ρ‚ΠΎΡ‡Π΅ΠΊ ΠΎΠ½ сдСлал Π²Ρ‹Π²ΠΎΠ΄, Ρ‡Ρ‚ΠΎ Π³Ρ€Π°Ρ„ΠΈΠΊ Ρ„ΡƒΠ½ΠΊΡ†ΠΈΠΈ прСдставляСт собой ΠΏΡ€ΡΠΌΡƒΡŽ (ΠΏΠΎΠΊΠ°Π·Π°Π½Π½ΡƒΡŽ Π½Π° рис. 48 ΠΏΡƒΠ½ΠΊΡ‚ΠΈΡ€ΠΎΠΌ). МоТно Π»ΠΈ ΡΡ‡ΠΈΡ‚Π°Ρ‚ΡŒ этот Π²Ρ‹Π²ΠΎΠ΄ Π½Π°Π΄Π΅ΠΆΠ½Ρ‹ΠΌ? Если Π½Π΅Ρ‚ Π΄ΠΎΠΏΠΎΠ»Π½ΠΈΡ‚Π΅Π»ΡŒΠ½Ρ‹Ρ… сообраТСний, ΠΏΠΎΠ΄Ρ‚Π²Π΅Ρ€ΠΆΠ΄Π°ΡŽΡ‰ΠΈΡ… этот Π²Ρ‹Π²ΠΎΠ΄, Π΅Π³ΠΎ вряд Π»ΠΈ ΠΌΠΎΠΆΠ½ΠΎ ΡΡ‡ΠΈΡ‚Π°Ρ‚ΡŒ Π½Π°Π΄Π΅ΠΆΠ½Ρ‹ΠΌ. Π½Π°Π΄Π΅ΠΆΠ½Ρ‹ΠΌ.

    Для обоснования своСго утвСрТдСния рассмотрим Ρ„ΡƒΠ½ΠΊΡ†ΠΈΡŽ

    .

    ВычислСния ΠΏΠΎΠΊΠ°Π·Ρ‹Π²Π°ΡŽΡ‚, Ρ‡Ρ‚ΠΎ значСния этой Ρ„ΡƒΠ½ΠΊΡ†ΠΈΠΈ Π² Ρ‚ΠΎΡ‡ΠΊΠ°Ρ… -2, -1, 0, 1, 2 ΠΊΠ°ΠΊ Ρ€Π°Π· ΠΎΠΏΠΈΡΡ‹Π²Π°ΡŽΡ‚ΡΡ ΠΏΡ€ΠΈΠ²Π΅Π΄Π΅Π½Π½ΠΎΠΉ Π²Ρ‹ΡˆΠ΅ Ρ‚Π°Π±Π»ΠΈΡ†Π΅ΠΉ. Однако Π³Ρ€Π°Ρ„ΠΈΠΊ этой Ρ„ΡƒΠ½ΠΊΡ†ΠΈΠΈ вовсС Π½Π΅ являСтся прямой Π»ΠΈΠ½ΠΈΠ΅ΠΉ (ΠΎΠ½ ΠΏΠΎΠΊΠ°Π·Π°Π½ Π½Π° рис. 49). Π”Ρ€ΡƒΠ³ΠΈΠΌ ΠΏΡ€ΠΈΠΌΠ΅Ρ€ΠΎΠΌ ΠΌΠΎΠΆΠ΅Ρ‚ ΡΠ»ΡƒΠΆΠΈΡ‚ΡŒ функция y = x + l + sinΟ€x; Π΅Π΅ значСния Ρ‚ΠΎΠΆΠ΅ ΠΎΠΏΠΈΡΡ‹Π²Π°ΡŽΡ‚ΡΡ ΠΏΡ€ΠΈΠ²Π΅Π΄Π΅Π½Π½ΠΎΠΉ Π²Ρ‹ΡˆΠ΅ Ρ‚Π°Π±Π»ΠΈΡ†Π΅ΠΉ.

    Π­Ρ‚ΠΈ ΠΏΡ€ΠΈΠΌΠ΅Ρ€Ρ‹ ΠΏΠΎΠΊΠ°Π·Ρ‹Π²Π°ΡŽΡ‚, Ρ‡Ρ‚ΠΎ Π² «чистом» Π²ΠΈΠ΄Π΅ ΠΌΠ΅Ρ‚ΠΎΠ΄ построСния Π³Ρ€Π°Ρ„ΠΈΠΊΠ° ΠΏΠΎ нСскольким Ρ‚ΠΎΡ‡ΠΊΠ°ΠΌ Π½Π΅Π½Π°Π΄Π΅ΠΆΠ΅Π½. ΠŸΠΎΡΡ‚ΠΎΠΌΡƒ для построСния Π³Ρ€Π°Ρ„ΠΈΠΊΠ° Π·Π°Π΄Π°Π½Π½ΠΎΠΉ Ρ„ΡƒΠ½ΠΊΡ†ΠΈΠΈ,ΠΊΠ°ΠΊ ΠΏΡ€Π°Π²ΠΈΠ»ΠΎ, ΠΏΠΎΡΡ‚ΡƒΠΏΠ°ΡŽΡ‚ ΡΠ»Π΅Π΄ΡƒΡŽΡ‰ΠΈΠΌ ΠΎΠ±Ρ€Π°Π·ΠΎΠΌ. Π‘Π½Π°Ρ‡Π°Π»Π° ΠΈΠ·ΡƒΡ‡Π°ΡŽΡ‚ свойства Π΄Π°Π½Π½ΠΎΠΉ Ρ„ΡƒΠ½ΠΊΡ†ΠΈΠΈ, с ΠΏΠΎΠΌΠΎΡ‰ΡŒΡŽ ΠΊΠΎΡ‚ΠΎΡ€Ρ‹Ρ… ΠΌΠΎΠΆΠ½ΠΎ ΠΏΠΎΡΡ‚Ρ€ΠΎΠΈΡ‚ΡŒ эскиз Π³Ρ€Π°Ρ„ΠΈΠΊΠ°. Π—Π°Ρ‚Π΅ΠΌ, вычисляя значСния Ρ„ΡƒΠ½ΠΊΡ†ΠΈΠΈ Π² Π½Π΅ΡΠΊΠΎΠ»ΡŒΠΊΠΈΡ… Ρ‚ΠΎΡ‡ΠΊΠ°Ρ… (Π²Ρ‹Π±ΠΎΡ€ ΠΊΠΎΡ‚ΠΎΡ€Ρ‹Ρ… зависит ΠΎΡ‚ установлСнных свойств Ρ„ΡƒΠ½ΠΊΡ†ΠΈΠΈ), находят ΡΠΎΠΎΡ‚Π²Π΅Ρ‚ΡΡ‚Π²ΡƒΡŽΡ‰ΠΈΠ΅ Ρ‚ΠΎΡ‡ΠΊΠΈ Π³Ρ€Π°Ρ„ΠΈΠΊΠ°. И, Π½Π°ΠΊΠΎΠ½Π΅Ρ†, Ρ‡Π΅Ρ€Π΅Π· построСнныС Ρ‚ΠΎΡ‡ΠΊΠΈ проводят ΠΊΡ€ΠΈΠ²ΡƒΡŽ, ΠΈΡΠΏΠΎΠ»ΡŒΠ·ΡƒΡ свойства Π΄Π°Π½Π½ΠΎΠΉ Ρ„ΡƒΠ½ΠΊΡ†ΠΈΠΈ.

    НСкоторыС (Π½Π°ΠΈΠ±ΠΎΠ»Π΅Π΅ простыС ΠΈ часто ΠΈΡΠΏΠΎΠ»ΡŒΠ·ΡƒΠ΅ΠΌΡ‹Π΅) свойства Ρ„ΡƒΠ½ΠΊΡ†ΠΈΠΉ, примСняСмыС для нахоТдСния эскиза Π³Ρ€Π°Ρ„ΠΈΠΊΠ°, ΠΌΡ‹ рассмотрим ΠΏΠΎΠ·ΠΆΠ΅, Π° сСйчас Ρ€Π°Π·Π±Π΅Ρ€Π΅ΠΌ Π½Π΅ΠΊΠΎΡ‚ΠΎΡ€Ρ‹Π΅ часто примСняСмыС способы построСния Π³Ρ€Π°Ρ„ΠΈΠΊΠΎΠ².

    Π“Ρ€Π°Ρ„ΠΈΠΊ Ρ„ΡƒΠ½ΠΊΡ†ΠΈΠΈ Ρƒ = |f(x)|.

    НСрСдко приходится ΡΡ‚Ρ€ΠΎΠΈΡ‚ΡŒ Π³Ρ€Π°Ρ„ΠΈΠΊ Ρ„ΡƒΠ½ΠΊΡ†ΠΈΠΈ y = |f(x) |, Π³Π΄Π΅ f(Ρ…) — заданная функция. Напомним, ΠΊΠ°ΠΊ это дСлаСтся. По ΠΎΠΏΡ€Π΅Π΄Π΅Π»Π΅Π½ΠΈΡŽ Π°Π±ΡΠΎΠ»ΡŽΡ‚Π½ΠΎΠΉ Π²Π΅Π»ΠΈΡ‡ΠΈΠ½Ρ‹ числа ΠΌΠΎΠΆΠ½ΠΎ Π½Π°ΠΏΠΈΡΠ°Ρ‚ΡŒ

    Π­Ρ‚ΠΎ Π·Π½Π°Ρ‡ΠΈΡ‚, Ρ‡Ρ‚ΠΎ Π³Ρ€Π°Ρ„ΠΈΠΊ Ρ„ΡƒΠ½ΠΊΡ†ΠΈΠΈ y =|f(x)| ΠΌΠΎΠΆΠ½ΠΎ ΠΏΠΎΠ»ΡƒΡ‡ΠΈΡ‚ΡŒ ΠΈΠ· Π³Ρ€Π°Ρ„ΠΈΠΊΠ°, Ρ„ΡƒΠ½ΠΊΡ†ΠΈΠΈ y = f(x) ΡΠ»Π΅Π΄ΡƒΡŽΡ‰ΠΈΠΌ ΠΎΠ±Ρ€Π°Π·ΠΎΠΌ: всС Ρ‚ΠΎΡ‡ΠΊΠΈ Π³Ρ€Π°Ρ„ΠΈΠΊΠ° Ρ„ΡƒΠ½ΠΊΡ†ΠΈΠΈ Ρƒ = f(Ρ…) , Ρƒ ΠΊΠΎΡ‚ΠΎΡ€Ρ‹Ρ… ΠΎΡ€Π΄ΠΈΠ½Π°Ρ‚Ρ‹ Π½Π΅ΠΎΡ‚Ρ€ΠΈΡ†Π°Ρ‚Π΅Π»ΡŒΠ½Ρ‹, слСдуСт ΠΎΡΡ‚Π°Π²ΠΈΡ‚ΡŒ Π±Π΅Π· измСнСния; Π΄Π°Π»Π΅Π΅, вмСсто Ρ‚ΠΎΡ‡Π΅ΠΊ Π³Ρ€Π°Ρ„ΠΈΠΊΠ° Ρ„ΡƒΠ½ΠΊΡ†ΠΈΠΈ y = f(x) , ΠΈΠΌΠ΅ΡŽΡ‰ΠΈΡ… ΠΎΡ‚Ρ€ΠΈΡ†Π°Ρ‚Π΅Π»ΡŒΠ½Ρ‹Π΅ ΠΊΠΎΠΎΡ€Π΄ΠΈΠ½Π°Ρ‚Ρ‹, слСдуСт ΠΏΠΎΡΡ‚Ρ€ΠΎΠΈΡ‚ΡŒ ΡΠΎΠΎΡ‚Π²Π΅Ρ‚ΡΡ‚Π²ΡƒΡŽΡ‰ΠΈΠ΅ Ρ‚ΠΎΡ‡ΠΊΠΈ Π³Ρ€Π°Ρ„ΠΈΠΊΠ° Ρ„ΡƒΠ½ΠΊΡ†ΠΈΠΈ Ρƒ = -f(x) (Ρ‚. Π΅. Ρ‡Π°ΡΡ‚ΡŒ Π³Ρ€Π°Ρ„ΠΈΠΊΠ° Ρ„ΡƒΠ½ΠΊΡ†ΠΈΠΈ
    y = f(x) , которая Π»Π΅ΠΆΠΈΡ‚ Π½ΠΈΠΆΠ΅ оси Ρ…, слСдуСт симмСтрично ΠΎΡ‚Ρ€Π°Π·ΠΈΡ‚ΡŒ ΠΎΡ‚Π½ΠΎΡΠΈΡ‚Π΅Π»ΡŒΠ½ΠΎ оси Ρ… ).

    ΠŸΡ€ΠΈΠΌΠ΅Ρ€ 2. ΠŸΠΎΡΡ‚Ρ€ΠΎΠΈΡ‚ΡŒ Π³Ρ€Π°Ρ„ΠΈΠΊ Ρ„ΡƒΠ½ΠΊΡ†ΠΈΠΈ Ρƒ = |Ρ…|.

    Π‘Π΅Ρ€Π΅ΠΌ Π³Ρ€Π°Ρ„ΠΈΠΊ Ρ„ΡƒΠ½ΠΊΡ†ΠΈΠΈ Ρƒ = Ρ… (рис. 50, Π°) ΠΈ Ρ‡Π°ΡΡ‚ΡŒ этого Π³Ρ€Π°Ρ„ΠΈΠΊΠ° ΠΏΡ€ΠΈ Ρ… (Π»Π΅ΠΆΠ°Ρ‰ΡƒΡŽ ΠΏΠΎΠ΄ осью Ρ… ) симмСтрично ΠΎΡ‚Ρ€Π°ΠΆΠ°Π΅ΠΌ ΠΎΡ‚Π½ΠΎΡΠΈΡ‚Π΅Π»ΡŒΠ½ΠΎ оси Ρ… . Π’ Ρ€Π΅Π·ΡƒΠ»ΡŒΡ‚Π°Ρ‚Π΅ ΠΌΡ‹ ΠΈ ΠΏΠΎΠ»ΡƒΡ‡Π°Π΅ΠΌ Π³Ρ€Π°Ρ„ΠΈΠΊ Ρ„ΡƒΠ½ΠΊΡ†ΠΈΠΈ Ρƒ = |Ρ…| (рис. 50, Π±).

    ΠŸΡ€ΠΈΠΌΠ΅Ρ€ 3 . ΠŸΠΎΡΡ‚Ρ€ΠΎΠΈΡ‚ΡŒ Π³Ρ€Π°Ρ„ΠΈΠΊ Ρ„ΡƒΠ½ΠΊΡ†ΠΈΠΈ y = |x 2 — 2x|.

    Π‘Π½Π°Ρ‡Π°Π»Π° построим Π³Ρ€Π°Ρ„ΠΈΠΊ Ρ„ΡƒΠ½ΠΊΡ†ΠΈΠΈ y = x 2 — 2x. Π“Ρ€Π°Ρ„ΠΈΠΊ этой Ρ„ΡƒΠ½ΠΊΡ†ΠΈΠΈ — ΠΏΠ°Ρ€Π°Π±ΠΎΠ»Π°, Π²Π΅Ρ‚Π²ΠΈ ΠΊΠΎΡ‚ΠΎΡ€ΠΎΠΉ Π½Π°ΠΏΡ€Π°Π²Π»Π΅Π½Ρ‹ Π²Π²Π΅Ρ€Ρ…, Π²Π΅Ρ€ΡˆΠΈΠ½Π° ΠΏΠ°Ρ€Π°Π±ΠΎΠ»Ρ‹ ΠΈΠΌΠ΅Π΅Ρ‚ ΠΊΠΎΠΎΡ€Π΄ΠΈΠ½Π°Ρ‚Ρ‹ (1; -1), Π΅Π΅ Π³Ρ€Π°Ρ„ΠΈΠΊ пСрСсСкаСт ось абсцисс Π² Ρ‚ΠΎΡ‡ΠΊΠ°Ρ… 0 ΠΈ 2. На ΠΏΡ€ΠΎΠΌΠ΅ΠΆΡƒΡ‚ΠΊΠ΅ (0; 2) фукция ΠΏΡ€ΠΈΠ½ΠΈΠΌΠ°Π΅Ρ‚ ΠΎΡ‚Ρ€ΠΈΡ†Π°Ρ‚Π΅Π»ΡŒΠ½Ρ‹Π΅ значСния, поэтому ΠΈΠΌΠ΅Π½Π½ΠΎ эту Ρ‡Π°ΡΡ‚ΡŒ Π³Ρ€Π°Ρ„ΠΈΠΊΠ° симмСтрично ΠΎΡ‚Ρ€Π°Π·ΠΈΠΌ ΠΎΡ‚Π½ΠΎΡΠΈΡ‚Π΅Π»ΡŒΠ½ΠΎ оси абсцисс. На рисункС 51 построСн Π³Ρ€Π°Ρ„ΠΈΠΊ Ρ„ΡƒΠ½ΠΊΡ†ΠΈΠΈ Ρƒ = |Ρ… 2 -2Ρ…| , исходя ΠΈΠ· Π³Ρ€Π°Ρ„ΠΈΠΊΠ° Ρ„ΡƒΠ½ΠΊΡ†ΠΈΠΈ Ρƒ = Ρ… 2 — 2x

    Π“Ρ€Π°Ρ„ΠΈΠΊ Ρ„ΡƒΠ½ΠΊΡ†ΠΈΠΈ y = f(x) + g(x)

    Рассмотрим Π·Π°Π΄Π°Ρ‡Ρƒ построСния Π³Ρ€Π°Ρ„ΠΈΠΊΠ° Ρ„ΡƒΠ½ΠΊΡ†ΠΈΠΈ y = f(x) + g(x). Ссли Π·Π°Π΄Π°Π½Ρ‹ Π³Ρ€Π°Ρ„ΠΈΠΊΠΈ Ρ„ΡƒΠ½ΠΊΡ†ΠΈΠΉ y = f(x) ΠΈ y = g(x) .

    Π—Π°ΠΌΠ΅Ρ‚ΠΈΠΌ, Ρ‡Ρ‚ΠΎ ΠΎΠ±Π»Π°ΡΡ‚ΡŒΡŽ опрСдСлСния Ρ„ΡƒΠ½ΠΊΡ†ΠΈΠΈ y = |f(x) + g(Ρ…)| являСтся мноТСство всСх Ρ‚Π΅Ρ… Π·Π½Π°Ρ‡Π΅Π½ΠΈΠΉ Ρ…, для ΠΊΠΎΡ‚ΠΎΡ€Ρ‹Ρ… ΠΎΠΏΡ€Π΅Π΄Π΅Π»Π΅Π½Ρ‹ ΠΎΠ±Π΅ Ρ„ΡƒΠ½ΠΊΡ†ΠΈΠΈ y = f{x) ΠΈ Ρƒ = g(Ρ…), Ρ‚. Π΅. эта ΠΎΠ±Π»Π°ΡΡ‚ΡŒ опрСдСлСния прСдставляСт собой пСрСсСчСниС областСй опрСдСлСния, Ρ„ΡƒΠ½ΠΊΡ†ΠΈΠΉ f{x) ΠΈ g{x).

    ΠŸΡƒΡΡ‚ΡŒ Ρ‚ΠΎΡ‡ΠΊΠΈ (Ρ… 0 , y 1 ) ΠΈ (Ρ… 0 , Ρƒ 2 ) соотвСтствСнно ΠΏΡ€ΠΈΠ½Π°Π΄Π»Π΅ΠΆΠ°Ρ‚ Π³Ρ€Π°Ρ„ΠΈΠΊΠ°ΠΌ Ρ„ΡƒΠ½ΠΊΡ†ΠΈΠΉ y = f{x) ΠΈ y = g(Ρ…) , Ρ‚. Π΅. y 1 = f(x 0), y 2 = g(Ρ… 0). Π’ΠΎΠ³Π΄Π° Ρ‚ΠΎΡ‡ΠΊΠ° (x0;. y1 + y2) ΠΏΡ€ΠΈΠ½Π°Π΄Π»Π΅ΠΆΠΈΡ‚ Π³Ρ€Π°Ρ„ΠΈΠΊΡƒ Ρ„ΡƒΠ½ΠΊΡ†ΠΈΠΈ Ρƒ = f(Ρ…) + g(Ρ…) (ΠΈΠ±ΠΎ f(Ρ… 0) + g(x 0 ) = y1 +y2 ),. ΠΏΡ€ΠΈΡ‡Π΅ΠΌ любая Ρ‚ΠΎΡ‡ΠΊΠ° Π³Ρ€Π°Ρ„ΠΈΠΊΠ° Ρ„ΡƒΠ½ΠΊΡ†ΠΈΠΈ y = f(x) + g(x) ΠΌΠΎΠΆΠ΅Ρ‚ Π±Ρ‹Ρ‚ΡŒ ΠΏΠΎΠ»ΡƒΡ‡Π΅Π½Π° Ρ‚Π°ΠΊΠΈΠΌ ΠΎΠ±Ρ€Π°Π·ΠΎΠΌ. Π‘Π»Π΅Π΄ΠΎΠ²Π°Ρ‚Π΅Π»ΡŒΠ½ΠΎ, Π³Ρ€Π°Ρ„ΠΈΠΊ Ρ„ΡƒΠ½ΠΊΡ†ΠΈΠΈ Ρƒ = f(Ρ…) + g(x) ΠΌΠΎΠΆΠ½ΠΎ ΠΏΠΎΠ»ΡƒΡ‡ΠΈΡ‚ΡŒ ΠΈΠ· Π³Ρ€Π°Ρ„ΠΈΠΊΠΎΠ² Ρ„ΡƒΠ½ΠΊΡ†ΠΈΠΉ y = f(x) . ΠΈ y = g(Ρ…) Π·Π°ΠΌΠ΅Π½ΠΎΠΉ ΠΊΠ°ΠΆΠ΄ΠΎΠΉ Ρ‚ΠΎΡ‡ΠΊΠΈ (Ρ… n , Ρƒ 1) Π³Ρ€Π°Ρ„ΠΈΠΊΠ° Ρ„ΡƒΠ½ΠΊΡ†ΠΈΠΈ y = f(x) Ρ‚ΠΎΡ‡ΠΊΠΎΠΉ (Ρ… n , y 1 + y 2), Π³Π΄Π΅ Ρƒ 2 = g(x n ), Ρ‚. Π΅. сдвигом ΠΊΠ°ΠΆΠ΄ΠΎΠΉ Ρ‚ΠΎΡ‡ΠΊΠΈ (Ρ… n , Ρƒ 1 ) Π³Ρ€Π°Ρ„ΠΈΠΊΠ° Ρ„ΡƒΠ½ΠΊΡ†ΠΈΠΈ y = f(x) вдоль оси Ρƒ Π½Π° Π²Π΅Π»ΠΈΡ‡ΠΈΠ½Ρƒ y 1 = g(Ρ… n ). ΠŸΡ€ΠΈ этом Ρ€Π°ΡΡΠΌΠ°Ρ‚Ρ€ΠΈΠ²Π°ΡŽΡ‚ΡΡ Ρ‚ΠΎΠ»ΡŒΠΊΠΎ Ρ‚Π°ΠΊΠΈΠ΅ Ρ‚ΠΎΡ‡ΠΊΠΈ Ρ… n для ΠΊΠΎΡ‚ΠΎΡ€Ρ‹Ρ… ΠΎΠΏΡ€Π΅Π΄Π΅Π»Π΅Π½Ρ‹ ΠΎΠ±Π΅ Ρ„ΡƒΠ½ΠΊΡ†ΠΈΠΈ y = f(x) ΠΈ y = g(x) .

    Π’Π°ΠΊΠΎΠΉ ΠΌΠ΅Ρ‚ΠΎΠ΄ построСния Π³Ρ€Π°Ρ„ΠΈΠΊΠ° Ρ„ΡƒΠ½ΠΊΡ†ΠΈΠΈ y = f(x) + g(Ρ… ) называСтся слоТСниСм Π³Ρ€Π°Ρ„ΠΈΠΊΠΎΠ² Ρ„ΡƒΠ½ΠΊΡ†ΠΈΠΉ y = f(x) ΠΈ y = g(x)

    ΠŸΡ€ΠΈΠΌΠ΅Ρ€ 4 . На рисункС ΠΌΠ΅Ρ‚ΠΎΠ΄ΠΎΠΌ слоТСния Π³Ρ€Π°Ρ„ΠΈΠΊΠΎΠ² построСн Π³Ρ€Π°Ρ„ΠΈΠΊ Ρ„ΡƒΠ½ΠΊΡ†ΠΈΠΈ
    y = x + sinx .

    ΠŸΡ€ΠΈ построСнии Π³Ρ€Π°Ρ„ΠΈΠΊΠ° Ρ„ΡƒΠ½ΠΊΡ†ΠΈΠΈ y = x + sinx ΠΌΡ‹ ΠΏΠΎΠ»Π°Π³Π°Π»ΠΈ, Ρ‡Ρ‚ΠΎ f(x) = x, Π° g(x) = sinx. Для построСния Π³Ρ€Π°Ρ„ΠΈΠΊΠ° Ρ„ΡƒΠ½ΠΊΡ†ΠΈΠΈ Π²Ρ‹Π±Π΅Ρ€Π΅ΠΌ Ρ‚ΠΎΡ‡ΠΊΠΈ с aбциссами -1,5Ο€, -, -0,5, 0, 0,5,, 1,5, 2. ЗначСния f(x) = x, g(x) = sinx, y = x + sinx вычислим Π² Π²Ρ‹Π±Ρ€Π°Π½Π½Ρ‹Ρ… Ρ‚ΠΎΡ‡ΠΊΠ°Ρ… ΠΈ Ρ€Π΅Π·ΡƒΠ»ΡŒΡ‚Π°Ρ‚Ρ‹ помСстим Π² Ρ‚Π°Π±Π»ΠΈΡ†Π΅.


    Π’ΠΈΠ·ΡƒΠ°Π»ΡŒΠ½ΠΎΠ΅ исчислСниС β€” полярныС ΠΊΠΎΠΎΡ€Π΄ΠΈΠ½Π°Ρ‚Ρ‹

    Π’ΠΈΠ·ΡƒΠ°Π»ΡŒΠ½ΠΎΠ΅ исчислСниС β€” полярныС ΠΊΠΎΠΎΡ€Π΄ΠΈΠ½Π°Ρ‚Ρ‹

    Π¦Π΅Π»ΠΈ: Π’ этом ΡƒΡ€ΠΎΠΊΠ΅ ΠΌΡ‹ опрСдСляСм полярныС ΠΊΠΎΠΎΡ€Π΄ΠΈΠ½Π°Ρ‚Ρ‹. ΠœΡ‹ ΠΊΠΎΠ½Π²Π΅Ρ€Ρ‚ΠΈΡ€ΡƒΠ΅ΠΌ ΠΎΡ‚ полярных ΠΊΠΎΠΎΡ€Π΄ΠΈΠ½Π°Ρ‚ ΠΊ ΠΏΡ€ΡΠΌΠΎΡƒΠ³ΠΎΠ»ΡŒΠ½Ρ‹ΠΌ ΠΊΠΎΠΎΡ€Π΄ΠΈΠ½Π°Ρ‚Π°ΠΌ ΠΈ ΠΎΡ‚ ΠΏΡ€ΡΠΌΠΎΡƒΠ³ΠΎΠ»ΡŒΠ½Ρ‹Ρ… ΠΊΠΎΠΎΡ€Π΄ΠΈΠ½Π°Ρ‚Ρ‹ Π² полярныС ΠΊΠΎΠΎΡ€Π΄ΠΈΠ½Π°Ρ‚Ρ‹. ΠœΡ‹ нарисуСм Π½Π΅ΠΊΠΎΡ‚ΠΎΡ€Ρ‹Π΅ ΠΈΠ· основных Ρ„ΡƒΠ½ΠΊΡ†ΠΈΠΉ Π² полярныС ΠΊΠΎΠΎΡ€Π΄ΠΈΠ½Π°Ρ‚Ρ‹ с ΠΏΠΎΠΌΠΎΡ‰ΡŒΡŽ LiveMath ΠΈ графичСского ΠΊΠ°Π»ΡŒΠΊΡƒΠ»ΡΡ‚ΠΎΡ€Π°. ПослС Ρ€Π°Π±ΠΎΡ‚Ρ‹ с этими ΠΌΠ°Ρ‚Π΅Ρ€ΠΈΠ°Π»Π°ΠΌΠΈ учащийся Π΄ΠΎΠ»ΠΆΠ΅Π½ ΡƒΠΌΠ΅Ρ‚ΡŒ

    • для прСобразования ΠΏΡ€ΡΠΌΠΎΡƒΠ³ΠΎΠ»ΡŒΠ½Ρ‹Ρ… ΠΊΠΎΠΎΡ€Π΄ΠΈΠ½Π°Ρ‚ Π² полярныС;
    • для прСобразования полярных ΠΊΠΎΠΎΡ€Π΄ΠΈΠ½Π°Ρ‚ Π² ΠΏΡ€ΡΠΌΠΎΡƒΠ³ΠΎΠ»ΡŒΠ½Ρ‹Π΅;
    • для распознавания Π½Π΅ΠΊΠΎΡ‚ΠΎΡ€Ρ‹Ρ… стандартных полярных Π³Ρ€Π°Ρ„ΠΈΠΊΠΎΠ²;
    • для построСния Ρ‚ΠΎΡ‡Π΅ΠΊ Π² полярных ΠΊΠΎΠΎΡ€Π΄ΠΈΠ½Π°Ρ‚Π°Ρ…;
    • для построСния полярных ΡƒΡ€Π°Π²Π½Π΅Π½ΠΈΠΉ с ΠΏΠΎΠΌΠΎΡ‰ΡŒΡŽ графичСского ΠΊΠ°Π»ΡŒΠΊΡƒΠ»ΡΡ‚ΠΎΡ€Π° ΠΈΠ»ΠΈ ΠΏΡ€ΠΎΠ³Ρ€Π°ΠΌΠΌΠ½ΠΎΠ³ΠΎ обСспСчСния.

    ΠœΠΎΠ΄ΡƒΠ»ΠΈ:

      ΠžΠΏΡ€Π΅Π΄Π΅Π»Π΅Π½ΠΈΠ΅. Π’ΠΎΡ‡ΠΊΠ° P Π² плоскости ΠΈΠΌΠ΅Π΅Ρ‚ полярныС ΠΊΠΎΠΎΡ€Π΄ΠΈΠ½Π°Ρ‚Ρ‹ (r,Β q) , Ссли линия сСгмСнт OP ΠΈΠΌΠ΅Π΅Ρ‚ Π΄Π»ΠΈΠ½Ρƒ r ΠΈ ΡƒΠ³ΠΎΠ», ΠΊΠΎΡ‚ΠΎΡ€Ρ‹ΠΉ OP ΠΎΠ±Ρ€Π°Π·ΡƒΠ΅Ρ‚ с ΠΏΠΎΠ»ΠΎΠΆΠΈΡ‚Π΅Π»ΡŒΠ½Π°Ρ ось q (ΠΈΠ·ΠΌΠ΅Ρ€Π΅Π½ΠΎ Π² счСтчикС ΠΏΠΎ часовой стрСлкС).

      Π­Ρ‚ΠΎ ΠΎΠΏΡ€Π΅Π΄Π΅Π»Π΅Π½ΠΈΠ΅ Ρ‚Ρ€Π΅Π±ΡƒΠ΅Ρ‚, Ρ‡Ρ‚ΠΎΠ±Ρ‹ r > Β 0 . Если rΒ , Ρ‚ΠΎ ΠΌΡ‹ рассматриваСм Ρ‚ΠΎΡ‡ΠΊΡƒ Q , ΠΈΠΌΠ΅ΡŽΡ‰ΡƒΡŽ полярныС ΠΊΠΎΠΎΡ€Π΄ΠΈΠ½Π°Ρ‚Ρ‹ (-r, q) .Π’ΠΎΠ³Π΄Π° Ρ‚ΠΎΡ‡ΠΊΠ° P ΠΈΠΌΠ΅Π΅Ρ‚ полярныС ΠΊΠΎΠΎΡ€Π΄ΠΈΠ½Π°Ρ‚Ρ‹ (r,Β q) , Ссли P Ρ‚ΠΎΡ‡ΠΊΠ° Π½Π° прямой, содСрТащСй O ΠΈ Q , которая -r ΡˆΡ‚ΡƒΠΊ ΠΈΠ· O с ΠΏΡ€ΠΎΡ‚ΠΈΠ²ΠΎΠΏΠΎΠ»ΠΎΠΆΠ½ΠΎΠΉ стороны O ΠΈΠ· Q .

    • ΠŸΠΎΡΡ‚Ρ€ΠΎΠ΅Π½ΠΈΠ΅ Ρ‚ΠΎΡ‡Π΅ΠΊ [с ΠΏΠΎΠΌΠΎΡ‰ΡŒΡŽ LiveMath].
      Π’Π΅ΠΎΡ€Π΅ΠΌΠ°. Если Ρ‚ΠΎΡ‡ΠΊΠ° P ΠΈΠΌΠ΅Π΅Ρ‚ полярныС ΠΊΠΎΠΎΡ€Π΄ΠΈΠ½Π°Ρ‚Ρ‹ (r,Β q) , Ρ‚ΠΎ ΠΏΡ€ΡΠΌΠΎΡƒΠ³ΠΎΠ»ΡŒΠ½Ρ‹Π΅ ΠΊΠΎΠΎΡ€Π΄ΠΈΠ½Π°Ρ‚Ρ‹ P Ρ€Π°Π²Π½Ρ‹ (rΒ cos(q), rΒ sin(q)) .Π”Ρ€ΡƒΠ³ΠΈΠΌΠΈ словами,
      Ρ… = r cos(q)
      y = r sin(q)
    • ΠŸΡ€ΠΈΠΌΠ΅Ρ€Ρ‹ [с использованиСм LiveMath].
      Π’Π΅ΠΎΡ€Π΅ΠΌΠ°. Если Ρ‚ΠΎΡ‡ΠΊΠ° P ΠΈΠΌΠ΅Π΅Ρ‚ ΠΏΡ€ΡΠΌΠΎΡƒΠ³ΠΎΠ»ΡŒΠ½Ρ‹Π΅ ΠΊΠΎΠΎΡ€Π΄ΠΈΠ½Π°Ρ‚Ρ‹ (x, y) , Ρ‚ΠΎ полярныС ΠΊΠΎΠΎΡ€Π΄ΠΈΠ½Π°Ρ‚Ρ‹ P Ρ€Π°Π²Π½Ρ‹ (r, q) Π³Π΄Π΅
      r 2 = x 2 + y 2
      q = арктангСнс (y/x).

      ΠœΡ‹ Π²Ρ‹Π±ΠΈΡ€Π°Π΅ΠΌ ΠΏΠΎΠ»ΠΎΠΆΠΈΡ‚Π΅Π»ΡŒΠ½Ρ‹ΠΉ ΠΊΠ²Π°Π΄Ρ€Π°Ρ‚Π½Ρ‹ΠΉ ΠΊΠΎΡ€Π΅Π½ΡŒ ΠΈΠ· x 2 Β +Β y 2 вмСсто r , Ссли xΒ >Β 0 , ΠΈ ΠΎΡ‚Ρ€ΠΈΡ†Π°Ρ‚Π΅Π»ΡŒΠ½Ρ‹ΠΉ ΠΊΠ²Π°Π΄Ρ€Π°Ρ‚Π½Ρ‹ΠΉ ΠΊΠΎΡ€Π΅Π½ΡŒ Π² ΠΏΡ€ΠΎΡ‚ΠΈΠ²Π½ΠΎΠΌ случаС.

    • ΠŸΡ€ΠΈΠΌΠ΅Ρ€Ρ‹ [с использованиСм LiveMath].
    • ΠŸΠΎΡΡ‚Ρ€ΠΎΠ΅Π½ΠΈΠ΅ Π³Ρ€Π°Ρ„ΠΈΠΊΠΎΠ²
    • ИспользованиС

    Как ΡΠΎΠ·Π΄Π°Ρ‚ΡŒ ΠΏΠΎΠ»ΡΡ€Π½ΡƒΡŽ Π΄ΠΈΠ°Π³Ρ€Π°ΠΌΠΌΡƒ Π² Excel

    Π’ этом руководствС ΠΏΠΎΠΊΠ°Π·Π°Π½ΠΎ, ΠΊΠ°ΠΊ ΡΠΎΠ·Π΄Π°Ρ‚ΡŒ ΠΏΠΎΠ»ΡΡ€Π½ΡƒΡŽ Π΄ΠΈΠ°Π³Ρ€Π°ΠΌΠΌΡƒ Π²ΠΎ всСх вСрсиях Excel: 2007, 2010, 2013, 2016 ΠΈ 2019.

    ΠŸΠΎΠ»ΡΡ€Π½Ρ‹ΠΉ участок β€” ΡΠΊΠ°Ρ‡Π°Ρ‚ΡŒ бСсплатный шаблон

    Π—Π°Π³Ρ€ΡƒΠ·ΠΈΡ‚Π΅ наш бСсплатный шаблон полярного Π³Ρ€Π°Ρ„ΠΈΠΊΠ° для Excel.

    Π—Π°Π³Ρ€ΡƒΠ·ΠΈΡ‚ΡŒ сСйчас

    ΠŸΠΎΠ»ΡΡ€Π½Ρ‹ΠΉ Π³Ρ€Π°Ρ„ΠΈΠΊ ΠΈΡΠΏΠΎΠ»ΡŒΠ·ΡƒΠ΅Ρ‚ΡΡ для опрСдСлСния Ρ‚ΠΎΡ‡ΠΊΠΈ Π² пространствС Π² Ρ‚Π°ΠΊ Π½Π°Π·Ρ‹Π²Π°Π΅ΠΌΠΎΠΉ полярной систСмС ΠΊΠΎΠΎΡ€Π΄ΠΈΠ½Π°Ρ‚, Π³Π΄Π΅ вмСсто стандартных ΠΊΠΎΠΎΡ€Π΄ΠΈΠ½Π°Ρ‚ x ΠΈ y каТдая Ρ‚ΠΎΡ‡ΠΊΠ° Π½Π° полярной плоскости выраТаСтся с использованиСм ΡΠ»Π΅Π΄ΡƒΡŽΡ‰ΠΈΡ… Π΄Π²ΡƒΡ… Π·Π½Π°Ρ‡Π΅Π½ΠΈΠΉ:

    • Радиус ( r ) – РасстояниС ΠΎΡ‚ Ρ†Π΅Π½Ρ‚Ρ€Π° участка
    • Π’Π΅Ρ‚Π° (ΞΈ) – Π£Π³ΠΎΠ» ΠΎΡ‚ ΠΎΠΏΠΎΡ€Π½ΠΎΠ³ΠΎ ΡƒΠ³Π»Π°

    Π‘Π°ΠΌΠ° ΠΏΠ»ΠΎΡΠΊΠΎΡΡ‚ΡŒ состоит ΠΈΠ· концСнтричСских окруТностСй, Ρ€Π°ΡΡˆΠΈΡ€ΡΡŽΡ‰ΠΈΡ…ΡΡ Π½Π°Ρ€ΡƒΠΆΡƒ ΠΎΡ‚ Π½Π°Ρ‡Π°Π»Π° ΠΊΠΎΠΎΡ€Π΄ΠΈΠ½Π°Ρ‚ ΠΈΠ»ΠΈ полюса β€” ΠΎΡ‚ΡΡŽΠ΄Π° ΠΈ Π½Π°Π·Π²Π°Π½ΠΈΠ΅.ΠŸΠΎΠ»ΡΡ€Π½Ρ‹ΠΉ Π³Ρ€Π°Ρ„ΠΈΠΊ пригодится, ΠΊΠΎΠ³Π΄Π° Π°Π½Π°Π»ΠΈΠ·ΠΈΡ€ΡƒΠ΅ΠΌΡ‹Π΅ Π΄Π°Π½Π½Ρ‹Π΅ ΠΈΠΌΠ΅ΡŽΡ‚ цикличСский Ρ…Π°Ρ€Π°ΠΊΡ‚Π΅Ρ€.

    Π’ качСствС ΠΏΡ€ΠΈΠΌΠ΅Ρ€Π° Π½Π° ΠΏΡ€ΠΈΠ²Π΅Π΄Π΅Π½Π½ΠΎΠΉ Π½ΠΈΠΆΠ΅ Π΄ΠΈΠ°Π³Ρ€Π°ΠΌΠΌΠ΅ ΡΡ€Π°Π²Π½ΠΈΠ²Π°ΡŽΡ‚ΡΡ ΠΎΡ†Π΅Π½ΠΊΠΈ удовлСтворСнности ΠΊΠ»ΠΈΠ΅Π½Ρ‚ΠΎΠ² (CSAT), ΠΏΠΎΠΊΠ°Π·Π°Ρ‚Π΅Π»ΡŒ, ΠΊΠΎΡ‚ΠΎΡ€Ρ‹ΠΉ ΠΈΠ»Π»ΡŽΡΡ‚Ρ€ΠΈΡ€ΡƒΠ΅Ρ‚ ΡƒΠ΄ΠΎΠ²Π»Π΅Ρ‚Π²ΠΎΡ€Π΅Π½Π½ΠΎΡΡ‚ΡŒ ΠΊΠ»ΠΈΠ΅Π½Ρ‚ΠΎΠ² Π±Ρ€Π΅Π½Π΄ΠΎΠΌ ΠΈΠ»ΠΈ ΠΏΡ€ΠΎΠ΄ΡƒΠΊΡ‚ΠΎΠΌ, Π΄Π²ΡƒΡ… ΠΎΡ€Π³Π°Π½ΠΈΠ·Π°Ρ†ΠΈΠΉ Π½Π° протяТСнии 2019 Π³ΠΎΠ΄Π°: Simpson Ltd ΠΈ Griffin Ltd.

    Π“Ρ€Π°Ρ„ΠΈΠΊ позволяСт быстро ΠΎΡ†Π΅Π½ΠΈΡ‚ΡŒ Ρ…ΠΎΡ€ΠΎΡˆΠΈΠ΅ ΠΈ ΠΏΠ»ΠΎΡ…ΠΈΠ΅ мСсяцы для ΠΊΠ°ΠΆΠ΄ΠΎΠΉ ΠΊΠΎΠΌΠΏΠ°Π½ΠΈΠΈ, Ρ‡Ρ‚ΠΎ ΠΎΠ±Π»Π΅Π³Ρ‡Π°Π΅Ρ‚ принятиС Π±ΠΎΠ»Π΅Π΅ эффСктивных Ρ€Π΅ΡˆΠ΅Π½ΠΈΠΉ.

    Однако Π²ΠΎΡ‚ Π² Ρ‡Π΅ΠΌ Π·Π°Π³Π²ΠΎΠ·Π΄ΠΊΠ°:

    Excel Π½Π΅ ΠΏΠΎΠ΄Π΄Π΅Ρ€ΠΆΠΈΠ²Π°Π΅Ρ‚ этот Ρ‚ΠΈΠΏ Π΄ΠΈΠ°Π³Ρ€Π°ΠΌΠΌΡ‹ β€” фактичСски, ΠΎΠ½ Π΄Π°ΠΆΠ΅ Π½Π΅ ΠΌΠΎΠΆΠ΅Ρ‚ ΡΡ‡ΠΈΡ‚Ρ‹Π²Π°Ρ‚ΡŒ полярныС ΠΊΠΎΠΎΡ€Π΄ΠΈΠ½Π°Ρ‚Ρ‹ β€” это ΠΎΠ·Π½Π°Ρ‡Π°Π΅Ρ‚, Ρ‡Ρ‚ΠΎ Π²Π°ΠΌ придСтся ΡΠΎΠ·Π΄Π°Π²Π°Ρ‚ΡŒ Π΅Π³ΠΎ с нуля.ΠšΡ€ΠΎΠΌΠ΅ Ρ‚ΠΎΠ³ΠΎ, Π½Π΅ Π·Π°Π±ΡƒΠ΄ΡŒΡ‚Π΅ ΠΏΡ€ΠΎΠ²Π΅Ρ€ΠΈΡ‚ΡŒ надстройку Chart Creator, ΠΌΠΎΡ‰Π½Ρ‹ΠΉ инструмСнт для создания ΡƒΠΌΠΎΠΏΠΎΠΌΡ€Π°Ρ‡ΠΈΡ‚Π΅Π»ΡŒΠ½Ρ‹Ρ… Ρ€Π°ΡΡˆΠΈΡ€Π΅Π½Π½Ρ‹Ρ… Π΄ΠΈΠ°Π³Ρ€Π°ΠΌΠΌ ΠΈ Π³Ρ€Π°Ρ„ΠΈΠΊΠΎΠ² Excel всСго Π·Π° нСсколько ΠΊΠ»ΠΈΠΊΠΎΠ².

    Π’ этом ΠΏΠΎΠ΄Ρ€ΠΎΠ±Π½ΠΎΠΌ пошаговом руководствС Π²Ρ‹ ΡƒΠ·Π½Π°Π΅Ρ‚Π΅, ΠΊΠ°ΠΊ ΠΏΡ€Π΅ΠΎΠ±Ρ€Π°Π·ΠΎΠ²Π°Ρ‚ΡŒ Π½Π΅ΠΎΠ±Ρ€Π°Π±ΠΎΡ‚Π°Π½Π½Ρ‹Π΅ Π΄Π°Π½Π½Ρ‹Π΅ Π² полярный Π³Ρ€Π°Ρ„ΠΈΠΊ Π² Excel с нуля. Напомним, Ρ‡Ρ‚ΠΎ эта ΡΡ‚Π°Ρ‚ΡŒΡ основана Π½Π° руководствС, созданном Π”ΠΆΠΎΠ½ΠΎΠΌ ΠŸΠ΅Π»ΡŒΡ‚ΡŒΠ΅.

    Начало Ρ€Π°Π±ΠΎΡ‚Ρ‹

    ΠŸΠΎΡΠΊΠΎΠ»ΡŒΠΊΡƒ Π±Π°Π»Π»Ρ‹ CSAT ΠΎΠ±Ρ‹Ρ‡Π½ΠΎ Π²Ρ‹Ρ€Π°ΠΆΠ°ΡŽΡ‚ΡΡ Π² Π²ΠΈΠ΄Π΅ ΠΏΡ€ΠΎΡ†Π΅Π½Ρ‚Π½ΠΎΠΉ ΡˆΠΊΠ°Π»Ρ‹, рассмотритС ΡΠ»Π΅Π΄ΡƒΡŽΡ‰ΡƒΡŽ Ρ‚Π°Π±Π»ΠΈΡ†Ρƒ:

    Π¨Π°Π³ β„– 1. НастройтС Π²ΡΠΏΠΎΠΌΠΎΠ³Π°Ρ‚Π΅Π»ΡŒΠ½ΡƒΡŽ Ρ‚Π°Π±Π»ΠΈΡ†Ρƒ.

    Π‘Ρ€Π°Π·Ρƒ набросайтС Π²ΡΠΏΠΎΠΌΠΎΠ³Π°Ρ‚Π΅Π»ΡŒΠ½ΡƒΡŽ Ρ‚Π°Π±Π»ΠΈΡ†Ρƒ, Π² ΠΊΠΎΡ‚ΠΎΡ€ΠΎΠΉ Π±ΡƒΠ΄ΡƒΡ‚ Π²Ρ‹ΠΏΠΎΠ»Π½ΡΡ‚ΡŒΡΡ всС расчСты для вашСй Π΄ΠΈΠ°Π³Ρ€Π°ΠΌΠΌΡ‹. Π§Ρ‚ΠΎΠ±Ρ‹ ΠΏΠΎΡΡ‚Ρ€ΠΎΠΈΡ‚ΡŒ Π³Ρ€Π°Ρ„ΠΈΠΊ, Π²Π°ΠΌ Π½ΡƒΠΆΠ½ΠΎ сначала Π²Ρ‹Ρ‡ΠΈΡΠ»ΠΈΡ‚ΡŒ полярныС ΠΊΠΎΠΎΡ€Π΄ΠΈΠ½Π°Ρ‚Ρ‹, Π° Π·Π°Ρ‚Π΅ΠΌ ΠΏΡ€Π΅ΠΎΠ±Ρ€Π°Π·ΠΎΠ²Π°Ρ‚ΡŒ ΠΈΡ… Π² значСния осСй x ΠΈ y, ΠΈΡΠΏΠΎΠ»ΡŒΠ·ΡƒΠ΅ΠΌΡ‹Π΅ Excel для создания Π΄ΠΈΠ°Π³Ρ€Π°ΠΌΠΌΡ‹.

    НастройтС ΠΎΡ‚Π΄Π΅Π»ΡŒΠ½ΡƒΡŽ Ρ„ΠΈΠΊΡ‚ΠΈΠ²Π½ΡƒΡŽ Ρ‚Π°Π±Π»ΠΈΡ†Ρƒ ΡΠ»Π΅Π΄ΡƒΡŽΡ‰ΠΈΠΌ ΠΎΠ±Ρ€Π°Π·ΠΎΠΌ:


    ΠžΠ±Ρ€Π°Ρ‚ΠΈΡ‚Π΅ Π²Π½ΠΈΠΌΠ°Π½ΠΈΠ΅, Ρ‡Ρ‚ΠΎ Π²ΡΠΏΠΎΠΌΠΎΠ³Π°Ρ‚Π΅Π»ΡŒΠ½Π°Ρ Ρ‚Π°Π±Π»ΠΈΡ†Π° начинаСтся с Ρ„ΠΈΠΊΡ‚ΠΈΠ²Π½ΠΎΠΉ строки ( E2:h3 ) β€” это опрСдСляСт ΠΎΠΏΠΎΡ€Π½Ρ‹ΠΉ ΡƒΠ³ΠΎΠ». РасскаТСм ΠΎ ΠΊΠ°ΠΆΠ΄ΠΎΠΌ элСмСнтС Ρ‚Π°Π±Π»ΠΈΡ†Ρ‹ Ρ‡ΡƒΡ‚ΡŒ ΠΏΠΎΠ΄Ρ€ΠΎΠ±Π½Π΅Π΅:

    • ΠœΠ΅ΡΡΡ† β€” этот столбСц содСрТит качСствСнныС ΠΊΠ°Ρ‚Π΅Π³ΠΎΡ€ΠΈΠΈ, ΠΏΠΎΠ»ΡƒΡ‡Π΅Π½Π½Ρ‹Π΅ ΠΈΠ· исходных Π΄Π°Π½Π½Ρ‹Ρ….Π’Π²Π΅Π΄ΠΈΡ‚Π΅ «Начало» Π² ΠΏΠ΅Ρ€Π²ΠΎΠΉ ячСйкС ( E2 ) ΠΈ скопируйтС ΠΊΠ°Ρ‚Π΅Π³ΠΎΡ€ΠΈΠΈ (Π² нашСм случаС мСсяцы) прямо ΠΏΠΎΠ΄ Π½Π΅ΠΉ ( E3:E14 ).
    • Π£Π³ΠΎΠ» (Ρ‚Π΅Ρ‚Π°) β€” этот столбСц содСрТит Ρ‚Π΅Ρ‚Π°-значСния, ΠΎΡ‚Π²Π΅Ρ‡Π°ΡŽΡ‰ΠΈΠ΅ Π·Π° рисованиС спиц, Π³Π΄Π΅ Π±ΡƒΠ΄ΡƒΡ‚ Ρ€Π°Π·ΠΌΠ΅Ρ‰Π΅Π½Ρ‹ фактичСскиС значСния. Π’Ρ‹ всСгда Π΄ΠΎΠ»ΠΆΠ½Ρ‹ Π²Π²ΠΎΠ΄ΠΈΡ‚ΡŒ Β«0Β» Π² ΠΏΠ΅Ρ€Π²ΡƒΡŽ ячСйку ( F2 ) этого столбца.
    • CSAT Simpson LTD (Radius) ΠΈ CSAT Griffin LTD (Radius) β€” эти столбцы содСрТат значСния радиуса, ΠΈΠ»Π»ΡŽΡΡ‚Ρ€ΠΈΡ€ΡƒΡŽΡ‰ΠΈΠ΅ Ρ€Π΅Π·ΡƒΠ»ΡŒΡ‚Π°Ρ‚Ρ‹ Π΄Π΅ΡΡ‚Π΅Π»ΡŒΠ½ΠΎΡΡ‚ΠΈ ΠΊΠ°ΠΆΠ΄ΠΎΠΉ ΠΊΠΎΠΌΠΏΠ°Π½ΠΈΠΈ Π² Ρ‚Π΅Ρ‡Π΅Π½ΠΈΠ΅ Π³ΠΎΠ΄Π°.

    Π¨Π°Π³ β„– 2. ВычислитС значСния ΡƒΠ³Π»Π° (Ρ‚Π΅Ρ‚Π°).

    Если Π²Ρ‹ ΡƒΠΆΠ΅ вычислили значСния r ΠΈ Ρ‚Π΅Ρ‚Π°, пропуститС эту Ρ‡Π°ΡΡ‚ΡŒ ΠΈ ΠΏΡ€ΠΎΠΊΡ€ΡƒΡ‚ΠΈΡ‚Π΅ Π²Π½ΠΈΠ· Π΄ΠΎ Π¨Π°Π³ β„– 4 .

    На этом шагС наша Ρ†Π΅Π»ΡŒ β€” Ρ€Π°Π²Π½ΠΎΠΌΠ΅Ρ€Π½ΠΎ Π½Π°ΠΌΠ΅Ρ‚ΠΈΡ‚ΡŒ спицы Π½Π° основС количСства ΠΊΠ°Ρ‚Π΅Π³ΠΎΡ€ΠΈΠΉ Π² Π½Π°Π±ΠΎΡ€Π΅ Π΄Π°Π½Π½Ρ‹Ρ…. ΠŸΠΎΡΠΊΠΎΠ»ΡŒΠΊΡƒ ΠΎΠ΄ΠΈΠ½ ΠΏΠΎΠ»Π½Ρ‹ΠΉ ΠΊΡ€ΡƒΠ³ΠΎΠ²ΠΎΠΉ ΠΎΠ±ΠΎΡ€ΠΎΡ‚ Ρ€Π°Π²Π΅Π½ 360 градусам, Ρ‡Ρ‚ΠΎΠ±Ρ‹ Π²Ρ‹ΠΏΠΎΠ»Π½ΠΈΡ‚ΡŒ Π·Π°Π΄Π°Ρ‡Ρƒ, Π²Π°ΠΌ Π½ΡƒΠΆΠ½ΠΎ Ρ€Π°Π·Π΄Π΅Π»ΠΈΡ‚ΡŒ 360 Π½Π° количСство ΠΊΠ°Ρ‚Π΅Π³ΠΎΡ€ΠΈΠΉ Π² вашСм Π½Π°Π±ΠΎΡ€Π΅ Π΄Π°Π½Π½Ρ‹Ρ… (Π² нашСм случаС Π΄Π²Π΅Π½Π°Π΄Ρ†Π°Ρ‚ΡŒ мСсяцСв).

    Π—Π°Ρ‚Π΅ΠΌ слоТитС это число ΠΏΠΎ ΠΌΠ΅Ρ€Π΅ продвиТСния ΠΎΡ‚ нуля Π΄ΠΎ 360.И здСсь Π² ΠΈΠ³Ρ€Ρƒ вступаСт функция COUNTA. По сути, ΠΎΠ½ подсчитываСт количСство нСпустых ячССк Π² ΡƒΠΊΠ°Π·Π°Π½Π½ΠΎΠΌ Π΄ΠΈΠ°ΠΏΠ°Π·ΠΎΠ½Π΅.

    Π‘ΠΊΠΎΠΏΠΈΡ€ΡƒΠΉΡ‚Π΅ эту Ρ„ΠΎΡ€ΠΌΡƒΠ»Ρƒ Π² ячСйку F3 :

    Π‘ ΠΏΠΎΠΌΠΎΡ‰ΡŒΡŽ этой Ρ„ΠΎΡ€ΠΌΡƒΠ»Ρ‹ Π² ячСйкС F3 ΠΈΡΠΏΠΎΠ»ΡŒΠ·ΡƒΠΉΡ‚Π΅ Π΄Ρ€ΡƒΠ³ΡƒΡŽ Ρ„ΠΎΡ€ΠΌΡƒΠ»Ρƒ Π² ячСйкС F4 , Ρ‡Ρ‚ΠΎΠ±Ρ‹ ΡΠ»ΠΎΠΆΠΈΡ‚ΡŒ Π·Π°Π΄Π°Π½Π½ΠΎΠ΅ Π·Π½Π°Ρ‡Π΅Π½ΠΈΠ΅ ΡƒΠ³Π»Π° с суммой всСх Ρ‚Π΅Ρ‚Π°-Π·Π½Π°Ρ‡Π΅Π½ΠΈΠΉ, ΠΊΠΎΡ‚ΠΎΡ€Ρ‹Π΅ ΠΈΠ΄ΡƒΡ‚ ΠΏΠ΅Ρ€Π΅Π΄ Π½ΠΈΠΌ Π² столбцС:

    =F3+360/БЧЕВЧАБВЬ($A$3:$A$14)

    Π’Π°ΠΆΠ½ΠΎ Π·Π°Π±Π»ΠΎΠΊΠΈΡ€ΠΎΠ²Π°Ρ‚ΡŒ Π΄ΠΈΠ°ΠΏΠ°Π·ΠΎΠ½ ячССк ( A3:A14 ), Ρ‡Ρ‚ΠΎΠ±Ρ‹ Π»Π΅Π³ΠΊΠΎ ΡΠΊΠΎΠΏΠΈΡ€ΠΎΠ²Π°Ρ‚ΡŒ Ρ„ΠΎΡ€ΠΌΡƒΠ»Ρƒ Π² ΠΎΡΡ‚Π°Π²ΡˆΠΈΠ΅ΡΡ ячСйки.

    Π’Π΅ΠΏΠ΅Ρ€ΡŒ Π²Ρ‹ΠΏΠΎΠ»Π½ΠΈΡ‚Π΅ Ρ„ΠΎΡ€ΠΌΡƒΠ»Ρƒ для ΠΎΡΡ‚Π°Π»ΡŒΠ½Ρ‹Ρ… ячССк Π² столбцС ( F5:F14 ), Π²Ρ‹Π±Ρ€Π°Π² F4 ΠΈ ΠΏΠ΅Ρ€Π΅Ρ‚Π°Ρ‰ΠΈΠ² ΠΌΠ°Ρ€ΠΊΠ΅Ρ€ заполнСния Π²Π½ΠΈΠ·.

    Π¨Π°Π³ β„– 3. ВычислитС значСния радиуса.

    Π“Ρ€Π°Ρ„ΠΈΠΊ Π² полярных ΠΊΠΎΠΎΡ€Π΄ΠΈΠ½Π°Ρ‚Π°Ρ… Π±ΡƒΠ΄Π΅Ρ‚ ΡΠΎΡΡ‚ΠΎΡΡ‚ΡŒ ΠΈΠ· 10 ΠΊΠΎΠ»Π΅Ρ† Π΄Π°Π½Π½Ρ‹Ρ…, каТдая Ρ€Π°Π΄ΠΈΠ°Π»ΡŒΠ½Π°Ρ Ρ‚ΠΎΡ‡ΠΊΠ° (расстояниС ΠΌΠ΅ΠΆΠ΄Ρƒ Π²Π½ΡƒΡ‚Ρ€Π΅Π½Π½ΠΈΠΌ ΠΈ внСшним краями ΠΊΠΎΠ»ΡŒΡ†Π°) прСдставляСт дСсятипроцСнтноС ΠΏΡ€ΠΈΡ€Π°Ρ‰Π΅Π½ΠΈΠ΅ ΠΏΠΎ шкалС ΠΎΡ‚ 0 Π΄ΠΎ 100.

    ΠŸΠΎΡΠΊΠΎΠ»ΡŒΠΊΡƒ Π±Π°Π»Π»Ρ‹ CSAT Ρ‚Π°ΠΊΠΆΠ΅ ΠΈΠ·ΠΌΠ΅Ρ€ΡΡŽΡ‚ΡΡ ΠΏΠΎ ΠΏΡ€ΠΎΡ†Π΅Π½Ρ‚Π½ΠΎΠΉ шкалС, просто Ρ€Π°Π·Π΄Π΅Π»ΠΈΡ‚Π΅ ΠΊΠ°ΠΆΠ΄ΡƒΡŽ Ρ‚Π°Π±Π»ΠΈΡ†Ρƒ Π±Π°Π»Π»ΠΎΠ² CSAT Π½Π° 10.

    Π’ΠΎΡ‚ ΠΊΠ°ΠΊ это ΡΠ΄Π΅Π»Π°Ρ‚ΡŒ быстро ΠΈ Π»Π΅Π³ΠΊΠΎ. Π§Ρ‚ΠΎΠ±Ρ‹ Π½Π°ΠΉΡ‚ΠΈ значСния радиуса для ΠΏΠ΅Ρ€Π²ΠΎΠΉ ΠΊΠΎΠΌΠΏΠ°Π½ΠΈΠΈ ( Simpson Ltd ), Π²Π²Π΅Π΄ΠΈΡ‚Π΅ эту ΠΊΡ€ΠΎΡˆΠ΅Ρ‡Π½ΡƒΡŽ Ρ„ΠΎΡ€ΠΌΡƒΠ»Ρƒ Π² ячСйку G3 ΠΈ скопируйтС Π΅Π΅ Π² ΠΎΡΡ‚Π°Π»ΡŒΠ½Ρ‹Π΅ ячСйки ( G4:G14 ):

    Π’Π΅ΠΏΠ΅Ρ€ΡŒ ΠΏΠΎ Ρ‚ΠΎΠΌΡƒ ΠΆΠ΅ ΠΏΡ€ΠΈΠ½Ρ†ΠΈΠΏΡƒ посчитайтС радиусы для Π²Ρ‚ΠΎΡ€ΠΎΠΉ ΠΊΠΎΠΌΠΏΠ°Π½ΠΈΠΈ ( Griffin Ltd ):

    Π’ этот ΠΌΠΎΠΌΠ΅Π½Ρ‚ Π²Ρ‹ ΠΌΠΎΠΆΠ΅Ρ‚Π΅ ΠΏΠΎΠ΄ΡƒΠΌΠ°Ρ‚ΡŒ ΠΏΡ€ΠΎ сСбя: Β«Π§Ρ‚ΠΎ, Ссли ΠΌΠΎΠΉ Ρ‚ΠΈΠΏ Π΄Π°Π½Π½Ρ‹Ρ… отличаСтся? Как Π²Ρ‹ ΠΊΠΎΡ€Ρ€Π΅ΠΊΡ‚ΠΈΡ€ΡƒΠ΅Ρ‚Π΅, сравнивая, Π½Π°ΠΏΡ€ΠΈΠΌΠ΅Ρ€, Π΄ΠΎΡ…ΠΎΠ΄, ΠΏΠΎΠ»ΡƒΡ‡Π΅Π½Π½Ρ‹ΠΉ компаниями, с показатСлями CSAT?Β»

    ΠŸΡ€ΠΎΡ‰Π΅ говоря, Π²Ρ‹ Π΄ΠΎΠ»ΠΆΠ½Ρ‹ ΠΏΡ€ΠΎΠ°Π½Π°Π»ΠΈΠ·ΠΈΡ€ΠΎΠ²Π°Ρ‚ΡŒ свои фактичСскиС Π΄Π°Π½Π½Ρ‹Π΅, ΠΎΠΏΡ€Π΅Π΄Π΅Π»ΠΈΡ‚ΡŒ эквивалСнт ΠΎΠ΄Π½ΠΎΠΉ Ρ€Π°Π΄ΠΈΠ°Π»ΡŒΠ½ΠΎΠΉ Ρ‚ΠΎΡ‡ΠΊΠΈ (скаТСм, 50 000 Π΄ΠΎΠ»Π»Π°Ρ€ΠΎΠ² БША) ΠΈ Ρ€Π°Π·Π΄Π΅Π»ΠΈΡ‚ΡŒ всС значСния Π² вашСм Π½Π°Π±ΠΎΡ€Π΅ Π΄Π°Π½Π½Ρ‹Ρ… Π½Π° это число.ΠŸΡ€Π΅Π΄ΠΏΠΎΠ»ΠΎΠΆΠΈΠΌ, Ρ‡Ρ‚ΠΎ какая-Ρ‚ΠΎ компания Π·Π°Ρ€Π°Π±ΠΎΡ‚Π°Π»Π° 250 000 Π΄ΠΎΠ»Π»Π°Ρ€ΠΎΠ² Π² ΠΌΠ°Π΅. Π§Ρ‚ΠΎΠ±Ρ‹ Π½Π°ΠΉΡ‚ΠΈ свой радиус, Ρ€Π°Π·Π΄Π΅Π»ΠΈΡ‚Π΅ 250 000 Π΄ΠΎΠ»Π»Π°Ρ€ΠΎΠ² Π½Π° 50 000. Π’Π°ΠΊ просто.

    Π¨Π°Π³ β„– 4: Π‘ΠΊΠΎΠΏΠΈΡ€ΡƒΠΉΡ‚Π΅ послСдниС значСния радиуса Π²ΠΎ Π²ΡΠΏΠΎΠΌΠΎΠ³Π°Ρ‚Π΅Π»ΡŒΠ½ΡƒΡŽ строку.

    Π—Π°ΠΏΠΎΠ»Π½ΠΈΡ‚Π΅ Ρ‚Π°Π±Π»ΠΈΡ†Ρƒ, скопировав значСния r Π² самом Π½ΠΈΠ·Ρƒ ( G14:h24 ) ΠΊΠ°ΠΆΠ΄ΠΎΠ³ΠΎ столбца Π² ΡΠΎΠΎΡ‚Π²Π΅Ρ‚ΡΡ‚Π²ΡƒΡŽΡ‰ΠΈΠ΅ Ρ„ΠΈΠΊΡ‚ΠΈΠ²Π½Ρ‹Π΅ ячСйки ( G2:h3 ).

    Π¨Π°Π³ β„– 5. РассчитайтС значСния ΠΏΠΎ осям X ΠΈ Y для ΠΊΠ°ΠΆΠ΄ΠΎΠΉ ΠΊΠΎΠΌΠΏΠ°Π½ΠΈΠΈ.

    ΠŸΡ€ΠΈΡˆΠ»ΠΎ врСмя ΠΏΠ΅Ρ€Π΅ΠΉΡ‚ΠΈ ΠΊ ΠΏΡ€Π΅ΠΎΠ±Ρ€Π°Π·ΠΎΠ²Π°Π½ΠΈΡŽ полярных ΠΊΠΎΠΎΡ€Π΄ΠΈΠ½Π°Ρ‚ Π² ΡΠΎΠΎΡ‚Π²Π΅Ρ‚ΡΡ‚Π²ΡƒΡŽΡ‰ΠΈΠ΅ значСния осСй x ΠΈ y.Благодаря Ρ‚Ρ€ΠΈΠ³ΠΎΠ½ΠΎΠΌΠ΅Ρ‚Ρ€ΠΈΠΈ Π²Ρ‹ ΠΌΠΎΠΆΠ΅Ρ‚Π΅ ΠΎΡΡƒΡ‰Π΅ΡΡ‚Π²ΠΈΡ‚ΡŒ ΠΏΠ΅Ρ€Π΅Ρ…ΠΎΠ΄, ΠΈΡΠΏΠΎΠ»ΡŒΠ·ΡƒΡ Π΄Π²Π΅ ΡΠΏΠ΅Ρ†ΠΈΠ°Π»ΡŒΠ½Ρ‹Π΅ Ρ„ΠΎΡ€ΠΌΡƒΠ»Ρ‹, ΠΊΠΎΡ‚ΠΎΡ€Ρ‹Π΅ Π²Ρ‹ ΡΠΎΠ±ΠΈΡ€Π°Π΅Ρ‚Π΅ΡΡŒ Π²Ρ‹ΡƒΡ‡ΠΈΡ‚ΡŒ Π·Π° нСсколько сСкунд.

    Π‘Π½Π°Ρ‡Π°Π»Π° Π½Π°Ρ‡Π½Π΅ΠΌ со Π·Π½Π°Ρ‡Π΅Π½ΠΈΠΉ ΠΏΠΎ оси X. Π’ ячСйку, ΠΏΡ€ΠΈΠΌΡ‹ΠΊΠ°ΡŽΡ‰ΡƒΡŽ ΠΊ Π²ΡΠΏΠΎΠΌΠΎΠ³Π°Ρ‚Π΅Π»ΡŒΠ½ΠΎΠΉ Ρ‚Π°Π±Π»ΠΈΡ†Π΅ ( I2 ), Π²Π²Π΅Π΄ΠΈΡ‚Π΅ ΡΠ»Π΅Π΄ΡƒΡŽΡ‰ΡƒΡŽ Ρ„ΠΎΡ€ΠΌΡƒΠ»Ρƒ:

    Π‘ΠΊΠΎΠΏΠΈΡ€ΡƒΠΉΡ‚Π΅ эту Ρ„ΠΎΡ€ΠΌΡƒΠ»Ρƒ Π² ΠΎΡΡ‚Π°Π»ΡŒΠ½Ρ‹Π΅ ячСйки ΠΏΠΎΠ΄ Π½Π΅ΠΉ ( I3:I14 ).

    Π’Π°ΠΊΠΈΠΌ ΠΆΠ΅ ΠΎΠ±Ρ€Π°Π·ΠΎΠΌ Π²ΡΡ‚Π°Π²ΡŒΡ‚Π΅ эту Ρ„ΠΎΡ€ΠΌΡƒΠ»Ρƒ Π² ячСйку J2 , Ρ‡Ρ‚ΠΎΠ±Ρ‹ Π½Π°ΠΉΡ‚ΠΈ значСния ΠΏΠΎ оси Y, ΠΈ Π²Ρ‹ΠΏΠΎΠ»Π½ΠΈΡ‚Π΅ Π΅Π΅ для ΠΎΡΡ‚Π°Π»ΡŒΠ½Ρ‹Ρ… ячССк ( J3:J14 ):

    Π’Π°ΠΆΠ½ΠΎΠ΅ ΠΏΡ€ΠΈΠΌΠ΅Ρ‡Π°Π½ΠΈΠ΅ . Π˜ΠΌΠ΅ΠΉΡ‚Π΅ Π² Π²ΠΈΠ΄Ρƒ, Ρ‡Ρ‚ΠΎ ячСйка строки Π·Π°Π³ΠΎΠ»ΠΎΠ²ΠΊΠ° (J1) столбца со значСниями ΠΏΠΎ оси Y (столбСц J) Π±ΡƒΠ΄Π΅Ρ‚ Π²Ρ‹ΡΡ‚ΡƒΠΏΠ°Ρ‚ΡŒ Π² качСствС ΠΈΠΌΠ΅Π½ΠΈ ряда, Ρ‚ΠΎ Π΅ΡΡ‚ΡŒ Π·Π½Π°Ρ‡Π΅Π½ΠΈΠ΅ Π² этой ячСйкС ΠΏΠΎΠΏΠ°Π΄Π΅Ρ‚ Π² Π»Π΅Π³Π΅Π½Π΄Ρƒ Π΄ΠΈΠ°Π³Ρ€Π°ΠΌΠΌΡ‹.

    ΠŸΠΎΠ²Ρ‚ΠΎΡ€ΠΈΡ‚Π΅ Ρ‚ΠΎΡ‡Π½ΠΎ Ρ‚Π°ΠΊΠΎΠΉ ΠΆΠ΅ процСсс для вычислСния Π·Π½Π°Ρ‡Π΅Π½ΠΈΠΉ X ΠΈ Y для Π²Ρ‚ΠΎΡ€ΠΎΠΉ ΠΊΠΎΠΌΠΏΠ°Π½ΠΈΠΈ, ΠΈΠ·ΠΌΠ΅Π½ΠΈΠ² Ρ„ΠΎΡ€ΠΌΡƒΠ»Ρƒ для использования Π΄Π°Π½Π½Ρ‹Ρ… Π² столбцС Griffin Ltd:

    Π¨Π°Π³ β„– 6: НастройтС Π²Ρ‚ΠΎΡ€ΡƒΡŽ Π²ΡΠΏΠΎΠΌΠΎΠ³Π°Ρ‚Π΅Π»ΡŒΠ½ΡƒΡŽ Ρ‚Π°Π±Π»ΠΈΡ†Ρƒ для сСтки полярного Π³Ρ€Π°Ρ„ΠΈΠΊΠ°.

    Π”Π°, Π²Ρ‹ Π½Π΅ ΠΎΡΠ»Ρ‹ΡˆΠ°Π»ΠΈΡΡŒ. Π’Π°ΠΌ Π½ΡƒΠΆΠ½Π° Π΅Ρ‰Π΅ ΠΎΠ΄Π½Π° Π²ΡΠΏΠΎΠΌΠΎΠ³Π°Ρ‚Π΅Π»ΡŒΠ½Π°Ρ Ρ‚Π°Π±Π»ΠΈΡ†Π°. К ΡΡ‡Π°ΡΡ‚ΡŒΡŽ, Ρ…ΡƒΠ΄ΡˆΠ΅Π΅ ΡƒΠΆΠ΅ ΠΏΠΎΠ·Π°Π΄ΠΈ, Ρ‚Π°ΠΊ ΠΊΠ°ΠΊ для составлСния Ρ‚Π°Π±Π»ΠΈΡ†Ρ‹ Π½Π΅ трСбуСтся Π΅Π΄ΠΈΠ½ΠΎΠΉ Ρ„ΠΎΡ€ΠΌΡƒΠ»Ρ‹.

    ВзглянитС Π½Π° Π½Π΅Π³ΠΎ:

    По сути, Ρ‚Π°Π±Π»ΠΈΡ†Π° состоит ΠΈΠ· Ρ‚Ρ€Π΅Ρ… элСмСнтов:

    • ΠšΠ°Ρ‡Π΅ΡΡ‚Π²Π΅Π½Π½Π°Ρ шкала (ТСлтая ΠΎΠ±Π»Π°ΡΡ‚ΡŒ ΠΈΠ»ΠΈ N2:N11) – ΠžΡ‚Ρ€Π°ΠΆΠ°Π΅Ρ‚ ΠΈΠ½Ρ‚Π΅Ρ€Π²Π°Π»Ρ‹ Π·Π½Π°Ρ‡Π΅Π½ΠΈΠΉ Π½Π° основС Π²Π°ΡˆΠΈΡ… фактичСских Π΄Π°Π½Π½Ρ‹Ρ….Π—Π°ΠΏΠΎΠ»Π½ΠΈΡ‚Π΅ ячСйки ΠΏΡ€ΠΎΡ†Π΅Π½Ρ‚Π°ΠΌΠΈ, ΠΊΠ°ΠΊ ΠΏΠΎΠΊΠ°Π·Π°Π½ΠΎ Π½Π° ΡΠΊΡ€ΠΈΠ½ΡˆΠΎΡ‚Π΅. Π’ качСствС ΠΏΡ€ΠΈΠΌΠ΅Ρ€Π° Π°Π»ΡŒΡ‚Π΅Ρ€Π½Π°Ρ‚ΠΈΠ²Π½Ρ‹Ρ… Π΄Π°Π½Π½Ρ‹Ρ…, Ссли Π±Ρ‹ ΠΌΡ‹ Π°Π½Π°Π»ΠΈΠ·ΠΈΡ€ΠΎΠ²Π°Π»ΠΈ Π΄ΠΎΡ…ΠΎΠ΄, упомянутый Ρ€Π°Π½Π΅Π΅, этот столбСц увСличился Π±Ρ‹ с 50 000 Π΄ΠΎ 500 000 Π΄ΠΎΠ»Π»Π°Ρ€ΠΎΠ².
    • Π‘Ρ‚Ρ€ΠΎΠΊΠ° Π·Π°Π³ΠΎΠ»ΠΎΠ²ΠΊΠ° (красная ΠΎΠ±Π»Π°ΡΡ‚ΡŒ ΠΈΠ»ΠΈ O1:Z1) β€” содСрТит всС названия ΠΊΠ°Ρ‚Π΅Π³ΠΎΡ€ΠΈΠΉ, ΠΏΠΎΠ»ΡƒΡ‡Π΅Π½Π½Ρ‹Π΅ ΠΈΠ· исходной Ρ‚Π°Π±Π»ΠΈΡ†Ρ‹ Π΄Π°Π½Π½Ρ‹Ρ…, просто располоТСнныС Π²Π΅Ρ€Ρ‚ΠΈΠΊΠ°Π»ΡŒΠ½ΠΎ.
    • ЗначСния сСтки (зСлСная ΠΎΠ±Π»Π°ΡΡ‚ΡŒ ΠΈΠ»ΠΈ O2:Z11) β€” Π­Ρ‚ΠΈ значСния раздСлят Π±ΡƒΠ΄ΡƒΡ‰ΠΈΠ΅ ΠΊΠΎΠ»ΡŒΡ†Π° Π΄Π°Π½Π½Ρ‹Ρ… Π½Π° Ρ€Π°Π²Π½Ρ‹Π΅ части, очСрчивая сСтку Π³Ρ€Π°Ρ„ΠΈΠΊΠ°.ΠŸΡ€ΠΎΡΡ‚ΠΎ Π²Ρ‹Π±Π΅Ρ€ΠΈΡ‚Π΅ число ΠΈΠ· Π²ΠΎΠ·Π΄ΡƒΡ…Π° ΠΈ скопируйтС Π΅Π³ΠΎ Π²ΠΎ всС ячСйки Π² ΠΏΡ€Π΅Π΄Π΅Π»Π°Ρ… Π΄ΠΈΠ°ΠΏΠ°Π·ΠΎΠ½Π°.

    Π¨Π°Π³ β„– 7: Π‘ΠΎΠ·Π΄Π°ΠΉΡ‚Π΅ Π½Π°Π±ΠΎΡ€ ΠΊΠΎΠ»ΡŒΡ†Π΅Π²Ρ‹Ρ… Π΄ΠΈΠ°Π³Ρ€Π°ΠΌΠΌ.

    НаконСц-Ρ‚ΠΎ Π²Ρ‹ собрали всС Π½Π΅ΠΎΠ±Ρ…ΠΎΠ΄ΠΈΠΌΡ‹Π΅ графичСскиС Π΄Π°Π½Π½Ρ‹Π΅ β€” это Π±Ρ‹Π»ΠΎ довольно напряТСнно. ΠŸΠΎΠΏΡ€ΠΎΡ‰Π°ΠΉΡ‚Π΅ΡΡŒ с функциями ΠΈ Ρ„ΠΎΡ€ΠΌΡƒΠ»Π°ΠΌΠΈ, ΠΏΠΎΡ‚ΠΎΠΌΡƒ Ρ‡Ρ‚ΠΎ Ρ‚Π΅ΠΏΠ΅Ρ€ΡŒ Π²Ρ‹ ΠΌΠΎΠΆΠ΅Ρ‚Π΅ Π½Π°Ρ‡Π°Ρ‚ΡŒ ΡΡ‚Ρ€ΠΎΠΈΡ‚ΡŒ сам полярный Π³Ρ€Π°Ρ„ΠΈΠΊ.

    НачнитС с настройки полярной плоскости, создав 10 ΠΊΠΎΠ»ΡŒΡ†Π΅Π²Ρ‹Ρ… Π΄ΠΈΠ°Π³Ρ€Π°ΠΌΠΌ, располоТСнных Π΄Ρ€ΡƒΠ³ Π½Π°Π΄ Π΄Ρ€ΡƒΠ³ΠΎΠΌ:

    1. Π’Ρ‹Π΄Π΅Π»ΠΈΡ‚Π΅ всС значСния сСтки ΠΈΠ· Π²Ρ‚ΠΎΡ€ΠΎΠΉ Π²ΡΠΏΠΎΠΌΠΎΠ³Π°Ρ‚Π΅Π»ΡŒΠ½ΠΎΠΉ Ρ‚Π°Π±Π»ΠΈΡ†Ρ‹ ( O2:Z11 ).
    2. ΠŸΠ΅Ρ€Π΅ΠΉΠ΄ΠΈΡ‚Π΅ Π½Π° Π²ΠΊΠ»Π°Π΄ΠΊΡƒ Вставка .
    3. НаТмитС ΠΊΠ½ΠΎΠΏΠΊΡƒ Β« Π’ΡΡ‚Π°Π²ΠΈΡ‚ΡŒ ΠΊΡ€ΡƒΠ³ΠΎΠ²ΡƒΡŽ ΠΈΠ»ΠΈ ΠΊΠΎΠ»ΡŒΡ†Π΅Π²ΡƒΡŽ Π΄ΠΈΠ°Π³Ρ€Π°ΠΌΠΌΡƒ Β».
    4. Π’Ρ‹Π±Π΅Ρ€ΠΈΡ‚Π΅ Β« ΠŸΠΎΠ½Ρ‡ΠΈΠΊ Β».

    Π’ Ρ€Π΅Π·ΡƒΠ»ΡŒΡ‚Π°Ρ‚Π΅ Excel Π΄ΠΎΠ»ΠΆΠ΅Π½ Π΄Π°Ρ‚ΡŒ Π²Π°ΠΌ Π½Π°Π±ΠΎΡ€ ΠΈΠ· 10 ΠΊΠΎΠ»Π΅Ρ†.

    Иногда Excel Π½Π΅ ΠΌΠΎΠΆΠ΅Ρ‚ ΠΏΡ€Π°Π²ΠΈΠ»ΡŒΠ½ΠΎ ΠΏΡ€ΠΎΡ‡ΠΈΡ‚Π°Ρ‚ΡŒ ваши Π΄Π°Π½Π½Ρ‹Π΅. Π§Ρ‚ΠΎΠ±Ρ‹ ΠΎΠ±ΠΎΠΉΡ‚ΠΈ ΠΏΡ€ΠΎΠ±Π»Π΅ΠΌΡƒ, Ссли это ΠΏΡ€ΠΎΠΈΠ·ΠΎΠΉΠ΄Π΅Ρ‚ с Π²Π°ΠΌΠΈ, слСдуйтС нСскольким простым инструкциям, Ρ‡Ρ‚ΠΎΠ±Ρ‹ ΡΠ»ΠΎΠΆΠΈΡ‚ΡŒ Π΄ΠΈΠ°Π³Ρ€Π°ΠΌΠΌΡ‹ Π²Ρ€ΡƒΡ‡Π½ΡƒΡŽ. Для наглядности ΠΏΡ€Π΅Π΄ΠΏΠΎΠ»ΠΎΠΆΠΈΠΌ, Ρ‡Ρ‚ΠΎ вмСсто этого Π²Ρ‹ Ρ€Π΅ΡˆΠΈΠ»ΠΈ ΠΏΡ€ΠΎΠ°Π½Π°Π»ΠΈΠ·ΠΈΡ€ΠΎΠ²Π°Ρ‚ΡŒ Π΄Π°Π½Π½Ρ‹Π΅ Π·Π° восСмь мСсяцСв.

    Π‘Π½Π°Ρ‡Π°Π»Π° Π²Ρ‹Π±Π΅Ρ€ΠΈΡ‚Π΅ Π»ΡŽΠ±ΡƒΡŽ ΠΏΡƒΡΡ‚ΡƒΡŽ ячСйку ΠΈ постройтС ΠΏΡƒΡΡ‚ΡƒΡŽ ΠΊΠΎΠ»ΡŒΡ†Π΅Π²ΡƒΡŽ Π΄ΠΈΠ°Π³Ρ€Π°ΠΌΠΌΡƒ, слСдуя описанным Π²Ρ‹ΡˆΠ΅ шагам.

    Π—Π°Ρ‚Π΅ΠΌ Ρ‰Π΅Π»ΠΊΠ½ΠΈΡ‚Π΅ ΠΏΡ€Π°Π²ΠΎΠΉ ΠΊΠ½ΠΎΠΏΠΊΠΎΠΉ ΠΌΡ‹ΡˆΠΈ Π³Ρ€Π°Ρ„ΠΈΠΊ ΠΈ Π²Ρ‹Π±Π΅Ρ€ΠΈΡ‚Π΅ Β« Π’Ρ‹Π±Ρ€Π°Ρ‚ΡŒ Π΄Π°Π½Π½Ρ‹Π΅. Β»

    ПослС этого Π² Π΄ΠΈΠ°Π»ΠΎΠ³ΠΎΠ²ΠΎΠΌ ΠΎΠΊΠ½Π΅ Select Data Source Π½Π°ΠΆΠΌΠΈΡ‚Π΅ ΠΊΠ½ΠΎΠΏΠΊΡƒ Β« Add Β».

    Π’ ΠΏΠΎΠ»Π΅ Edit Series Π²Ρ‹Π±Π΅Ρ€ΠΈΡ‚Π΅ всС значСния сСтки Π² ΠΏΠ΅Ρ€Π²ΠΎΠΉ строкС ( O2:V2 ) ΠΈ Π½Π°ΠΆΠΌΠΈΡ‚Π΅ Β« OK Β».

    Как Π²Ρ‹ ΡƒΠΆΠ΅ догадались, ΠΏΡ€ΠΎΠΌΠΎΠΉΡ‚Π΅ ΠΈ ΠΏΠΎΠ²Ρ‚ΠΎΡ€ΠΈΡ‚Π΅ для ΠΊΠ°ΠΆΠ΄ΠΎΠ³ΠΎ ряда, Ρ‡Ρ‚ΠΎΠ±Ρ‹ ΠΏΠΎΠ»ΡƒΡ‡ΠΈΡ‚ΡŒ Ρ‚Π΅ ΠΆΠ΅ самыС 10 ΠΊΠΎΠ»Π΅Ρ†, нанСсСнных Π½Π° Π³Ρ€Π°Ρ„ΠΈΠΊ.

    Π¨Π°Π³ β„– 8. Π£ΠΌΠ΅Π½ΡŒΡˆΠΈΡ‚Π΅ Ρ€Π°Π·ΠΌΠ΅Ρ€ отвСрстия Π±ΡƒΠ±Π»ΠΈΠΊΠ°.

    Как Π²ΠΈΠ΄ΠΈΡ‚Π΅, всС ΠΊΠΎΠ»ΡŒΡ†Π° сТаты вмСстС Π²Π΄Π°Π»ΠΈ ΠΎΡ‚ Ρ†Π΅Π½Ρ‚Ρ€Π°. Π”Π°Π²Π°ΠΉΡ‚Π΅ ΠΈΠ·ΠΌΠ΅Π½ΠΈΠΌ это, ΡƒΠΌΠ΅Π½ΡŒΡˆΠΈΠ² Ρ€Π°Π·ΠΌΠ΅Ρ€ отвСрстия для ΠΏΠΎΠ½Ρ‡ΠΈΠΊΠ°.

    1. Π©Π΅Π»ΠΊΠ½ΠΈΡ‚Π΅ ΠΏΡ€Π°Π²ΠΎΠΉ ΠΊΠ½ΠΎΠΏΠΊΠΎΠΉ ΠΌΡ‹ΡˆΠΈ любоС ΠΊΠΎΠ»ΡŒΡ†ΠΎ Π΄Π°Π½Π½Ρ‹Ρ….
    2. Π’Ρ‹Π±Π΅Ρ€ΠΈΡ‚Π΅ Β« Π€ΠΎΡ€ΠΌΠ°Ρ‚ сСрии Π΄Π°Π½Π½Ρ‹Ρ…. Β»

    Π’ появившСйся ΠΏΠ°Π½Π΅Π»ΠΈ Π·Π°Π΄Π°Ρ‡ ΠΈΠ·ΠΌΠ΅Π½ΠΈΡ‚Π΅ Π·Π½Π°Ρ‡Π΅Π½ΠΈΠ΅ Ρ€Π°Π·ΠΌΠ΅Ρ€Π° отвСрстия Π±ΡƒΠ±Π»ΠΈΠΊΠ° ΠΏΠΎ ΡƒΠΌΠΎΠ»Ρ‡Π°Π½ΠΈΡŽ, Ρ‡Ρ‚ΠΎΠ±Ρ‹ ΠΏΡ€ΠΎΠΈΠ·ΠΎΡˆΠ»ΠΎ Π²ΠΎΠ»ΡˆΠ΅Π±ΡΡ‚Π²ΠΎ:

    1. ΠŸΠ΅Ρ€Π΅ΠΉΠ΄ΠΈΡ‚Π΅ Π½Π° Π²ΠΊΠ»Π°Π΄ΠΊΡƒ ΠžΠΏΡ†ΠΈΠΈ сСрии .
    2. УстановитС для ΠΏΠ°Ρ€Π°ΠΌΠ΅Ρ‚Ρ€Π° Π Π°Π·ΠΌΠ΅Ρ€ отвСрстия Π±ΡƒΠ±Π»ΠΈΠΊΠ° Π·Π½Π°Ρ‡Π΅Π½ΠΈΠ΅ Β« 10%. Β»

    Π¨Π°Π³ β„– 9. НастройтС сСтку Π΄ΠΈΠ°Π³Ρ€Π°ΠΌΠΌΡ‹.

    Π’ Ρ‚ΠΎΠΉ ΠΆΠ΅ области Π·Π°Π΄Π°Ρ‡ ΠΏΡ€Π΅ΠΎΠ±Ρ€Π°Π·ΡƒΠΉΡ‚Π΅ ΠΊΠΎΠ»ΡŒΡ†Π° Π² сСтку, Π²Ρ‹ΠΏΠΎΠ»Π½ΠΈΠ² ΡΠ»Π΅Π΄ΡƒΡŽΡ‰ΠΈΠ΅ простыС дСйствия:

    1. ΠŸΠ΅Ρ€Π΅ΠΉΠ΄ΠΈΡ‚Π΅ Π½Π° Π²ΠΊΠ»Π°Π΄ΠΊΡƒ Fill & Line .
    2. Π’ Ρ€Π°Π·Π΄Π΅Π»Π΅ Β« Π—Π°ΠΏΠΎΠ»Π½ΠΈΡ‚ΡŒ, Β» Π²Ρ‹Π±Π΅Ρ€ΠΈΡ‚Π΅ Β« Π‘Π΅Π· заполнСния». Β»
    3. Π’ Ρ€Π°Π·Π΄Π΅Π»Π΅ Β« Π“Ρ€Π°Π½ΠΈΡ†Π°, Β» Π²Ρ‹Π±Π΅Ρ€ΠΈΡ‚Π΅ Β« Бплошная линия». Β»
    4. Π©Π΅Π»ΠΊΠ½ΠΈΡ‚Π΅ Π·Π½Π°Ρ‡ΠΎΠΊ Β« Π¦Π²Π΅Ρ‚ ΠΊΠΎΠ½Ρ‚ΡƒΡ€Π° Β», Ρ‡Ρ‚ΠΎΠ±Ρ‹ ΠΎΡ‚ΠΊΡ€Ρ‹Ρ‚ΡŒ Ρ†Π²Π΅Ρ‚ΠΎΠ²ΡƒΡŽ ΠΏΠ°Π»ΠΈΡ‚Ρ€Ρƒ ΠΈ Π²Ρ‹Π±Ρ€Π°Ρ‚ΡŒ свСтло-сСрый.
    5. УстановитС ΡˆΠΈΡ€ΠΈΠ½Ρƒ Π½Π° Β« 5 pt. Β»

    ΠŸΡ€ΠΎΠΌΠΎΠΉΡ‚Π΅ ΠΈ ΠΏΠΎΠ²Ρ‚ΠΎΡ€ΠΈΡ‚Π΅ для ΠΎΡΡ‚Π°Π»ΡŒΠ½Ρ‹Ρ… ΠΊΠΎΠ»Π΅Ρ†.

    Π¨Π°Π³ β„– 10. Π”ΠΎΠ±Π°Π²ΡŒΡ‚Π΅ Π΄Π°Π½Π½Ρ‹Π΅ Π΄ΠΈΠ°Π³Ρ€Π°ΠΌΠΌΡ‹.

    Π’Π΅ΠΏΠ΅Ρ€ΡŒ, ΠΊΠΎΠ³Π΄Π° основа Π·Π°Π»ΠΎΠΆΠ΅Π½Π°, Π΄ΠΎΠ±Π°Π²ΡŒΡ‚Π΅ Π½Π° Π΄ΠΈΠ°Π³Ρ€Π°ΠΌΠΌΡƒ значСния x ΠΈ y ΠΈΠ· ΠΏΠ΅Ρ€Π²ΠΎΠΉ Π²ΡΠΏΠΎΠΌΠΎΠ³Π°Ρ‚Π΅Π»ΡŒΠ½ΠΎΠΉ Ρ‚Π°Π±Π»ΠΈΡ†Ρ‹.

    1. Π’Ρ‹Π΄Π΅Π»ΠΈΡ‚Π΅ всС значСния ΠΏΠΎ осям X ΠΈ Y, ΠΈΠ»Π»ΡŽΡΡ‚Ρ€ΠΈΡ€ΡƒΡŽΡ‰ΠΈΠ΅ Ρ€Π΅Π·ΡƒΠ»ΡŒΡ‚Π°Ρ‚Ρ‹ CSAT ΠΏΠ΅Ρ€Π²ΠΎΠΉ ΠΊΠΎΠΌΠΏΠ°Π½ΠΈΠΈ (Simpson Ltd), Π° Ρ‚Π°ΠΊΠΆΠ΅ ячСйки строки Π·Π°Π³ΠΎΠ»ΠΎΠ²ΠΊΠ° ( I1:J14 ) ΠΈ скопируйтС Π΄Π°Π½Π½Ρ‹Π΅ (Ρ‰Π΅Π»ΠΊΠ½ΠΈΡ‚Π΅ ΠΏΡ€Π°Π²ΠΎΠΉ ΠΊΠ½ΠΎΠΏΠΊΠΎΠΉ ΠΌΡ‹ΡˆΠΈ ΠΈ Π²Ρ‹Π±Π΅Ρ€ΠΈΡ‚Π΅ ΠšΠΎΠΏΠΈΡ€ΠΎΠ²Π°Ρ‚ΡŒ ).
    2. Π’Ρ‹Π±Π΅Ρ€ΠΈΡ‚Π΅ ΠΎΠ±Π»Π°ΡΡ‚ΡŒ Π΄ΠΈΠ°Π³Ρ€Π°ΠΌΠΌΡ‹.
    3. ΠŸΠ΅Ρ€Π΅ΠΉΠ΄ΠΈΡ‚Π΅ Π½Π° Π²ΠΊΠ»Π°Π΄ΠΊΡƒ Главная .
    4. НаТмитС ΠΊΠ½ΠΎΠΏΠΊΡƒ Β« Π’ΡΡ‚Π°Π²ΠΈΡ‚ΡŒ Β».
    5. Π’Ρ‹Π±Π΅Ρ€ΠΈΡ‚Π΅ Β« Π‘ΠΏΠ΅Ρ†ΠΈΠ°Π»ΡŒΠ½Π°Ρ вставка. Β»

    Π’ ΠΊΡ€ΠΎΡˆΠ΅Ρ‡Π½ΠΎΠΌ Π΄ΠΈΠ°Π»ΠΎΠ³ΠΎΠ²ΠΎΠΌ ΠΎΠΊΠ½Π΅ Π‘ΠΏΠ΅Ρ†ΠΈΠ°Π»ΡŒΠ½Π°Ρ вставка Π²Ρ‹ΠΏΠΎΠ»Π½ΠΈΡ‚Π΅ ΡΠ»Π΅Π΄ΡƒΡŽΡ‰ΠΈΠ΅ дСйствия:

    1. Π’ Ρ€Π°Π·Π΄Π΅Π»Π΅ Β« Π”ΠΎΠ±Π°Π²ΠΈΡ‚ΡŒ ячСйки ΠΊΠ°ΠΊ Β» Π²Ρ‹Π±Π΅Ρ€ΠΈΡ‚Π΅ Β« Новая сСрия». Β»
    2. Π’ Ρ€Π°Π·Π΄Π΅Π»Π΅ Β« Π·Π½Π°Ρ‡Π΅Π½ΠΈΠΉ (Y) in, Β» Π²Ρ‹Π±Π΅Ρ€ΠΈΡ‚Π΅ Β« столбцов». Β»
    3. УстановитС Ρ„Π»Π°ΠΆΠΊΠΈ «Названия сСрий Π² ΠΏΠ΅Ρ€Π²ΠΎΠΉ строкС Β» ΠΈ Β«ΠšΠ°Ρ‚Π΅Π³ΠΎΡ€ΠΈΠΈ (ΠΌΠ΅Ρ‚ΠΊΠΈ X) Π² ΠΏΠ΅Ρ€Π²ΠΎΠΌ столбцС Β».
    4. НаТмитС Β« ОК. Β»

    ΠŸΠΎΠ²Ρ‚ΠΎΡ€ΠΈΡ‚Π΅ процСсс, Ρ‡Ρ‚ΠΎΠ±Ρ‹ Π΄ΠΎΠ±Π°Π²ΠΈΡ‚ΡŒ Π΄Π°Π½Π½Ρ‹Π΅ Π΄ΠΈΠ°Π³Ρ€Π°ΠΌΠΌΡ‹, связанныС со Π²Ρ‚ΠΎΡ€ΠΎΠΉ ΠΊΠΎΠΌΠΏΠ°Π½ΠΈΠ΅ΠΉ (Griffin Ltd).

    Π¨Π°Π³ β„– 11. Π˜Π·ΠΌΠ΅Π½ΠΈΡ‚Π΅ Ρ‚ΠΈΠΏ Π΄ΠΈΠ°Π³Ρ€Π°ΠΌΠΌΡ‹ для вставлСнного ряда Π΄Π°Π½Π½Ρ‹Ρ….

    Π’Π΅ΠΏΠ΅Ρ€ΡŒ ΠΈΠ·ΠΌΠ΅Π½ΠΈΡ‚Π΅ Ρ‚ΠΈΠΏ Π΄ΠΈΠ°Π³Ρ€Π°ΠΌΠΌΡ‹ ΠΎΠ±ΠΎΠΈΡ… Π½Π΅Π΄Π°Π²Π½ΠΎ Π΄ΠΎΠ±Π°Π²Π»Π΅Π½Π½Ρ‹Ρ… рядов, ΠΏΡ€Π΅Π΄ΡΡ‚Π°Π²Π»ΡΡŽΡ‰ΠΈΡ… фактичСскиС значСния.

    1. Π©Π΅Π»ΠΊΠ½ΠΈΡ‚Π΅ ΠΏΡ€Π°Π²ΠΎΠΉ ΠΊΠ½ΠΎΠΏΠΊΠΎΠΉ ΠΌΡ‹ΡˆΠΈ любой ΠΈΠ· рядов, ΠΏΡ€Π΅Π΄ΡΡ‚Π°Π²Π»ΡΡŽΡ‰ΠΈΡ… фактичСскиС значСния (Π»ΠΈΠ±ΠΎ Series Β«Simpson LtdΒ» , Π»ΠΈΠ±ΠΎ Series Β«Griffin LtdΒ» ).
    2. Π’Ρ‹Π±Π΅Ρ€ΠΈΡ‚Π΅ Β« Π˜Π·ΠΌΠ΅Π½ΠΈΡ‚ΡŒ Ρ‚ΠΈΠΏ Π΄ΠΈΠ°Π³Ρ€Π°ΠΌΠΌΡ‹ сСрии. Β»

    Оказавшись Ρ‚Π°ΠΌ, ΠΈΠ·ΠΌΠ΅Π½ΠΈΡ‚Π΅ Ρ‚ΠΈΠΏ Π΄ΠΈΠ°Π³Ρ€Π°ΠΌΠΌΡ‹ для сСрии Β«Simpson LtdΒ» ΠΈ сСрии Β«Griffin LtdΒ» Π½Π° Β«Scatter with Smooth Lines and MarkersΒ».

    Π¨Π°Π³ β„– 12. Π˜Π·ΠΌΠ΅Π½ΠΈΡ‚Π΅ ΠΌΠ°ΡΡˆΡ‚Π°Π± Π³ΠΎΡ€ΠΈΠ·ΠΎΠ½Ρ‚Π°Π»ΡŒΠ½ΠΎΠΉ ΠΈ Π²Π΅Ρ€Ρ‚ΠΈΠΊΠ°Π»ΡŒΠ½ΠΎΠΉ осСй.

    ПослС появлСния Π²ΡΠΏΠ»Ρ‹Π²Π°ΡŽΡ‰ΠΈΡ… осСй Π΄ΠΈΠ°Π³Ρ€Π°ΠΌΠΌΡ‹ ΠΈΠ·ΠΌΠ΅Π½ΠΈΡ‚Π΅ Π΄ΠΈΠ°ΠΏΠ°Π·ΠΎΠ½Ρ‹ шкал Π³ΠΎΡ€ΠΈΠ·ΠΎΠ½Ρ‚Π°Π»ΡŒΠ½ΠΎΠΉ ΠΈ Π²Π΅Ρ€Ρ‚ΠΈΠΊΠ°Π»ΡŒΠ½ΠΎΠΉ осСй для Π΄ΠΈΠ°Π³Ρ€Π°ΠΌΠΌΡ‹, Ρ‡Ρ‚ΠΎΠ±Ρ‹ Ρ‚ΠΎΡ‡Π½ΠΎ ΠΎΡ‚Ρ€Π°ΠΆΠ°Ρ‚ΡŒ нанСсСнныС Π½Π° Π½Π΅Π΅ Π΄Π°Π½Π½Ρ‹Π΅.

    1. Π©Π΅Π»ΠΊΠ½ΠΈΡ‚Π΅ ΠΏΡ€Π°Π²ΠΎΠΉ ΠΊΠ½ΠΎΠΏΠΊΠΎΠΉ ΠΌΡ‹ΡˆΠΈ ΠΏΠΎ Π²Π΅Ρ€Ρ‚ΠΈΠΊΠ°Π»ΡŒΠ½ΠΎΠΉ оси.
    2. Π’Ρ‹Π±Π΅Ρ€ΠΈΡ‚Π΅ Β« Π€ΠΎΡ€ΠΌΠ°Ρ‚ оси. Β»

    Когда появится панСль Π·Π°Π΄Π°Ρ‡, ΠΎΠΏΡ€Π΅Π΄Π΅Π»ΠΈΡ‚Π΅ Π½ΠΎΠ²Ρ‹Π΅ Π΄ΠΈΠ°ΠΏΠ°Π·ΠΎΠ½Ρ‹ ΠΌΠ°ΡΡˆΡ‚Π°Π±Π° оси:

    1. ΠŸΠ΅Ρ€Π΅ΠΉΠ΄ΠΈΡ‚Π΅ Π½Π° Π²ΠΊΠ»Π°Π΄ΠΊΡƒ ΠŸΠ°Ρ€Π°ΠΌΠ΅Ρ‚Ρ€Ρ‹ оси .
    2. УстановитС для ΠΏΠ°Ρ€Π°ΠΌΠ΅Ρ‚Ρ€Π° ΠœΠΈΠ½ΠΈΠΌΠ°Π»ΡŒΠ½Ρ‹Π΅ Π³Ρ€Π°Π½ΠΈΡ†Ρ‹ Π·Π½Π°Ρ‡Π΅Π½ΠΈΠ΅ Β«-10Β».
    3. Π˜Π·ΠΌΠ΅Π½ΠΈΡ‚Π΅ Π·Π½Π°Ρ‡Π΅Π½ΠΈΠ΅ ΠΏΠ°Ρ€Π°ΠΌΠ΅Ρ‚Ρ€Π° ΠœΠ°ΠΊΡΠΈΠΌΠ°Π»ΡŒΠ½Ρ‹Π΅ Π³Ρ€Π°Π½ΠΈΡ†Ρ‹ Π½Π° Β«10Β».

    ПослС этого ΠΏΠ΅Ρ€Π΅ΠΉΠ΄ΠΈΡ‚Π΅ ΠΊ Π³ΠΎΡ€ΠΈΠ·ΠΎΠ½Ρ‚Π°Π»ΡŒΠ½ΠΎΠΉ оси ΠΈ сдСлайтС Ρ‚ΠΎ ΠΆΠ΅ самоС.

    Π¨Π°Π³ β„– 13. Π£Π΄Π°Π»ΠΈΡ‚Π΅ Π»ΠΈΠ½ΠΈΠΈ сСтки, оси ΠΈ Π½Π΅Π½ΡƒΠΆΠ½Ρ‹Π΅ элСмСнты Π»Π΅Π³Π΅Π½Π΄Ρ‹.

    ΠžΡ‡ΠΈΡΡ‚ΠΈΡ‚Π΅ Π³Ρ€Π°Ρ„ΠΈΠΊ, ΡƒΠ΄Π°Π»ΠΈΠ² элСмСнты Π΄ΠΈΠ°Π³Ρ€Π°ΠΌΠΌΡ‹, ΠΊΠΎΡ‚ΠΎΡ€Ρ‹Π΅ Π½Π΅ ΠΈΠΌΠ΅ΡŽΡ‚ Π½ΠΈΠΊΠ°ΠΊΠΎΠ³ΠΎ практичСского значСния: Π»ΠΈΠ½ΠΈΠΈ сСтки, оси, Π° Ρ‚Π°ΠΊΠΆΠ΅ всС элСмСнты Π»Π΅Π³Π΅Π½Π΄Ρ‹ β€” Π·Π° ΠΈΡΠΊΠ»ΡŽΡ‡Π΅Π½ΠΈΠ΅ΠΌ Π΄Π²ΡƒΡ…, ΠΊΠΎΡ‚ΠΎΡ€Ρ‹Π΅ Π²Π°ΠΌ Π΄Π΅ΠΉΡΡ‚Π²ΠΈΡ‚Π΅Π»ΡŒΠ½ΠΎ Π½ΡƒΠΆΠ½Ρ‹ (ΠΎΡ‚ΠΌΠ΅Ρ‚ΠΊΠ° ΠΈΠ½Ρ„ΠΎΡ€ΠΌΠ°Ρ†ΠΈΠΈ ΠΎ ΠΊΠΎΠΌΠΏΠ°Π½ΠΈΠΈ).

    Для этого Π΄Π²Π°ΠΆΠ΄Ρ‹ Ρ‰Π΅Π»ΠΊΠ½ΠΈΡ‚Π΅ ΠΊΠ°ΠΆΠ΄Ρ‹ΠΉ элСмСнт, Π·Π°Ρ‚Π΅ΠΌ снова Ρ‰Π΅Π»ΠΊΠ½ΠΈΡ‚Π΅ Π΅Π³ΠΎ ΠΏΡ€Π°Π²ΠΎΠΉ ΠΊΠ½ΠΎΠΏΠΊΠΎΠΉ ΠΌΡ‹ΡˆΠΈ ΠΈ Π²Ρ‹Π±Π΅Ρ€ΠΈΡ‚Π΅ Β« Π£Π΄Π°Π»ΠΈΡ‚ΡŒ. Β»

    Π¨Π°Π³ β„– 14. Π”ΠΎΠ±Π°Π²ΡŒΡ‚Π΅ ΠΌΠ΅Ρ‚ΠΊΠΈ Π΄Π°Π½Π½Ρ‹Ρ….

    По ΠΌΠ΅Ρ€Π΅ Ρ‚ΠΎΠ³ΠΎ, ΠΊΠ°ΠΊ ΠΌΡ‹ постСпСнно приблиТаСмся ΠΊ ΠΊΠΎΠ½Ρ†Ρƒ нашСго Π³Ρ€Π°Π½Π΄ΠΈΠΎΠ·Π½ΠΎΠ³ΠΎ ΠΏΡ€ΠΈΠΊΠ»ΡŽΡ‡Π΅Π½ΠΈΡ с Excel, ΠΏΡ€ΠΈΡˆΠ»ΠΎ врСмя Π΄ΠΎΠ±Π°Π²ΠΈΡ‚ΡŒ ΠΌΠ΅Ρ‚ΠΊΠΈ Π΄Π°Π½Π½Ρ‹Ρ…, ΠΏΡ€Π΅Π΄ΡΡ‚Π°Π²Π»ΡΡŽΡ‰ΠΈΠ΅ ΠΊΠ°ΠΆΠ΄ΡƒΡŽ ΠΊΠ°Ρ‡Π΅ΡΡ‚Π²Π΅Π½Π½ΡƒΡŽ ΠΊΠ°Ρ‚Π΅Π³ΠΎΡ€ΠΈΡŽ Π² вашСм Π½Π°Π±ΠΎΡ€Π΅ Π΄Π°Π½Π½Ρ‹Ρ….

    Π©Π΅Π»ΠΊΠ½ΠΈΡ‚Π΅ ΠΏΡ€Π°Π²ΠΎΠΉ ΠΊΠ½ΠΎΠΏΠΊΠΎΠΉ ΠΌΡ‹ΡˆΠΈ внСшнСС ΠΊΠΎΠ»ΡŒΡ†ΠΎ ( БСрия Β«10Β» ) ΠΈ Π²Ρ‹Π±Π΅Ρ€ΠΈΡ‚Π΅ Β« Π”ΠΎΠ±Π°Π²ΠΈΡ‚ΡŒ ΠΌΠ΅Ρ‚ΠΊΠΈ Π΄Π°Π½Π½Ρ‹Ρ…Β». Β»

    Π¨Π°Π³ β„– 15. НастройтС ΠΌΠ΅Ρ‚ΠΊΠΈ Π΄Π°Π½Π½Ρ‹Ρ….

    По сути, всС, Ρ‡Ρ‚ΠΎ Π²Π°ΠΌ Π½ΡƒΠΆΠ½ΠΎ ΡΠ΄Π΅Π»Π°Ρ‚ΡŒ здСсь, это Π·Π°ΠΌΠ΅Π½ΠΈΡ‚ΡŒ ΠΌΠ΅Ρ‚ΠΊΠΈ Π΄Π°Π½Π½Ρ‹Ρ… ΠΏΠΎ ΡƒΠΌΠΎΠ»Ρ‡Π°Π½ΠΈΡŽ ΠΈΠΌΠ΅Π½Π°ΠΌΠΈ ΠΊΠ°Ρ‚Π΅Π³ΠΎΡ€ΠΈΠΉ ΠΈΠ· Ρ‚Π°Π±Π»ΠΈΡ†Ρ‹, содСрТащСй ваши фактичСскиС Π΄Π°Π½Π½Ρ‹Π΅.

    Π©Π΅Π»ΠΊΠ½ΠΈΡ‚Π΅ ΠΏΡ€Π°Π²ΠΎΠΉ ΠΊΠ½ΠΎΠΏΠΊΠΎΠΉ ΠΌΡ‹ΡˆΠΈ Π»ΡŽΠ±ΡƒΡŽ ΠΌΠ΅Ρ‚ΠΊΡƒ Π΄Π°Π½Π½Ρ‹Ρ… ΠΈ Π²Ρ‹Π±Π΅Ρ€ΠΈΡ‚Π΅ Β« Π€ΠΎΡ€ΠΌΠ°Ρ‚ΠΈΡ€ΠΎΠ²Π°Ρ‚ΡŒ ΠΌΠ΅Ρ‚ΠΊΠΈ Π΄Π°Π½Π½Ρ‹Ρ…. Β»

    Когда откроСтся панСль Π·Π°Π΄Π°Ρ‡, Π·Π°ΠΌΠ΅Π½ΠΈΡ‚Π΅ значСния, Π²Ρ‹ΠΏΠΎΠ»Π½ΠΈΠ² ΡΠ»Π΅Π΄ΡƒΡŽΡ‰ΠΈΠ΅ дСйствия:

    1. ΠŸΠ΅Ρ€Π΅ΠΉΠ΄ΠΈΡ‚Π΅ Π½Π° Π²ΠΊΠ»Π°Π΄ΠΊΡƒ ΠžΠΏΡ†ΠΈΠΈ этикСтки .
    2. УстановитС Ρ„Π»Π°ΠΆΠΎΠΊ Β« Π—Π½Π°Ρ‡Π΅Π½ΠΈΠ΅ ΠΈΠ· ячССк Β».
    3. Π’Ρ‹Π΄Π΅Π»ΠΈΡ‚Π΅ значСния ΠΊΠ°Ρ‚Π΅Π³ΠΎΡ€ΠΈΠΈ ΠΈΠ· исходной Ρ‚Π°Π±Π»ΠΈΡ†Ρ‹ Π΄Π°Π½Π½Ρ‹Ρ… ( A3:A14 ).
    4. НаТмитС Β« ОК. Β»
    5. Π‘Π½ΠΈΠΌΠΈΡ‚Π΅ Ρ„Π»Π°ΠΆΠΎΠΊ Β« Π—Π½Π°Ρ‡Π΅Π½ΠΈΠ΅ Β».
    6. Π‘Π½ΠΈΠΌΠΈΡ‚Π΅ Ρ„Π»Π°ΠΆΠΎΠΊ Β« ΠŸΠΎΠΊΠ°Π·Π°Ρ‚ΡŒ Π»ΠΈΠ½ΠΈΠΈ выноски Β».

    Π¨Π°Π³ β„– 16. ΠŸΠ΅Ρ€Π΅ΠΌΠ΅ΡΡ‚ΠΈΡ‚Π΅ этикСтки.

    Π’Π΅ΠΏΠ΅Ρ€ΡŒ Π½Π΅ΠΌΠ½ΠΎΠ³ΠΎ ΡΠ΄Π²ΠΈΠ½ΡŒΡ‚Π΅ ΠΌΠ΅Ρ‚ΠΊΠΈ, размСстив ΠΈΡ… вдоль края внСшнСго ΠΊΠΎΠ»ΡŒΡ†Π° Π² порядкС, ΠΏΠΎΠΊΠ°Π·Π°Π½Π½ΠΎΠΌ Π½Π° ΡΠΊΡ€ΠΈΠ½ΡˆΠΎΡ‚Π΅ Π½ΠΈΠΆΠ΅. Π­Ρ‚ΠΎ Π½ΡƒΠΆΠ½ΠΎ Π±ΡƒΠ΄Π΅Ρ‚ ΡΠ΄Π΅Π»Π°Ρ‚ΡŒ Π²Ρ€ΡƒΡ‡Π½ΡƒΡŽ, ΠΏΠ΅Ρ€Π΅Ρ‚Π°Ρ‰ΠΈΠ² ΠΊΠ°ΠΆΠ΄Ρ‹ΠΉ Π·Π°Π³ΠΎΠ»ΠΎΠ²ΠΎΠΊ Π² Π½ΡƒΠΆΠ½ΠΎΠ΅ ΠΏΠΎΠ»ΠΎΠΆΠ΅Π½ΠΈΠ΅.

    НаконСц, ΠΈΠ·ΠΌΠ΅Π½ΠΈΡ‚Π΅ Π½Π°Π·Π²Π°Π½ΠΈΠ΅ Π΄ΠΈΠ°Π³Ρ€Π°ΠΌΠΌΡ‹, ΠΈ всС Π³ΠΎΡ‚ΠΎΠ²ΠΎ!

    Π‘ΠΊΠ°Ρ‡Π°Ρ‚ΡŒ шаблон полярного Π³Ρ€Π°Ρ„ΠΈΠΊΠ°

    Π—Π°Π³Ρ€ΡƒΠ·ΠΈΡ‚Π΅ наш бСсплатный шаблон полярного Π³Ρ€Π°Ρ„ΠΈΠΊΠ° для Excel.

    Π—Π°Π³Ρ€ΡƒΠ·ΠΈΡ‚ΡŒ сСйчас

    БСсплатная полярная ΠΌΠΈΠ»Π»ΠΈΠΌΠ΅Ρ‚Ρ€ΠΎΠ²ΠΊΠ° | Π‘ΡƒΠΌΠ°Π³Π° для ΠΏΠ΅Ρ‡Π°Ρ‚ΠΈ полярных ΠΊΠΎΠΎΡ€Π΄ΠΈΠ½Π°Ρ‚

    Π§Ρ‚ΠΎ Ρ‚Π°ΠΊΠΎΠ΅ полярная ΠΌΠΈΠ»Π»ΠΈΠΌΠ΅Ρ‚Ρ€ΠΎΠ²ΠΊΠ°?

    ΠŸΠΎΠ»ΡΡ€Π½Π°Ρ миллимСтровая Π±ΡƒΠΌΠ°Π³Π°, Ρ‚Π°ΠΊΠΆΠ΅ извСстная ΠΊΠ°ΠΊ полярная координатная Π±ΡƒΠΌΠ°Π³Π°, прСдставляСт собой ΠΌΠΈΠ»Π»ΠΈΠΌΠ΅Ρ‚Ρ€ΠΎΠ²ΡƒΡŽ Π±ΡƒΠΌΠ°Π³Ρƒ с концСнтричСскими ΠΊΡ€ΡƒΠ³Π°ΠΌΠΈ (с Ρ€Π°Π²Π½Ρ‹ΠΌ расстояниСм ΠΌΠ΅ΠΆΠ΄Ρƒ Π½ΠΈΠΌΠΈ), ΠΊΠΎΡ‚ΠΎΡ€Ρ‹Π΅ Ρ€Π°Π·Π΄Π΅Π»Π΅Π½Ρ‹ Π½Π° нСбольшиС Π΄ΡƒΠ³ΠΈ. ΠŸΡ€ΠΈ построСнии Π³Ρ€Π°Ρ„ΠΈΠΊΠ° Π² этой ΡΡ‚Π°Ρ‚ΡŒΠ΅ измСряСтся Ρ€Π°Π΄ΠΈΠ°Π»ΡŒΠ½ΠΎΠ΅ расстояниС ΠΎΡ‚ полюса (Π½Π°Ρ‡Π°Π»ΠΎ ΠΊΠΎΠΎΡ€Π΄ΠΈΠ½Π°Ρ‚), Π° Π·Π°Ρ‚Π΅ΠΌ выполняСтся ΠΏΠΎΠ²ΠΎΡ€ΠΎΡ‚ Π½Π° ΡƒΠ³ΠΎΠ» Ρ‚Π΅Ρ‚Π°-градусов ΠΎΡ‚ полярной оси (ΠΏΠΎΠ»ΠΎΠΆΠΈΡ‚Π΅Π»ΡŒΠ½Π°Ρ ось x).

    Π’ полярных ΠΊΠΎΠΎΡ€Π΄ΠΈΠ½Π°Ρ‚Π°Ρ… Ρ‚ΠΎΡ‡ΠΊΠ° (r, ?) описываСтся ΠΊΠ°ΠΊ находящаяся Π½Π° ΠΎΠΏΡ€Π΅Π΄Π΅Π»Π΅Π½Π½ΠΎΠΌ расстоянии (r) ΠΎΡ‚ полюса (Π½Π°Ρ‡Π°Π»ΠΎ ΠΊΠΎΠΎΡ€Π΄ΠΈΠ½Π°Ρ‚) ΠΈ ΠΏΠΎΠ΄ ΠΎΠΏΡ€Π΅Π΄Π΅Π»Π΅Π½Π½Ρ‹ΠΌ ΡƒΠ³Π»ΠΎΠΌ (?) ΠΎΡ‚ полярной оси (ΠΏΠΎΠ»ΠΎΠΆΠΈΡ‚Π΅Π»ΡŒΠ½Π°Ρ Π³ΠΎΡ€ΠΈΠ·ΠΎΠ½Ρ‚Π°Π»ΡŒΠ½Π°Ρ ось).

    ΠŸΡ€ΠΈΠΌΠ΅Ρ€ построСния

    На ΠΏΡ€ΠΈΠ²Π΅Π΄Π΅Π½Π½ΠΎΠΌ Π½ΠΈΠΆΠ΅ Π³Ρ€Π°Ρ„ΠΈΠΊΠ΅ полярных ΠΊΠΎΠΎΡ€Π΄ΠΈΠ½Π°Ρ‚ ΠΏΠΎΠΊΠ°Π·Π°Π½ΠΎ построСниС Ρ‚Ρ€Π΅Ρ… Ρ‚ΠΎΡ‡Π΅ΠΊ. ΠšΡ€Π°ΡΠ½Π°Ρ Ρ‚ΠΎΡ‡ΠΊΠ° ΠΈΠΌΠ΅Π΅Ρ‚ полярныС ΠΊΠΎΠΎΡ€Π΄ΠΈΠ½Π°Ρ‚Ρ‹ (6, 60 o ). ЗСлСная Ρ‚ΠΎΡ‡ΠΊΠ° ΠΈΠΌΠ΅Π΅Ρ‚ полярныС ΠΊΠΎΠΎΡ€Π΄ΠΈΠ½Π°Ρ‚Ρ‹ (2, 170 o ). И фиолСтовая Ρ‚ΠΎΡ‡ΠΊΠ° ΠΈΠΌΠ΅Π΅Ρ‚ полярныС ΠΊΠΎΠΎΡ€Π΄ΠΈΠ½Π°Ρ‚Ρ‹ (8, 260 o ).

    Π Π•ΠšΠ›ΠΠœΠ


    О нашСй Π±ΡƒΠΌΠ°Π³Π΅ с полярными ΠΊΠΎΠΎΡ€Π΄ΠΈΠ½Π°Ρ‚Π°ΠΌΠΈ

    Π£ нас Π΅ΡΡ‚ΡŒ Ρ‡Π΅Ρ‚Ρ‹Ρ€Π΅ стиля ΠΌΠΈΠ»Π»ΠΈΠΌΠ΅Ρ‚Ρ€ΠΎΠ²ΠΎΠΉ Π±ΡƒΠΌΠ°Π³ΠΈ с полярными ΠΊΠΎΠΎΡ€Π΄ΠΈΠ½Π°Ρ‚Π°ΠΌΠΈ…

    — ΡˆΠ΅ΡΡ‚ΠΈΠ΄ΡŽΠΉΠΌΠΎΠ²Π°Ρ Π΄ΠΈΠ°Π³Ρ€Π°ΠΌΠΌΠ°, Ρ‡Π΅Ρ€Π½Ρ‹Π΅ Ρ‡Π΅Ρ€Π½ΠΈΠ»Π°, ΠΎΠ΄Π½Π° Π΄ΠΈΠ°Π³Ρ€Π°ΠΌΠΌΠ°
    — Ρ‡Π΅Ρ‚Ρ‹Ρ€Π΅Ρ…Π΄ΡŽΠΉΠΌΠΎΠ²Π°Ρ Π΄ΠΈΠ°Π³Ρ€Π°ΠΌΠΌΠ°, Ρ‡Π΅Ρ€Π½Ρ‹Π΅ Ρ‡Π΅Ρ€Π½ΠΈΠ»Π°, Π΄Π²Π΅ Π΄ΠΈΠ°Π³Ρ€Π°ΠΌΠΌΡ‹
    — ΡˆΠ΅ΡΡ‚ΠΈΠ΄ΡŽΠΉΠΌΠΎΠ²Π°Ρ Π΄ΠΈΠ°Π³Ρ€Π°ΠΌΠΌΠ°, синиС Ρ‡Π΅Ρ€Π½ΠΈΠ»Π°, ΠΎΠ΄Π½Π° Π΄ΠΈΠ°Π³Ρ€Π°ΠΌΠΌΠ°
    — — Ρ‡Π΅Ρ‚Ρ‹Ρ€Π΅Ρ…Π΄ΡŽΠΉΠΌΠΎΠ²Π°Ρ Π΄ΠΈΠ°Π³Ρ€Π°ΠΌΠΌΠ°, синиС Ρ‡Π΅Ρ€Π½ΠΈΠ»Π°, Π΄Π²Π΅ Π΄ΠΈΠ°Π³Ρ€Π°ΠΌΠΌΡ‹

    Π’Ρ‹Π±Π΅Ρ€ΠΈΡ‚Π΅ Ρ€Π°Π·ΠΌΠ΅Ρ€ ΠΈ Ρ†Π²Π΅Ρ‚ Ρ‡Π΅Ρ€Π½ΠΈΠ», ΠΊΠΎΡ‚ΠΎΡ€Ρ‹Π΅ Π±ΡƒΠ΄ΡƒΡ‚ Ρ…ΠΎΡ€ΠΎΡˆΠΎ ΡΠΎΡ‡Π΅Ρ‚Π°Ρ‚ΡŒΡΡ с вашим Π³Ρ€Π°Ρ„ΠΈΠΊΠΎΠΌ.Π­Ρ‚ΠΈ Π΄ΠΈΠ°Π³Ρ€Π°ΠΌΠΌΡ‹ ΠΏΠ΅Ρ‡Π°Ρ‚Π°ΡŽΡ‚ΡΡ Π½Π° стандартном листС Π±ΡƒΠΌΠ°Π³ΠΈ Ρ€Π°Π·ΠΌΠ΅Ρ€ΠΎΠΌ 8 1/2 x 11. Π˜Ρ… ΠΌΠΎΠΆΠ½ΠΎ ΠΎΡ‚ΠΊΡ€Ρ‹Ρ‚ΡŒ ΠΈ Ρ€Π°ΡΠΏΠ΅Ρ‡Π°Ρ‚Π°Ρ‚ΡŒ с ΠΏΠΎΠΌΠΎΡ‰ΡŒΡŽ ΠΏΡ€ΠΎΠ³Ρ€Π°ΠΌΠΌΡ‹ чтСния Π΄ΠΎΠΊΡƒΠΌΠ΅Π½Ρ‚ΠΎΠ² .pdf, Π½Π°ΠΏΡ€ΠΈΠΌΠ΅Ρ€ Adobe Reader.

    ΠŸΠΎΠ΄Ρ€ΠΎΠ±Π½Π΅Π΅ ΠΎ Π±ΡƒΠΌΠ°Π³Π΅ с полярными ΠΊΠΎΠΎΡ€Π΄ΠΈΠ½Π°Ρ‚Π°ΠΌΠΈ

    [1] ΠœΠΈΠ»Π»ΠΈΠΌΠ΅Ρ‚Ρ€ΠΎΠ²Π°Ρ Π±ΡƒΠΌΠ°Π³Π° для бСсплатной ΠΏΠ΅Ρ‡Π°Ρ‚ΠΈ . Наша коллСкция ΠΈΠ· Π΄Π²Π°Π΄Ρ†Π°Ρ‚ΠΈ Ρ€Π°Π·Π»ΠΈΡ‡Π½Ρ‹Ρ… Π΄ΠΈΠ·Π°ΠΉΠ½ΠΎΠ² ΠΌΠΈΠ»Π»ΠΈΠΌΠ΅Ρ‚Ρ€ΠΎΠ²ΠΊΠΈ, ΠΊΠΎΡ‚ΠΎΡ€Ρ‹Π΅ Π²Ρ‹ ΠΌΠΎΠΆΠ΅Ρ‚Π΅ Ρ€Π°ΡΠΏΠ΅Ρ‡Π°Ρ‚Π°Ρ‚ΡŒ ΠΈ ΠΈΡΠΏΠΎΠ»ΡŒΠ·ΠΎΠ²Π°Ρ‚ΡŒ бСсплатно. ВодонСпроницаСмая Π±ΡƒΠΌΠ°Π³Π°.com.

    [2] ΠŸΠΎΠ»ΡΡ€Π½Π°Ρ систСма ΠΊΠΎΠΎΡ€Π΄ΠΈΠ½Π°Ρ‚, Бводная ΡΡ‚Π°Ρ‚ΡŒΡ ΠΎ полярной систСмС ΠΊΠΎΠΎΡ€Π΄ΠΈΠ½Π°Ρ‚. Π‘Ρ‚Π°Ρ‚ΡŒΡ Π² Π’ΠΈΠΊΠΈΠΏΠ΅Π΄ΠΈΠΈ, послСднСС ΠΎΠ±Ρ€Π°Ρ‰Π΅Π½ΠΈΠ΅ 05/2020.

    [3] ΠŸΡ€Π΅ΠΎΠ±Ρ€Π°Π·ΠΎΠ²Π°Ρ‚Π΅Π»ΡŒ ΠΊΠΎΠΎΡ€Π΄ΠΈΠ½Π°Ρ‚. ΠšΠ°Π»ΡŒΠΊΡƒΠ»ΡΡ‚ΠΎΡ€, ΠΊΠΎΡ‚ΠΎΡ€Ρ‹ΠΉ позволяСт Π²Ρ‹ΠΏΠΎΠ»Π½ΡΡ‚ΡŒ ΠΏΡ€Π΅ΠΎΠ±Ρ€Π°Π·ΠΎΠ²Π°Π½ΠΈΠ΅ ΠΌΠ΅ΠΆΠ΄Ρƒ Π΄Π΅ΠΊΠ°Ρ€Ρ‚ΠΎΠ²Ρ‹ΠΌΠΈ, полярными ΠΈ цилиндричСскими ΠΊΠΎΠΎΡ€Π΄ΠΈΠ½Π°Ρ‚Π°ΠΌΠΈ.Random-Science-Tools.com ΠΈΠ½Ρ‚Π΅Ρ€Π°ΠΊΡ‚ΠΈΠ²Π½Ρ‹ΠΉ, послСдний доступ 05/2020.

    [4] ΠŸΠΎΠ»ΡΡ€Π½Ρ‹Π΅ ΠΊΠΎΠΎΡ€Π΄ΠΈΠ½Π°Ρ‚Ρ‹. ΠšΠΎΡ€ΠΎΡ‚ΠΊΠ°Ρ ΡΡ‚Π°Ρ‚ΡŒΡ с ΠΏΡ€ΠΈΠΌΠ΅Ρ€Π°ΠΌΠΈ Π³Ρ€Π°Ρ„ΠΈΠΊΠΎΠ² ΠΈ Π·Π°Π΄Π°Ρ‡, Π΄Π΅ΠΌΠΎΠ½ΡΡ‚Ρ€ΠΈΡ€ΡƒΡŽΡ‰ΠΈΡ… ΠΏΠΎΠ»ΡΡ€Π½ΡƒΡŽ систСму ΠΊΠΎΠΎΡ€Π΄ΠΈΠ½Π°Ρ‚. Π‘Ρ‚Π°Ρ‚ΡŒΡ Π½Π° Π²Π΅Π±-сайтС Interactive Mathematics, послСднСС ΠΎΠ±Ρ€Π°Ρ‰Π΅Π½ΠΈΠ΅ 05/2020.

    ГрафичСскиС Ρ„ΡƒΠ½ΠΊΡ†ΠΈΠΈ Π² полярных ΠΊΠΎΠΎΡ€Π΄ΠΈΠ½Π°Ρ‚Π°Ρ…: процСсс ΠΈ ΠΏΡ€ΠΈΠΌΠ΅Ρ€Ρ‹ β€” Π²ΠΈΠ΄Π΅ΠΎ ΠΈ Ρ€Π°ΡΡˆΠΈΡ„Ρ€ΠΎΠ²ΠΊΠ° ΡƒΡ€ΠΎΠΊΠ°

    Π”Π΅ΠΊΠ°Ρ€Ρ‚ΠΎΠ²Ρ‹ ΠΊΠΎΠΎΡ€Π΄ΠΈΠ½Π°Ρ‚Ρ‹ Π² полярныС

    Π’Ρ‹ Π±Ρ‹Π»ΠΈ Π±Ρ‹ ΡƒΠ΄ΠΈΠ²Π»Π΅Π½Ρ‹, ΠΎΠ±Π½Π°Ρ€ΡƒΠΆΠΈΠ², Ρ‡Ρ‚ΠΎ эти Π΄Π²Π΅ систСмы фактичСски связаны Π΄Ρ€ΡƒΠ³ с Π΄Ρ€ΡƒΠ³ΠΎΠΌ матСматичСски? Π”Π°, ΠΎΠ½ΠΈ.И Π΄Π°, Ρƒ нас Π΅ΡΡ‚ΡŒ Ρ„ΠΎΡ€ΠΌΡƒΠ»Ρ‹, ΠΊΠΎΡ‚ΠΎΡ€Ρ‹Π΅ ΠΏΠΎΠΌΠΎΠ³ΡƒΡ‚ Π½Π°ΠΌ ΠΏΠ΅Ρ€Π΅ΠΉΡ‚ΠΈ ΠΎΡ‚ ΠΎΠ΄Π½ΠΎΠΉ систСмы ΠΊ Π΄Ρ€ΡƒΠ³ΠΎΠΉ. Но, Π²ΠΎ-ΠΏΠ΅Ρ€Π²Ρ‹Ρ…, ΠΊΠ°ΠΊ ΠΎΠ½ΠΈ связаны, спроситС Π²Ρ‹?

    Π˜Ρ‚Π°ΠΊ, посмотритС Π½Π° Π½Π°ΡˆΡƒ Π΄Π΅ΠΊΠ°Ρ€Ρ‚ΠΎΠ²Ρƒ Ρ‚ΠΎΡ‡ΠΊΡƒ ΠΊΠΎΠΎΡ€Π΄ΠΈΠ½Π°Ρ‚ (1, 20) Π½ΠΈΠΆΠ΅. Если ΠΌΡ‹ ΠΏΡ€ΠΎΠ²Π΅Π΄Π΅ΠΌ линию ΠΈΠ· этой Ρ‚ΠΎΡ‡ΠΊΠΈ Π² Π½Π°Ρ‡Π°Π»ΠΎ ΠΊΠΎΠΎΡ€Π΄ΠΈΠ½Π°Ρ‚, ΠΌΡ‹ ΠΏΠΎΠ»ΡƒΡ‡ΠΈΠΌ наш радиус. ΠœΡ‹ ΠΌΠΎΠΆΠ΅ΠΌ ΠΏΡ€ΠΎΠ΄ΠΎΠ»ΠΆΠΈΡ‚ΡŒ ΠΈ Π½Π°Ρ€ΠΈΡΠΎΠ²Π°Ρ‚ΡŒ наши Π²Π΅Ρ€Ρ‚ΠΈΠΊΠ°Π»ΡŒΠ½Ρ‹Π΅ ΠΈ Π³ΠΎΡ€ΠΈΠ·ΠΎΠ½Ρ‚Π°Π»ΡŒΠ½Ρ‹Π΅ Π»ΠΈΠ½ΠΈΠΈ, Ρ‡Ρ‚ΠΎΠ±Ρ‹ ΠΏΡ€Π΅Π΄ΡΡ‚Π°Π²ΠΈΡ‚ΡŒ наши значСния x ΠΈ y . Π£Π³ΠΎΠ», ΠΎΠ±Ρ€Π°Π·ΠΎΠ²Π°Π½Π½Ρ‹ΠΉ Π»ΠΈΠ½ΠΈΠ΅ΠΉ x ΠΈ Π»ΠΈΠ½ΠΈΠ΅ΠΉ r , ΠΊΠΎΡ‚ΠΎΡ€ΡƒΡŽ ΠΌΡ‹ нарисовали, Ρ‚Π΅ΠΏΠ΅Ρ€ΡŒ являСтся нашСй Ρ‚Π΅Ρ‚Π°. Π’Π΅ΠΏΠ΅Ρ€ΡŒ посмотритС, какая Ρ„ΠΎΡ€ΠΌΠ° Ρƒ нас Π½ΠΈΠΆΠ΅.-1 я Π˜ΡΠΏΠΎΠ»ΡŒΠ·ΠΎΠ²Π°Ρ‚ΡŒ Π·Π½Π°Ρ‡Π΅Π½ΠΈΠ΅ ΠΊΠ°Π»ΡŒΠΊΡƒΠ»ΡΡ‚ΠΎΡ€Π° II Π”ΠΎΠ±Π°Π²ΡŒΡ‚Π΅ 180 градусов ΠΊ Π·Π½Π°Ρ‡Π΅Π½ΠΈΡŽ ΠΊΠ°Π»ΡŒΠΊΡƒΠ»ΡΡ‚ΠΎΡ€Π° III Π”ΠΎΠ±Π°Π²ΡŒΡ‚Π΅ 180 градусов ΠΊ Π·Π½Π°Ρ‡Π΅Π½ΠΈΡŽ ΠΊΠ°Π»ΡŒΠΊΡƒΠ»ΡΡ‚ΠΎΡ€Π° IV Π”ΠΎΠ±Π°Π²ΠΈΡ‚ΡŒ 360 градусов ΠΊ Π·Π½Π°Ρ‡Π΅Π½ΠΈΡŽ ΠΊΠ°Π»ΡŒΠΊΡƒΠ»ΡΡ‚ΠΎΡ€Π°

    Π“Ρ€Π°Ρ„ΠΈΠΊΠΈ полярных ΡƒΡ€Π°Π²Π½Π΅Π½ΠΈΠΉ

    Π“Ρ€Π°Ρ„ΠΈΠΊΠΈ полярных ΡƒΡ€Π°Π²Π½Π΅Π½ΠΈΠΉ Ρ‚Π°ΠΊΠΆΠ΅ Π½Π΅ΠΌΠ½ΠΎΠ³ΠΎ ΠΎΡ‚Π»ΠΈΡ‡Π°ΡŽΡ‚ΡΡ ΠΎΡ‚ Π³Ρ€Π°Ρ„ΠΈΠΊΠΎΠ² Π΄Π΅ΠΊΠ°Ρ€Ρ‚ΠΎΠ²Ρ‹Ρ… ΡƒΡ€Π°Π²Π½Π΅Π½ΠΈΠΉ. Для Π΄Π΅ΠΊΠ°Ρ€Ρ‚ΠΎΠ²Ρ‹Ρ… ΡƒΡ€Π°Π²Π½Π΅Π½ΠΈΠΉ Ρƒ нас Π΅ΡΡ‚ΡŒ Ρ‚ΠΎ, Ρ‡Ρ‚ΠΎ ΠΌΡ‹ Π½Π°Π·Ρ‹Π²Π°Π΅ΠΌ Ρ„ΠΎΡ€ΠΌΠΎΠΉ пСрСсСчСния Π½Π°ΠΊΠ»ΠΎΠ½Π°, y = m x + b , Π³Π΄Π΅ ΠΌΡ‹ смотрим Π½Π° наш y -ΠΎΡ‚Ρ€Π΅Π·ΠΎΠΊ, Π·Π½Π°Ρ‡Π΅Π½ΠΈΠ΅ b , Π° Π·Π°Ρ‚Π΅ΠΌ Π½Π° наш Π½Π°ΠΊΠ»ΠΎΠ½, ΠΌ , Ρ‡Ρ‚ΠΎΠ±Ρ‹ ΡƒΠ·Π½Π°Ρ‚ΡŒ, насколько ΠΊΡ€ΡƒΡ‚Π° наша линия.ΠŸΠΎΠ»ΡΡ€Π½Ρ‹Π΅ уравнСния Π½Π΅ Ρ‚Π°ΠΊ просты. Π›ΡƒΡ‡ΡˆΠΈΠΉ способ графичСского отобраТСния полярных ΡƒΡ€Π°Π²Π½Π΅Π½ΠΈΠΉ β€” ΡΠΎΠ·Π΄Π°Ρ‚ΡŒ Ρ‚Π°Π±Π»ΠΈΡ†Ρƒ Π·Π½Π°Ρ‡Π΅Π½ΠΈΠΉ.

    ΠšΡ€ΠΎΠΌΠ΅ Ρ‚ΠΎΠ³ΠΎ, полярныС уравнСния выглядят Π½Π΅ΠΌΠ½ΠΎΠ³ΠΎ ΠΈΠ½Π°Ρ‡Π΅, Ρ‡Π΅ΠΌ Π΄Π΅ΠΊΠ°Ρ€Ρ‚ΠΎΠ²Ρ‹ уравнСния. ΠŸΠΎΠ»ΡΡ€Π½Ρ‹Π΅ уравнСния часто содСрТат Π² сСбС тригономСтричСскиС Ρ„ΡƒΠ½ΠΊΡ†ΠΈΠΈ. НапримСр, Π²Ρ‹ ΡƒΠ²ΠΈΠ΄ΠΈΡ‚Π΅ Ρ‚Π°ΠΊΠΈΠ΅ уравнСния, ΠΊΠ°ΠΊ r = 2 cos (Ρ‚Π΅Ρ‚Π°) ΠΈΠ»ΠΈ r = 2 sin (Ρ‚Π΅Ρ‚Π°). Π”Ρ€ΡƒΠ³ΠΈΠ΅ ΠΏΡ€ΠΈΠΌΠ΅Ρ€Ρ‹ полярных ΡƒΡ€Π°Π²Π½Π΅Π½ΠΈΠΉ Π²ΠΊΠ»ΡŽΡ‡Π°ΡŽΡ‚ r = 2 ΠΈ r = 3 + 2 cos (Ρ‚Π΅Ρ‚Π°).

    Π”Π°Π²Π°ΠΉΡ‚Π΅ построим Π³Ρ€Π°Ρ„ΠΈΠΊ r = 2 cos (Ρ‚Π΅Ρ‚Π°), Ρ‡Ρ‚ΠΎΠ±Ρ‹ ΠΏΠΎΡΠΌΠΎΡ‚Ρ€Π΅Ρ‚ΡŒ, Ρ‡Ρ‚ΠΎ Ρƒ нас получится.ΠœΡ‹ составим сСбС Ρ‚Π°Π±Π»ΠΈΡ†Ρƒ Π·Π½Π°Ρ‡Π΅Π½ΠΈΠΉ r ΠΈ Ρ‚Π΅Ρ‚Π°. ΠœΡ‹ Π²Ρ‹Π±ΠΈΡ€Π°Π΅ΠΌ Ρ€Π°Π·Π»ΠΈΡ‡Π½Ρ‹Π΅ ΡƒΠ³Π»Ρ‹, Π° Π·Π°Ρ‚Π΅ΠΌ вычисляСм, Ρ‡Ρ‚ΠΎ ΠΌΡ‹ ΠΏΠΎΠ»ΡƒΡ‡Π°Π΅ΠΌ для r . Π—Π°Ρ‚Π΅ΠΌ ΠΌΡ‹ наносим эти полярныС ΠΊΠΎΠΎΡ€Π΄ΠΈΠ½Π°Ρ‚Ρ‹, Ρ‡Ρ‚ΠΎΠ±Ρ‹ ΡƒΠ²ΠΈΠ΄Π΅Ρ‚ΡŒ, какая Ρ„ΠΎΡ€ΠΌΠ° развиваСтся. ΠŸΠΎΠ»ΡƒΡ‡Π°Π΅ΠΌ Π²ΠΎΡ‚ Ρ‚Π°ΠΊΡƒΡŽ ​​таблицу Π·Π½Π°Ρ‡Π΅Π½ΠΈΠΉ:

    r Ρ‚Π΅Ρ‚Π° (градусов)
    2 0
    1,73 30
    1 60
    0 90
    -1 120
    -1.73 150
    -2 180
    -1,73 210
    -1 240
    0 270
    1 300
    1,73 330

    ΠŸΠΎΡΡ‚Ρ€ΠΎΠΈΠ² ΠΈΡ… Π² нашСй полярной систСмС ΠΊΠΎΠΎΡ€Π΄ΠΈΠ½Π°Ρ‚, Π° Π·Π°Ρ‚Π΅ΠΌ соСдинив Ρ‚ΠΎΡ‡ΠΊΠΈ, ΠΌΡ‹ ΠΏΠΎΠ»ΡƒΡ‡ΠΈΠΌ ΠΎΠΊΡ€ΡƒΠΆΠ½ΠΎΡΡ‚ΡŒ радиусом 1, ΠΊΠ°ΡΠ°ΡŽΡ‰ΡƒΡŽΡΡ оси y справа.

    Π“Ρ€Π°Ρ„ для ΠΏΡ€ΠΈΠΌΠ΅Ρ€Π° ΠΏΡ€ΠΎΠ±Π»Π΅ΠΌΡ‹

    Π˜Ρ‚ΠΎΠ³ΠΈ ΡƒΡ€ΠΎΠΊΠ°

    Π”Π°Π²Π°ΠΉΡ‚Π΅ ΠΏΠΎΠ²Ρ‚ΠΎΡ€ΠΈΠΌ, Ρ‡Ρ‚ΠΎ ΠΌΡ‹ ΡƒΠ·Π½Π°Π»ΠΈ:

    ΠœΡ‹ ΡƒΠ·Π½Π°Π»ΠΈ, Ρ‡Ρ‚ΠΎ систСма полярных ΠΊΠΎΠΎΡ€Π΄ΠΈΠ½Π°Ρ‚ β€” это Π΅Ρ‰Π΅ ΠΎΠ΄ΠΈΠ½ способ нанСсСния Ρ‚ΠΎΡ‡Π΅ΠΊ.-1 ( y / x ).

    Π Π΅Π·ΡƒΠ»ΡŒΡ‚Π°Ρ‚Ρ‹ обучСния

    ПослС этого ΡƒΡ€ΠΎΠΊΠ° Π²Ρ‹ смоТСтС:

    • ΠžΡ‚Π»ΠΈΡ‡Π°Ρ‚ΡŒ ΠΏΠΎΠ»ΡΡ€Π½ΡƒΡŽ систСму ΠΊΠΎΠΎΡ€Π΄ΠΈΠ½Π°Ρ‚ ΠΎΡ‚ Π΄Π΅ΠΊΠ°Ρ€Ρ‚ΠΎΠ²ΠΎΠΉ систСмы
    • ΠŸΡ€Π΅ΠΎΠ±Ρ€Π°Π·ΠΎΠ²Π°Π½ΠΈΠ΅ Π΄Π΅ΠΊΠ°Ρ€Ρ‚ΠΎΠ²Ρ‹Ρ… Ρ‚ΠΎΡ‡Π΅ΠΊ Π² полярныС Ρ‚ΠΎΡ‡ΠΊΠΈ
    • Π“Ρ€Π°Ρ„ΠΈΠΊ полярного уравнСния

    БфСричСскиС ΠΊΠΎΠΎΡ€Π΄ΠΈΠ½Π°Ρ‚Ρ‹ — Math Insight

    ΠŸΠΎΠ½Π°Ρ‡Π°Π»Ρƒ сфСричСскиС ΠΊΠΎΠΎΡ€Π΄ΠΈΠ½Π°Ρ‚Ρ‹ ΠΌΠΎΠ³ΡƒΡ‚ Π±Ρ‹Ρ‚ΡŒ Π½Π΅ΠΌΠ½ΠΎΠ³ΠΎ слоТными для понимания. БфСричСскиС ΠΊΠΎΠΎΡ€Π΄ΠΈΠ½Π°Ρ‚Ρ‹ ΠΎΠΏΡ€Π΅Π΄Π΅Π»ΡΡŽΡ‚ ΠΏΠΎΠ»ΠΎΠΆΠ΅Π½ΠΈΠ΅ Ρ‚ΠΎΡ‡ΠΊΠΈ Π² Ρ‚Ρ€Π΅Ρ…ΠΌΠ΅Ρ€Π½ΠΎΠΌ пространствС Π½Π° основС расстояния $\rho$ ΠΎΡ‚ Π½Π°Ρ‡Π°Π»Π° ΠΊΠΎΠΎΡ€Π΄ΠΈΠ½Π°Ρ‚ ΠΈ Π΄Π²ΡƒΡ… ΡƒΠ³Π»ΠΎΠ² $\theta$ ΠΈ $\phi$.Если ΠΊΡ‚ΠΎ-Ρ‚ΠΎ Π·Π½Π°ΠΊΠΎΠΌ с полярными ΠΊΠΎΠΎΡ€Π΄ΠΈΠ½Π°Ρ‚Π°ΠΌΠΈ, Ρ‚ΠΎ ΡƒΠ³ΠΎΠ» $\theta$ ΠΏΠΎΠ½ΡΡ‚ΡŒ нСслоТно, Ρ‚Π°ΠΊ ΠΊΠ°ΠΊ ΠΎΠ½ практичСски Ρ‚Π°ΠΊΠΎΠΉ ΠΆΠ΅, ΠΊΠ°ΠΊ ΡƒΠ³ΠΎΠ» $\theta$ Π² полярных ΠΊΠΎΠΎΡ€Π΄ΠΈΠ½Π°Ρ‚Π°Ρ…. Но Π½Π΅ΠΊΠΎΡ‚ΠΎΡ€Ρ‹ΠΌ людям Ρ‚Ρ€ΡƒΠ΄Π½ΠΎ ΠΏΠΎΠ½ΡΡ‚ΡŒ, Ρ‡Ρ‚ΠΎ ΡƒΠ³ΠΎΠ» $\phi$ это всС ΠΎ.

    Π‘Π»Π΅Π΄ΡƒΡŽΡ‰Π°Ρ Π³Ρ€Π°Ρ„ΠΈΠΊΠ° ΠΈ ΠΈΠ½Ρ‚Π΅Ρ€Π°ΠΊΡ‚ΠΈΠ²Π½Ρ‹Π΅ Π°ΠΏΠΏΠ»Π΅Ρ‚Ρ‹ ΠΌΠΎΠ³ΡƒΡ‚ ΠΏΠΎΠΌΠΎΡ‡ΡŒ Π²Π°ΠΌ ΠΏΠΎΠ½ΡΡ‚ΡŒ сфСричСскиС Π»ΡƒΡ‡ΡˆΠ΅ ΠΊΠΎΠΎΡ€Π΄ΠΈΠ½ΠΈΡ€ΡƒΠ΅Ρ‚. На этой страницС ΠΌΡ‹ Π²Ρ‹Π²ΠΎΠ΄ΠΈΠΌ взаимосвязь ΠΌΠ΅ΠΆΠ΄Ρƒ сфСричСскими ΠΈ Π΄Π΅ΠΊΠ°Ρ€Ρ‚ΠΎΠ²Ρ‹ΠΌΠΈ ΠΊΠΎΠΎΡ€Π΄ΠΈΠ½Π°Ρ‚Π°ΠΌΠΈ, ΠΏΠΎΠΊΠ°Π·Ρ‹Π²Π°Π΅ΠΌ Π°ΠΏΠΏΠ»Π΅Ρ‚, ΠΊΠΎΡ‚ΠΎΡ€Ρ‹ΠΉ позволяСт ΠΈΡΡΠ»Π΅Π΄ΠΎΠ²Π°Ρ‚ΡŒ влияниС ΠΊΠ°ΠΆΠ΄ΠΎΠΉ сфСричСской ΠΊΠΎΠΎΡ€Π΄ΠΈΠ½Π°Ρ‚Ρ‹, ΠΈ ΠΈΠ»Π»ΡŽΡΡ‚Ρ€ΠΈΡ€ΡƒΠ΅ΠΌ простыС сфСричСскиС ΠΊΠΎΠΎΡ€Π΄ΠΈΠ½Π°Ρ‚Π½Ρ‹Π΅ повСрхности.

    Бвязь ΠΌΠ΅ΠΆΠ΄Ρƒ сфСричСскими ΠΈ Π΄Π΅ΠΊΠ°Ρ€Ρ‚ΠΎΠ²Ρ‹ΠΌΠΈ ΠΊΠΎΠΎΡ€Π΄ΠΈΠ½Π°Ρ‚Π°ΠΌΠΈ

    БфСричСскиС ΠΊΠΎΠΎΡ€Π΄ΠΈΠ½Π°Ρ‚Ρ‹ ΠΎΠΏΡ€Π΅Π΄Π΅Π»ΡΡŽΡ‚ΡΡ Ρ‚Π°ΠΊ, ΠΊΠ°ΠΊ ΡƒΠΊΠ°Π·Π°Π½ΠΎ Π² ΡΠ»Π΅Π΄ΡƒΡŽΡ‰ΠΈΠΉ рисунок, ΠΊΠΎΡ‚ΠΎΡ€Ρ‹ΠΉ ΠΈΠ»Π»ΡŽΡΡ‚Ρ€ΠΈΡ€ΡƒΠ΅Ρ‚ сфСричСскиС ΠΊΠΎΠΎΡ€Π΄ΠΈΠ½Π°Ρ‚Ρ‹ Ρ‚ΠΎΡ‡ΠΊΠ° $P$.

    ΠšΠΎΠΎΡ€Π΄ΠΈΠ½Π°Ρ‚Π° $\rho$ β€” это расстояниС ΠΎΡ‚ $P$ Π΄ΠΎ Π½Π°Ρ‡Π°Π»Π° ΠΊΠΎΠΎΡ€Π΄ΠΈΠ½Π°Ρ‚. Если Ρ‚ΠΎΡ‡ΠΊΠ° $Q$ являСтся ΠΏΡ€ΠΎΠ΅ΠΊΡ†ΠΈΠ΅ΠΉ $P$ Π½Π° ΠΏΠ»ΠΎΡΠΊΠΎΡΡ‚ΡŒ $xy$, Ρ‚ΠΎΠ³Π΄Π° $\theta$ ΡƒΠ³ΠΎΠ» ΠΌΠ΅ΠΆΠ΄Ρƒ ΠΏΠΎΠ»ΠΎΠΆΠΈΡ‚Π΅Π»ΡŒΠ½ΠΎΠΉ осью $x$ ΠΈ ΠΎΡ‚Ρ€Π΅Π·ΠΊΠΎΠΌ ΠΎΡ‚ Π½Π°Ρ‡Π°Π»Π° ΠΊΠΎΠΎΡ€Π΄ΠΈΠ½Π°Ρ‚ Π΄ΠΎ $Q$.НаконСц, $\phi$ β€” это ΡƒΠ³ΠΎΠ» ΠΌΠ΅ΠΆΠ΄Ρƒ ΠΏΠΎΠ»ΠΎΠΆΠΈΡ‚Π΅Π»ΡŒΠ½ΠΎΠΉ осью $z$ ΠΈ ΠΎΡ‚Ρ€Π΅Π·ΠΎΠΊ прямой ΠΎΡ‚ Π½Π°Ρ‡Π°Π»Π° ΠΊΠΎΠΎΡ€Π΄ΠΈΠ½Π°Ρ‚ Π΄ΠΎ $P$.

    Π‘ ΠΏΠΎΠΌΠΎΡ‰ΡŒΡŽ Ρ‚Ρ€ΠΈΠ³ΠΎΠ½ΠΎΠΌΠ΅Ρ‚Ρ€ΠΈΠΈ ΠΌΠΎΠΆΠ½ΠΎ Π²Ρ‹Ρ‡ΠΈΡΠ»ΠΈΡ‚ΡŒ связь ΠΌΠ΅ΠΆΠ΄Ρƒ Π΄Π΅ΠΊΠ°Ρ€Ρ‚ΠΎΠ²Ρ‹ΠΌΠΈ ΠΊΠΎΠΎΡ€Π΄ΠΈΠ½Π°Ρ‚Π°ΠΌΠΈ $(x,y,z)$ Ρ‚ΠΎΡ‡ΠΊΠΈ $P$ ΠΈ Π΅Π΅ сфСричСскими ΠΊΠΎΠΎΡ€Π΄ΠΈΠ½Π°Ρ‚Π°ΠΌΠΈ $(\rho,\theta,\phi)$. Π ΠΎΠ·ΠΎΠ²Ρ‹ΠΉ Ρ‚Ρ€Π΅ΡƒΠ³ΠΎΠ»ΡŒΠ½ΠΈΠΊ Π²Π²Π΅Ρ€Ρ…Ρƒ β€” это ΠΏΡ€ΡΠΌΠΎΡƒΠ³ΠΎΠ»ΡŒΠ½Ρ‹ΠΉ Ρ‚Ρ€Π΅ΡƒΠ³ΠΎΠ»ΡŒΠ½ΠΈΠΊ, Π²Π΅Ρ€ΡˆΠΈΠ½Π°ΠΌΠΈ ΠΊΠΎΡ‚ΠΎΡ€ΠΎΠ³ΠΎ ΡΠ²Π»ΡΡŽΡ‚ΡΡ Π½Π°Ρ‡Π°Π»ΠΎ ΠΊΠΎΠΎΡ€Π΄ΠΈΠ½Π°Ρ‚, Ρ‚ΠΎΡ‡ΠΊΠ° $P$ ΠΈ Π΅Π΅ проСкция Π½Π° ось $z$. ΠŸΠΎΡΠΊΠΎΠ»ΡŒΠΊΡƒ Π΄Π»ΠΈΠ½Π° Π³ΠΈΠΏΠΎΡ‚Π΅Π½ΡƒΠ·Ρ‹ Ρ€Π°Π²Π½Π° $\rho$, Π° $\phi$Β β€” это ΡƒΠ³ΠΎΠ», ΠΊΠΎΡ‚ΠΎΡ€Ρ‹ΠΉ Π³ΠΈΠΏΠΎΡ‚Π΅Π½ΡƒΠ·Π° ΠΎΠ±Ρ€Π°Π·ΡƒΠ΅Ρ‚ с ΠΊΠ°Ρ‚Π΅Ρ‚ΠΎΠΌ ΠΏΡ€ΡΠΌΠΎΡƒΠ³ΠΎΠ»ΡŒΠ½ΠΎΠ³ΠΎ Ρ‚Ρ€Π΅ΡƒΠ³ΠΎΠ»ΡŒΠ½ΠΈΠΊΠ° ΠΏΠΎ оси $z$, ΠΊΠΎΠΎΡ€Π΄ΠΈΠ½Π°Ρ‚Π° $z$ Ρ‚ΠΎΡ‡ΠΊΠΈ $P$ (Ρ‚.Π΅., высота Ρ‚Ρ€Π΅ΡƒΠ³ΠΎΠ»ΡŒΠ½ΠΈΠΊΠ°) Ρ€Π°Π²Π½Π° $z=\rho\cos\phi$. Π”Π»ΠΈΠ½Π° Π΄Ρ€ΡƒΠ³ΠΎΠ³ΠΎ ΠΊΠ°Ρ‚Π΅Ρ‚Π° ΠΏΡ€ΡΠΌΠΎΡƒΠ³ΠΎΠ»ΡŒΠ½ΠΎΠ³ΠΎ Ρ‚Ρ€Π΅ΡƒΠ³ΠΎΠ»ΡŒΠ½ΠΈΠΊΠ° β€” это расстояниС ΠΎΡ‚ $P$ Π΄ΠΎ оси $z$, Ρ€Π°Π²Π½ΠΎΠ΅ $r=\rho\sin\phi$. Π‘Ρ‚ΠΎΠ»ΡŒΠΊΠΎ ΠΆΠ΅ находится ΠΈ расстояниС Ρ‚ΠΎΡ‡ΠΊΠΈ $Q$ ΠΎΡ‚ Π½Π°Ρ‡Π°Π»Π° ΠΊΠΎΠΎΡ€Π΄ΠΈΠ½Π°Ρ‚.

    Π“ΠΎΠ»ΡƒΠ±ΠΎΠΉ Ρ‚Ρ€Π΅ΡƒΠ³ΠΎΠ»ΡŒΠ½ΠΈΠΊ, ΠΏΠΎΠΊΠ°Π·Π°Π½Π½Ρ‹ΠΉ ΠΊΠ°ΠΊ Π² исходной Ρ‚Ρ€Π΅Ρ…ΠΌΠ΅Ρ€Π½ΠΎΠΉ систСмС ΠΊΠΎΠΎΡ€Π΄ΠΈΠ½Π°Ρ‚ слСва, Ρ‚Π°ΠΊ ΠΈ Π² плоскости $xy$ справа, прСдставляСт собой ΠΏΡ€ΡΠΌΠΎΡƒΠ³ΠΎΠ»ΡŒΠ½Ρ‹ΠΉ Ρ‚Ρ€Π΅ΡƒΠ³ΠΎΠ»ΡŒΠ½ΠΈΠΊ, Π²Π΅Ρ€ΡˆΠΈΠ½Π°ΠΌΠΈ ΠΊΠΎΡ‚ΠΎΡ€ΠΎΠ³ΠΎ ΡΠ²Π»ΡΡŽΡ‚ΡΡ Π½Π°Ρ‡Π°Π»ΠΎ ΠΊΠΎΠΎΡ€Π΄ΠΈΠ½Π°Ρ‚, Ρ‚ΠΎΡ‡ΠΊΠ° $Q$ ΠΈ Π΅Π΅ проСкция Π½Π° $ ось x$. На ΠΏΡ€Π°Π²ΠΎΠΌ Π³Ρ€Π°Ρ„ΠΈΠΊΠ΅ расстояниС ΠΎΡ‚ $Q$ Π΄ΠΎ Π½Π°Ρ‡Π°Π»Π° ΠΊΠΎΠΎΡ€Π΄ΠΈΠ½Π°Ρ‚, ΠΊΠΎΡ‚ΠΎΡ€ΠΎΠ΅ являСтся Π΄Π»ΠΈΠ½ΠΎΠΉ Π³ΠΈΠΏΠΎΡ‚Π΅Π½ΡƒΠ·Ρ‹ ΠΏΡ€ΡΠΌΠΎΡƒΠ³ΠΎΠ»ΡŒΠ½ΠΎΠ³ΠΎ Ρ‚Ρ€Π΅ΡƒΠ³ΠΎΠ»ΡŒΠ½ΠΈΠΊΠ°, ΠΎΠ±ΠΎΠ·Π½Π°Ρ‡Π΅Π½ΠΎ ΠΊΠ°ΠΊ $r$.ΠŸΠΎΡΠΊΠΎΠ»ΡŒΠΊΡƒ $\theta$ β€” это ΡƒΠ³ΠΎΠ», ΠΊΠΎΡ‚ΠΎΡ€Ρ‹ΠΉ эта Π³ΠΈΠΏΠΎΡ‚Π΅Π½ΡƒΠ·Π° ΠΎΠ±Ρ€Π°Π·ΡƒΠ΅Ρ‚ с осью $x$, $x$- ΠΈ $y$-ΠΊΠΎΠΌΠΏΠΎΠ½Π΅Π½Ρ‚Ρ‹ Ρ‚ΠΎΡ‡ΠΊΠΈ $Q$ (ΠΊΠΎΡ‚ΠΎΡ€Ρ‹Π΅ ΡΠΎΠ²ΠΏΠ°Π΄Π°ΡŽΡ‚ с $x$- ΠΈ $y$-ΠΊΠΎΠΌΠΏΠΎΠ½Π΅Π½Ρ‚Π°ΠΌΠΈ Ρ‚ΠΎΡ‡ΠΊΠΈ $Q$ $-ΠΊΠΎΠΌΠΏΠΎΠ½Π΅Π½Ρ‚Ρ‹ Ρ‚ΠΎΡ‡ΠΊΠΈ $P$) Π·Π°Π΄Π°ΡŽΡ‚ΡΡ выраТСниями $x=r\cos\theta$ ΠΈ $y=r\sin\theta$. ΠŸΠΎΡΠΊΠΎΠ»ΡŒΠΊΡƒ $r=\rho\sin\phi$, эти ΠΊΠΎΠΌΠΏΠΎΠ½Π΅Π½Ρ‚Ρ‹ ΠΌΠΎΠΆΠ½ΠΎ ΠΏΠ΅Ρ€Π΅ΠΏΠΈΡΠ°Ρ‚ΡŒ ΠΊΠ°ΠΊ $x=\rho\sin\phi\cos\theta$ ΠΈ $y=\rho\sin\phi\sin\theta$. Π’Π°ΠΊΠΈΠΌ ΠΎΠ±Ρ€Π°Π·ΠΎΠΌ, Ρ„ΠΎΡ€ΠΌΡƒΠ»Ρ‹ для Π΄Π΅ΠΊΠ°Ρ€Ρ‚ΠΎΠ²Ρ‹Ρ… ΠΊΠΎΠΎΡ€Π΄ΠΈΠ½Π°Ρ‚ Π² Ρ‚Π΅Ρ€ΠΌΠΈΠ½Π°Ρ… сфСричСских ΠΊΠΎΠΎΡ€Π΄ΠΈΠ½Π°Ρ‚: \Π½Π°Ρ‡Π°Ρ‚ΡŒ{Π²Ρ‹Ρ€Π°Π²Π½ΠΈΠ²Π°Ρ‚ΡŒ} x &= \rho\sin\phi\cos\theta\notag\\ y &= \rho\sin\phi\sin\theta\label{spherical_cartesian}\tag{1}\\ z &= \rho\cos\phi\notag.\end{Π²Ρ‹Ρ€Π°Π²Π½ΠΈΠ²Π°Π½ΠΈΠ΅}

    Π˜Π·ΡƒΡ‡Π΅Π½ΠΈΠ΅ влияния ΠΊΠ°ΠΆΠ΄ΠΎΠΉ сфСричСской ΠΊΠΎΠΎΡ€Π΄ΠΈΠ½Π°Ρ‚Ρ‹

    ΠŸΡ€ΠΈΠ²Π΅Π΄Π΅Π½Π½Ρ‹ΠΉ Π½ΠΈΠΆΠ΅ Π°ΠΏΠΏΠ»Π΅Ρ‚ позволяСт ΡƒΠ²ΠΈΠ΄Π΅Ρ‚ΡŒ, ΠΊΠ°ΠΊ мСняСтся ΠΏΠΎΠ»ΠΎΠΆΠ΅Π½ΠΈΠ΅ Ρ‚ΠΎΡ‡ΠΊΠΈ ΠΏΡ€ΠΈ Π²Ρ‹ мСняСтС $\rho$, $\theta$ ΠΈ $\phi$. Π’ΠΎΡ‡ΠΊΠ° $P$, ΡΠΎΠΎΡ‚Π²Π΅Ρ‚ΡΡ‚Π²ΡƒΡŽΡ‰Π°Ρ Π·Π½Π°Ρ‡Π΅Π½ΠΈΡŽ ΠΊΠΎΠΎΡ€Π΄ΠΈΠ½Π°Ρ‚, ΠΏΠΎΠΊΠ°Π·Π°Π½Π° большой Ρ„ΠΈΠΎΠ»Π΅Ρ‚ΠΎΠ²ΠΎΠΉ Ρ‚ΠΎΡ‡ΠΊΠΎΠΉ. ЗСлСная Ρ‚ΠΎΡ‡ΠΊΠ° β€” это Ρ‚ΠΎΡ‡ΠΊΠ° $Q$, Ρ‚. Π΅. проСкция $P$ Π½Π° $xy$-ΠΏΠ»ΠΎΡΠΊΠΎΡΡ‚ΡŒ.

    Π—Π°Π³Ρ€ΡƒΠ·ΠΊΠ° Π°ΠΏΠΏΠ»Π΅Ρ‚Π°

    БфСричСскиС ΠΊΠΎΠΎΡ€Π΄ΠΈΠ½Π°Ρ‚Ρ‹. Учитывая значСния сфСричСских ΠΊΠΎΠΎΡ€Π΄ΠΈΠ½Π°Ρ‚ $\rho$, $\theta$ ΠΈ $\phi$, ΠΊΠΎΡ‚ΠΎΡ€Ρ‹Π΅ ΠΌΠΎΠΆΠ½ΠΎ ΠΈΠ·ΠΌΠ΅Π½ΠΈΡ‚ΡŒ, пСрСтаскивая Ρ‚ΠΎΡ‡ΠΊΠΈ Π½Π° ΠΏΠΎΠ»Π·ΡƒΠ½ΠΊΠ°Ρ…, большая красная Ρ‚ΠΎΡ‡ΠΊΠ° ΠΏΠΎΠΊΠ°Π·Ρ‹Π²Π°Π΅Ρ‚ ΡΠΎΠΎΡ‚Π²Π΅Ρ‚ΡΡ‚Π²ΡƒΡŽΡ‰Π΅Π΅ ΠΏΠΎΠ»ΠΎΠΆΠ΅Π½ΠΈΠ΅ Π² Π”Π΅ΠΊΠ°Ρ€Ρ‚ΠΎΠ²Ρ‹ ΠΊΠΎΠΎΡ€Π΄ΠΈΠ½Π°Ρ‚Ρ‹.ЗСлСная Ρ‚ΠΎΡ‡ΠΊΠ° β€” это проСкция Ρ‚ΠΎΡ‡ΠΊΠΈ Π½Π° $xy$-ΠΏΠ»ΠΎΡΠΊΠΎΡΡ‚ΡŒ. Π’Ρ‹ ΠΌΠΎΠΆΠ΅Ρ‚Π΅ Π²ΠΈΠ·ΡƒΠ°Π»ΠΈΠ·ΠΈΡ€ΠΎΠ²Π°Ρ‚ΡŒ ΠΊΠ°ΠΆΠ΄ΡƒΡŽ ΠΈΠ· сфСричСских ΠΊΠΎΠΎΡ€Π΄ΠΈΠ½Π°Ρ‚ с ΠΏΠΎΠΌΠΎΡ‰ΡŒΡŽ гСомСтричСских структур, ΠΊΠΎΡ‚ΠΎΡ€Ρ‹Π΅ ΠΎΠΊΡ€Π°ΡˆΠ΅Π½Ρ‹ Π² соотвСтствии с Ρ†Π²Π΅Ρ‚Π°ΠΌΠΈ ΠΏΠΎΠ»Π·ΡƒΠ½ΠΊΠ°. Π”Π»ΠΈΠ½Π° ΠΎΡ‚Ρ€Π΅Π·ΠΊΠ° красной Π»ΠΈΠ½ΠΈΠΈ ΠΎΡ‚ Π½Π°Ρ‡Π°Π»Π° ΠΊΠΎΠΎΡ€Π΄ΠΈΠ½Π°Ρ‚ Ρ€Π°Π²Π½Π° $\rho$. Π£Π³ΠΎΠ» Π·Π΅Π»Π΅Π½ΠΎΠΉ части диска Π² плоскости $xy$ Ρ€Π°Π²Π΅Π½ $\theta$. Π£Π³ΠΎΠ» синСй части Π²Π΅Ρ€Ρ‚ΠΈΠΊΠ°Π»ΡŒΠ½ΠΎΠ³ΠΎ диска Ρ€Π°Π²Π΅Π½ $\phi$. Π’Ρ‹ Ρ‚Π°ΠΊΠΆΠ΅ ΠΌΠΎΠΆΠ΅Ρ‚Π΅ ΠΏΠ΅Ρ€Π΅ΠΌΠ΅Ρ‰Π°Ρ‚ΡŒ Π±ΠΎΠ»ΡŒΡˆΡƒΡŽ ΠΊΡ€Π°ΡΠ½ΡƒΡŽ Ρ‚ΠΎΡ‡ΠΊΡƒ ΠΈ Π·Π΅Π»Π΅Π½ΡƒΡŽ ΠΏΡ€ΠΎΠ΅ΠΊΡ†ΠΈΡŽ этой Ρ‚ΠΎΡ‡ΠΊΠΈ нСпосрСдствСнно с ΠΏΠΎΠΌΠΎΡ‰ΡŒΡŽ ΠΌΡ‹ΡˆΠΈ.

    Π”ΠΎΠΏΠΎΠ»Π½ΠΈΡ‚Π΅Π»ΡŒΠ½Π°Ρ информация ΠΎΠ± Π°ΠΏΠΏΠ»Π΅Ρ‚Π΅.

    ΠžΠ±Ρ€Π°Ρ‚ΠΈΡ‚Π΅ Π²Π½ΠΈΠΌΠ°Π½ΠΈΠ΅, ΠΊΠ°ΠΊ Π²Ρ‹ ΠΌΠΎΠΆΠ΅Ρ‚Π΅ ΠΏΠΎΠ»ΡƒΡ‡ΠΈΡ‚ΡŒ Π»ΡŽΠ±ΡƒΡŽ Ρ‚ΠΎΡ‡ΠΊΡƒ, Π΄Π°ΠΆΠ΅ Ссли ΠΌΡ‹ ΠΎΠ³Ρ€Π°Π½ΠΈΡ‡ΠΈΠ²Π°Π΅ΠΌ $\rho \ge 0$, $0 \le\Ρ‚Π΅Ρ‚Π°

    Π­Ρ‚ΠΈ ограничСния устранили Π±ΠΎΠ»ΡŒΡˆΡƒΡŽ Ρ‡Π°ΡΡ‚ΡŒ Π½Π΅ΡƒΠ½ΠΈΠΊΠ°Π»ΡŒΠ½ΠΎΡΡ‚ΠΈ сфСричСских ΠΊΠΎΠΎΡ€Π΄ΠΈΠ½Π°Ρ‚Ρ‹. ΠžΠ±Ρ€Π°Ρ‚ΠΈΡ‚Π΅ Π²Π½ΠΈΠΌΠ°Π½ΠΈΠ΅, Ρ‡Ρ‚ΠΎ Π² Ρ‚ΠΎΡ‡ΠΊΠ΅ $\rho =0$ всС Π΅Ρ‰Π΅ присутствуСт Π½Π΅Π΅Π΄ΠΈΠ½ΡΡ‚Π²Π΅Π½Π½ΠΎΡΡ‚ΡŒ, Π² Ρ‚ΠΎΡ‡ΠΊΠ΅ $\phi = 0$ ΠΈ ΠΏΡ€ΠΈ $\phi=\pi$. Когда любоС ΠΈΠ· этих условий Π²Π΅Ρ€Π½ΠΎ, Π²Ρ‹ ΠΌΠΎΠΆΠ΅Ρ‚Π΅ ΠΈΠ·ΠΌΠ΅Π½ΠΈΡ‚ΡŒ Π·Π½Π°Ρ‡Π΅Π½ΠΈΠ΅ ΠΎΠ΄Π½ΠΎΠΉ ΠΈΠ»ΠΈ Π½Π΅ΡΠΊΠΎΠ»ΡŒΠΊΠΈΡ… Π΄Ρ€ΡƒΠ³ΠΈΡ… ΠΊΠΎΠΎΡ€Π΄ΠΈΠ½Π°Ρ‚ Π±Π΅Π· пСрСмСщСния Ρ‚ΠΎΡ‡ΠΊΠΈ.

    К соТалСнию, соглашСниС ΠΎΠ± ΠΎΠ±ΠΎΠ·Π½Π°Ρ‡Π΅Π½ΠΈΠΈ сфСричСских ΠΊΠΎΠΎΡ€Π΄ΠΈΠ½Π°Ρ‚ Π½Π΅ стандартизировано Π² Ρ€Π°Π·Π½Ρ‹Ρ… дисциплинах.НапримСр, Π² Ρ„ΠΈΠ·ΠΈΠΊΠ΅ Ρ€ΠΎΠ»ΠΈ $\theta$ ΠΈ $\phi$ ΠΎΠ±Ρ‹Ρ‡Π½ΠΎ ΠΌΠ΅Π½ΡΡŽΡ‚ΡΡ мСстами. Π§Ρ‚ΠΎΠ±Ρ‹ ΠΏΡ€Π°Π²ΠΈΠ»ΡŒΠ½ΠΎ ΠΏΠΎΠ½ΡΡ‚ΡŒ, ΠΊΠ°ΠΊ ΠΊΡ‚ΠΎ-Ρ‚ΠΎ ΠΈΡΠΏΠΎΠ»ΡŒΠ·ΡƒΠ΅Ρ‚ сфСричСскиС ΠΊΠΎΠΎΡ€Π΄ΠΈΠ½Π°Ρ‚Ρ‹, Π²Ρ‹ Π΄ΠΎΠ»ΠΆΠ½Ρ‹ сначала ΠΎΠΏΡ€Π΅Π΄Π΅Π»ΠΈΡ‚ΡŒ, ΠΊΠ°ΠΊΠΈΠ΅ обозначСния ΠΈΡΠΏΠΎΠ»ΡŒΠ·ΡƒΡŽΡ‚ΡΡ ΠΏΡ€ΠΈ этом. Π’Ρ‹ Π½Π΅ ΠΌΠΎΠΆΠ΅Ρ‚Π΅ ΠΏΡ€Π΅Π΄ΠΏΠΎΠ»ΠΎΠΆΠΈΡ‚ΡŒ, Ρ‡Ρ‚ΠΎ ΠΎΠ½ΠΈ ΡΠ»Π΅Π΄ΡƒΡŽΡ‚ соглашСнию, ΠΈΡΠΏΠΎΠ»ΡŒΠ·ΡƒΠ΅ΠΌΠΎΠΌΡƒ здСсь.

    ΠŸΡ€ΠΎΡΡ‚Ρ‹Π΅ сфСричСскиС ΠΊΠΎΠΎΡ€Π΄ΠΈΠ½Π°Ρ‚Π½Ρ‹Π΅ повСрхности

    Π­Ρ‚ΠΈ Ρ‚Ρ€ΠΈ ΡΠ»Π΅Π΄ΡƒΡŽΡ‰ΠΈΡ… Π°ΠΏΠΏΠ»Π΅Ρ‚Π° ΠΌΠΎΠ³ΡƒΡ‚ ΠΏΠΎΠΌΠΎΡ‡ΡŒ Π²Π°ΠΌ ΠΏΠΎΠ½ΡΡ‚ΡŒ, Ρ‡Ρ‚ΠΎ Π΄Π΅Π»Π°Π΅Ρ‚ ΠΊΠ°ΠΆΠ΄Ρ‹ΠΉ ΠΈΠ· Ρ‚Ρ€Π΅Ρ… Π·Π½Π°Ρ‡ΠΈΡ‚ сфСричСскиС ΠΊΠΎΠΎΡ€Π΄ΠΈΠ½Π°Ρ‚Ρ‹. Они ΠΏΠΎΠΊΠ°Π·Ρ‹Π²Π°ΡŽΡ‚, ΠΊΠ°ΠΊΠΈΠ΅ повСрхности $\phi=$ константа, константа $\theta=$ ΠΈ константа $\rho=$ выглядят Ρ‚Π°ΠΊ.Π’ΠΎ Π·Π½Π°Ρ‡Π΅Π½ΠΈΠ΅ константы опрСдСляСтся ΠΏΠΎΠ»ΠΎΠΆΠ΅Π½ΠΈΠ΅ΠΌ ΠΏΠΎΠ»Π·ΡƒΠ½ΠΊΠΎΠ². Π’ΠΎ всСх случаях ΠΌΡ‹ ΠΎΠ³Ρ€Π°Π½ΠΈΡ‡ΠΈΠ²Π°Π΅ΠΌ повСрхности ΠΎΠ±Π»Π°ΡΡ‚ΡŒΡŽ $\rho

    ΠšΠΎΠ½ΡΡ‚Π°Π½Ρ‚Π° $\phi$

    Π§Ρ‚ΠΎ ΠΎΠ·Π½Π°Ρ‡Π°Π΅Ρ‚, Ρ‡Ρ‚ΠΎ Ρ‚ΠΎΡ‡ΠΊΠ° ΠΈΠΌΠ΅Π΅Ρ‚ ΡΡ„Π΅Ρ€ΠΈΡ‡Π΅ΡΠΊΡƒΡŽ ΠΊΠΎΠΎΡ€Π΄ΠΈΠ½Π°Ρ‚Ρƒ $\phi=\pi/3$? ВзглянитС Π½Π° повСрхности, ΠΊΠΎΡ‚ΠΎΡ€Ρ‹Π΅ опрСдСляСтся ΡƒΡ€Π°Π²Π½Π΅Π½ΠΈΠ΅ΠΌ $\phi=$ постоянная.

    Π—Π°Π³Ρ€ΡƒΠ·ΠΊΠ° Π°ΠΏΠΏΠ»Π΅Ρ‚Π°

    ΠŸΠΎΠ²Π΅Ρ€Ρ…Π½ΠΎΡΡ‚ΠΈ постоянных $\phi$ Π² сфСричСских ΠΊΠΎΠΎΡ€Π΄ΠΈΠ½Π°Ρ‚Π°Ρ…. Показана коничСская ΠΏΠΎΠ²Π΅Ρ€Ρ…Π½ΠΎΡΡ‚ΡŒ константы $\phi=$, Π³Π΄Π΅ Π·Π½Π°Ρ‡Π΅Π½ΠΈΠ΅ $\phi$ опрСдСляСтся синСй Ρ‚ΠΎΡ‡ΠΊΠΎΠΉ Π½Π° Π±Π΅Π³ΡƒΠ½ΠΊΠ΅.Волько Ρ‡Π°ΡΡ‚ΡŒ повСрхности Π³Π΄Π΅ $\rho

    ΠŸΠΎΠ΄Ρ€ΠΎΠ±Π½Π΅Π΅ ΠΎ Π°ΠΏΠΏΠ»Π΅Ρ‚Π΅.

    ΠŸΠΎΠ²Π΅Ρ€Ρ…Π½ΠΎΡΡ‚Π½Π°Ρ константа $\phi=$ прСдставляСт собой просто ΠΎΠ΄ΠΈΠ½ΠΎΡ‡Π½Ρ‹ΠΉ конус, Π½Π°ΠΏΡ€Π°Π²Π»Π΅Π½Π½Ρ‹ΠΉ Π»ΠΈΠ±ΠΎ Π²Π²Π΅Ρ€Ρ… ΠΈΠ»ΠΈ Π²Π½ΠΈΠ·. Если Π²Ρ‹ Π·Π½Π°Π΅Ρ‚Π΅, Ρ‡Ρ‚ΠΎ $\phi=\pi/3$, Ρ‚ΠΎ Π²Ρ‹ Π·Π½Π°Π΅Ρ‚Π΅ Ρ‚ΠΎΡ‡ΠΊΠ° находится Π³Π΄Π΅-Ρ‚ΠΎ Π½Π° (ΡˆΠΈΡ€ΠΎΠΊΠΎΠΌ) СдинствСнном конусС, ΠΊΠΎΡ‚ΠΎΡ€Ρ‹ΠΉ открываСтся Π²Π²Π΅Ρ€Ρ…, Ρ‚. Π΅. Π£Ρ€Π°Π²Π½Π΅Π½ΠΈΠ΅ $\phi=\pi/3$ Π·Π°Π΄Π°Π΅Ρ‚ ΠΏΠΎΠ²Π΅Ρ€Ρ…Π½ΠΎΡΡ‚ΡŒ, которая прСдставляСт собой ΠΎΠ΄Π½ΠΎ конусноС отвСрстиС Π²Π²Π΅Ρ€Ρ…. Π£Ρ€Π°Π²Π½Π΅Π½ΠΈΠ΅ $\phi=\pi/2$ соотвСтствуСт плоскости $xy$.

    ΠŸΠΎΠ²Π΅Ρ€Ρ…Π½ΠΎΡΡ‚Π½Π°Ρ константа $\phi=$ осСсиммСтрична ΠΎΡ‚Π½ΠΎΡΠΈΡ‚Π΅Π»ΡŒΠ½ΠΎ оси $z$.2}$$ Π³Π΄Π΅ $C=1/\tan \phi$, Ρ‡Ρ‚ΠΎ Π΄Π΅ΠΉΡΡ‚Π²ΠΈΡ‚Π΅Π»ΡŒΠ½ΠΎ являСтся ΡƒΡ€Π°Π²Π½Π΅Π½ΠΈΠ΅ΠΌ для конуса.

    ΠšΠΎΠ½ΡΡ‚Π°Π½Ρ‚Π° $\theta$

    ΠšΠΎΠ½ΡΡ‚Π°Π½Ρ‚Π° $\theta=$ прСдставляСт собой ΠΏΠΎΠ»ΡƒΠΏΠ»ΠΎΡΠΊΠΎΡΡ‚ΡŒ Π²Π½Π΅ оси $z$. (Π­Ρ‚ΠΎ отобраТаСтся ΠΊΠ°ΠΊ ΠΏΠΎΠ»ΡƒΠΊΡ€ΡƒΠ³ Ρ‚ΠΎΠ»ΡŒΠΊΠΎ ΠΏΠΎΡ‚ΠΎΠΌΡƒ, Ρ‡Ρ‚ΠΎ ΠΌΡ‹ ΠΎΠ³Ρ€Π°Π½ΠΈΡ‡ΠΈΠ²Π°Π΅ΠΌ Π³Ρ€Π°Ρ„ΠΈΠΊ $\Ρ€ΠΎ

    Π—Π°Π³Ρ€ΡƒΠ·ΠΊΠ° Π°ΠΏΠΏΠ»Π΅Ρ‚Π°

    ΠŸΠΎΠ²Π΅Ρ€Ρ…Π½ΠΎΡΡ‚ΠΈ постоянных $\theta$ Π² сфСричСских ΠΊΠΎΠΎΡ€Π΄ΠΈΠ½Π°Ρ‚Π°Ρ…. Показана полуплоская ΠΏΠΎΠ²Π΅Ρ€Ρ…Π½ΠΎΡΡ‚ΡŒ константы $\theta=$, Π³Π΄Π΅ Π·Π½Π°Ρ‡Π΅Π½ΠΈΠ΅ $\theta$ опрСдСляСтся синСй Ρ‚ΠΎΡ‡ΠΊΠΎΠΉ Π½Π° Π±Π΅Π³ΡƒΠ½ΠΊΠ΅.Волько Ρ‡Π°ΡΡ‚ΡŒ повСрхности Π³Π΄Π΅ $\rho

    ΠŸΠΎΠ΄Ρ€ΠΎΠ±Π½Π΅Π΅ ΠΎ Π°ΠΏΠΏΠ»Π΅Ρ‚Π΅.

    Если Ρ‚ΠΎΡ‡ΠΊΠ° ΠΈΠΌΠ΅Π΅Ρ‚ $\theta=\pi/2$, Ρ‚ΠΎ Π²Ρ‹ Π·Π½Π°Π΅Ρ‚Π΅, Ρ‡Ρ‚ΠΎ Ρ‚ΠΎΡ‡ΠΊΠ° находится Π½Π° ΠΏΠΎΠ»ΠΎΠ²ΠΈΠ½Π° плоскости $yz$, Π³Π΄Π΅ значСния $y$ ΠΏΠΎΠ»ΠΎΠΆΠΈΡ‚Π΅Π»ΡŒΠ½Ρ‹. Π£Ρ€Π°Π²Π½Π΅Π½ΠΈΠ΅ $\theta=\pi/2$ β€” ΡƒΡ€Π°Π²Π½Π΅Π½ΠΈΠ΅ для этой полуплоскости.

    Из ΡΠΎΠΎΡ‚Π½ΠΎΡˆΠ΅Π½ΠΈΡ \eqref{spherical_cartesian} ΠΎΡ‚Π½ΠΎΡˆΠ΅Π½ΠΈΠ΅ ΠΌΠ΅ΠΆΠ΄Ρƒ $x$ ΠΈ $y$ ΠΌΠΎΠΆΠ½ΠΎ Π·Π°ΠΏΠΈΡΠ°Ρ‚ΡŒ, Π½Π°ΠΏΡ€ΠΈΠΌΠ΅Ρ€, ΠΊΠ°ΠΊ $y/x = \tan \theta$. Если $\theta$ поддСрТиваСтся постоянным, Ρ‚ΠΎ ΠΎΡ‚Π½ΠΎΡˆΠ΅Π½ΠΈΠ΅ ΠΌΠ΅ΠΆΠ΄Ρƒ $x$ ΠΈ $y$ остаСтся постоянным.Π’Π°ΠΊΠΈΠΌ ΠΎΠ±Ρ€Π°Π·ΠΎΠΌ, константа $\theta=$ Π΄Π°Π΅Ρ‚ ΠΏΡ€ΡΠΌΡƒΡŽ, ΠΏΡ€ΠΎΡ…ΠΎΠ΄ΡΡ‰ΡƒΡŽ Ρ‡Π΅Ρ€Π΅Π· Π½Π°Ρ‡Π°Π»ΠΎ ΠΊΠΎΠΎΡ€Π΄ΠΈΠ½Π°Ρ‚ Π² плоскости $xy$. Π’Π°ΠΊ ΠΊΠ°ΠΊ $z$ Π½Π΅ ΠΈΠΌΠ΅Π΅Ρ‚ ΠΎΠ³Ρ€Π°Π½ΠΈΡ‡Π΅Π½ΠΈΠΉ, ΠΌΡ‹ ΠΏΠΎΠ»ΡƒΡ‡Π°Π΅ΠΌ Π²Π΅Ρ€Ρ‚ΠΈΠΊΠ°Π»ΡŒΠ½ΡƒΡŽ ΠΏΠ»ΠΎΡΠΊΠΎΡΡ‚ΡŒ. ΠžΠ³Π»ΡΠ΄Ρ‹Π²Π°ΡΡΡŒ Π½Π°Π·Π°Π΄ Π½Π° ΠΎΡ‚Π½ΠΎΡˆΠ΅Π½ΠΈΠ΅ \eqref{spherical_cartesian}, ΠΌΡ‹ Π²ΠΈΠ΄ΠΈΠΌ, Ρ‡Ρ‚ΠΎ это всСго лишь ΠΏΠΎΠ»ΡƒΠΏΠ»ΠΎΡΠΊΠΎΡΡ‚ΡŒ, ΠΏΠΎΡ‚ΠΎΠΌΡƒ Ρ‡Ρ‚ΠΎ $\rho\sin\phi$ Π½Π΅ ΠΌΠΎΠΆΠ΅Ρ‚ Π±Ρ‹Ρ‚ΡŒ ΠΎΡ‚Ρ€ΠΈΡ†Π°Ρ‚Π΅Π»ΡŒΠ½Ρ‹ΠΌ.

    ΠšΠΎΠ½ΡΡ‚Π°Π½Ρ‚Π° $\rho$

    Π£ Π±ΠΎΠ»ΡŒΡˆΠΈΠ½ΡΡ‚Π²Π° людСй Π½Π΅Ρ‚ ΠΏΡ€ΠΎΠ±Π»Π΅ΠΌ с ΠΏΠΎΠ½ΠΈΠΌΠ°Π½ΠΈΠ΅ΠΌ Ρ‚ΠΎΠ³ΠΎ, Ρ‡Ρ‚ΠΎ ΠΎΠ·Π½Π°Ρ‡Π°Π΅Ρ‚ $\rho=3$. Π­Ρ‚ΠΎ сфСра радиуса 3 с Ρ†Π΅Π½Ρ‚Ρ€ΠΎΠΌ Π² Π½Π°Ρ‡Π°Π»Π΅ ΠΊΠΎΠΎΡ€Π΄ΠΈΠ½Π°Ρ‚. Π’ ΠΎΠ±Ρ‰Π΅ΠΌ, ΠΏΠΎΠ²Π΅Ρ€Ρ…Π½ΠΎΡΡ‚ΡŒ ΠšΠΎΠ½ΡΡ‚Π°Π½Ρ‚Π° $\rho=$ β€” это сфСра радиуса $\rho$ с Ρ†Π΅Π½Ρ‚Ρ€ΠΎΠΌ Π² Π½Π°Ρ‡Π°Π»Π΅ ΠΊΠΎΠΎΡ€Π΄ΠΈΠ½Π°Ρ‚.2 \ΠΊΠΎΠ½Π΅Ρ†{Π²Ρ‹Ρ€Π°Π²Π½ΠΈΠ²Π°Π½ΠΈΠ΅*} провСряя, Ρ‡Ρ‚ΠΎ константа $\rho=$ Π΅ΡΡ‚ΡŒ сфСра радиуса $\rho$ с Ρ†Π΅Π½Ρ‚Ρ€ΠΎΠΌ Π² Π½Π°Ρ‡Π°Π»Π΅ ΠΊΠΎΠΎΡ€Π΄ΠΈΠ½Π°Ρ‚.

    полярных ΠΊΠΎΠΎΡ€Π΄ΠΈΠ½Π°Ρ‚ πŸŒ€ | Varun Vachhar

    Π”Π΅ΠΊΠ°Ρ€Ρ‚ΠΎΠ²Π° систСма ΠΊΠΎΠΎΡ€Π΄ΠΈΠ½Π°Ρ‚

    БистСма ΠΊΠΎΠΎΡ€Π΄ΠΈΠ½Π°Ρ‚ позволяСт Π½Π°ΠΌ ΠΈΡΠΏΠΎΠ»ΡŒΠ·ΠΎΠ²Π°Ρ‚ΡŒ числа для опрСдСлСния полоТСния Ρ‚ΠΎΡ‡Π΅ΠΊ Π² 2D ΠΈΠ»ΠΈ 3D пространствС. НаиболСС популярной систСмой ΠΊΠΎΠΎΡ€Π΄ΠΈΠ½Π°Ρ‚ являСтся, вСроятно, Π΄Π΅ΠΊΠ°Ρ€Ρ‚ΠΎΠ²Π° систСма ΠΊΠΎΠΎΡ€Π΄ΠΈΠ½Π°Ρ‚. Он позволяСт Π½Π°ΠΉΡ‚ΠΈ ΠΊΠ°ΠΆΠ΄ΡƒΡŽ Ρ‚ΠΎΡ‡ΠΊΡƒ ΠΏΠΎ ΠΏΠ°Ρ€Π΅ числовых ΠΊΠΎΠΎΡ€Π΄ΠΈΠ½Π°Ρ‚ (x, y) .Π•ΡΡ‚ΡŒ ΠΎΡ‡Π΅Π½ΡŒ Ρ…ΠΎΡ€ΠΎΡˆΠΈΠΉ шанс, Ρ‡Ρ‚ΠΎ Π²Ρ‹ использовали эту систСму. Он Π²Π΅Π·Π΄Π΅ β€” SVG, Canvas, WebGL ΠΈ Π΄Π°ΠΆΠ΅ Sketch & Illustrator.

    (x, y)xyPO Рисунок 1: Π”Π΅ΠΊΠ°Ρ€Ρ‚ΠΎΠ²Π° систСма ΠΊΠΎΠΎΡ€Π΄ΠΈΠ½Π°Ρ‚

    ΠŸΠΎΠ»ΡΡ€Π½Π°Ρ систСма ΠΊΠΎΠΎΡ€Π΄ΠΈΠ½Π°Ρ‚

    ΠŸΠΎΠ»ΡΡ€Π½Π°Ρ систСма ΠΊΠΎΠΎΡ€Π΄ΠΈΠ½Π°Ρ‚ β€” это двумСрная систСма ΠΊΠΎΠΎΡ€Π΄ΠΈΠ½Π°Ρ‚, Π² ΠΊΠΎΡ‚ΠΎΡ€ΠΎΠΉ каТдая Ρ‚ΠΎΡ‡ΠΊΠ° опрСдСляСтся r ΠΈ ΞΈ . Π“Π΄Π΅ r β€” расстояниС ΠΎΡ‚ Π½Π°Ρ‡Π°Π»Π° ΠΊΠΎΠΎΡ€Π΄ΠΈΠ½Π°Ρ‚, Π° ΞΈ β€” ΡƒΠ³ΠΎΠ» ΠΎΡ‚ оси x.

    ΞΈ = 30Β°r = 2xyPO Рис. 2: ΠŸΠΎΠ»ΡΡ€Π½Π°Ρ систСма ΠΊΠΎΠΎΡ€Π΄ΠΈΠ½Π°Ρ‚

    ΠŸΡ€Π΅ΠΎΠ±Ρ€Π°Π·ΠΎΠ²Π°Π½ΠΈΠ΅ полярных ΠΈ Π΄Π΅ΠΊΠ°Ρ€Ρ‚ΠΎΠ²Ρ‹Ρ… ΠΊΠΎΠΎΡ€Π΄ΠΈΠ½Π°Ρ‚

    ВригономСтрия! Π›ΡŽΠ±ΠΈΡ‚Π΅ это ΠΈΠ»ΠΈ Π½Π΅Π½Π°Π²ΠΈΠ΄ΠΈΡ‚Π΅, это Π²Π΅Π·Π΄Π΅.Π’Ρ‹ ΠΌΠΎΠΆΠ΅Ρ‚Π΅ ΠΏΡ€Π΅ΠΎΠ±Ρ€Π°Π·ΠΎΠ²Π°Ρ‚ΡŒ Ρ‚ΠΎΡ‡ΠΊΡƒ ΠΈΠ· полярных ΠΊΠΎΠΎΡ€Π΄ΠΈΠ½Π°Ρ‚ (r, ΞΈ) Π² Π΄Π΅ΠΊΠ°Ρ€Ρ‚ΠΎΠ²Ρ‹ ΠΊΠΎΠΎΡ€Π΄ΠΈΠ½Π°Ρ‚Ρ‹ (x, y) , ΠΈΡΠΏΠΎΠ»ΡŒΠ·ΡƒΡ ΠΏΡ€ΠΈΠ²Π΅Π΄Π΅Π½Π½Ρ‹Π΅ Π½ΠΈΠΆΠ΅ уравнСния. Π­Ρ‚ΠΎ ΠΎΡ‡Π΅Π½ΡŒ ΡƒΠ΄ΠΎΠ±Π½ΠΎ, Ρ‚Π°ΠΊ ΠΊΠ°ΠΊ Π±ΠΎΠ»ΡŒΡˆΠΈΠ½ΡΡ‚Π²ΠΎ инструмСнтов ΠΏΡ€ΠΈΠ½ΠΈΠΌΠ°ΡŽΡ‚ Ρ‚ΠΎΠ»ΡŒΠΊΠΎ x ΠΈ y Π² качСствС Ρ‚ΠΎΡ‡Π΅ΠΊ.

      const x = r * Math.cos(Ρ‚Π΅Ρ‚Π°);
    const y = r * Math.sin(theta);  

    Π£Π·ΠΎΡ€Ρ‹

    НиТС Ρƒ мСня Π΅ΡΡ‚ΡŒ нСсколько Π°Π½ΠΈΠΌΠ°Ρ†ΠΈΠΉ Дэйва Π£Π°ΠΉΡ‚Π°, Ρ‚Π°ΠΊΠΆΠ΅ извСстного ΠΊΠ°ΠΊ ΠΏΡ‡Π΅Π»Ρ‹ ΠΈ Π±ΠΎΠΌΠ±Ρ‹. ВсС ΠΎΠ½ΠΈ ΠΈΠΌΠ΅ΡŽΡ‚ ΠΎΠ΄Π½Ρƒ ΠΎΠ±Ρ‰ΡƒΡŽ Ρ‡Π΅Ρ€Ρ‚Ρƒ: ΠΎΠ½ΠΈ ΠΈΡΠΏΠΎΠ»ΡŒΠ·ΡƒΡŽΡ‚ полярныС ΠΊΠΎΠΎΡ€Π΄ΠΈΠ½Π°Ρ‚Ρ‹ для создания шаблона.

    Рисунок 3: Π¨Π°Π±Π»ΠΎΠ½Ρ‹ , сгСнСрированныС с использованиСм полярных ΠΊΠΎΠΎΡ€Π΄ΠΈΠ½Π°Ρ‚

    Π”Π΅ΠΊΠ°Ρ€Ρ‚ΠΎΠ²Ρ‹ ΠΊΠΎΠΎΡ€Π΄ΠΈΠ½Π°Ρ‚Ρ‹ β€” ΠΎΡ‚Π»ΠΈΡ‡Π½Ρ‹ΠΉ Π²Ρ‹Π±ΠΎΡ€ для Ρ€Π°Π²Π½ΠΎΠΌΠ΅Ρ€Π½ΠΎΠ³ΠΎ размСщСния элСмСнтов Π½Π° ΠΏΡ€ΡΠΌΠΎΡƒΠ³ΠΎΠ»ΡŒΠ½ΠΎΠΉ сСткС. Однако, Ссли Π²Ρ‹ Ρ…ΠΎΡ‚ΠΈΡ‚Π΅ Ρ€Π°Π²Π½ΠΎΠΌΠ΅Ρ€Π½ΠΎ Ρ€Π°ΡΠΏΡ€Π΅Π΄Π΅Π»ΠΈΡ‚ΡŒ ΠΎΠ±ΡŠΠ΅ΠΊΡ‚Ρ‹ ΠΏΠΎ ΠΊΡ€ΡƒΠ³Ρƒ, Π»ΡƒΡ‡ΡˆΠ΅ всСго ΠΈΡΠΏΠΎΠ»ΡŒΠ·ΠΎΠ²Π°Ρ‚ΡŒ полярныС ΠΊΠΎΠΎΡ€Π΄ΠΈΠ½Π°Ρ‚Ρ‹. Бохраняя радиус постоянным, ΠΌΡ‹ вычисляСм ΡƒΠ³ΠΎΠ» для ΠΊΠ°ΠΆΠ΄ΠΎΠΉ Ρ‚ΠΎΡ‡ΠΊΠΈ ΠΊΠ°ΠΊ ΡƒΠ³ΠΎΠ» = 360Β° * индСкс / количСство сторон .

    360Β°30Β°60Β°90Β°120Β°150Β°180Β°210Β°240Β°270Β°300Β°330Β° Рисунок 4: Π Π°Π²Π½ΠΎΠΌΠ΅Ρ€Π½ΠΎΠ΅ Ρ€Π°Π·ΠΌΠ΅Ρ‰Π΅Π½ΠΈΠ΅ элСмСнтов ΠΏΠΎ ΠΊΡ€ΡƒΠ³Ρƒ

    НиТС ΠΏΡ€ΠΈΠ²Π΅Π΄Π΅Π½Π° рСализация этой ΠΈΠ΄Π΅ΠΈ Π½Π° JavaScript.Ѐункция Ρ‚ΠΎΡ‡Π΅ΠΊ Ρ‚Π°ΠΊΠΆΠ΅ ΠΏΡ€ΠΈΠ½ΠΈΠΌΠ°Π΅Ρ‚ Π½Π΅ΠΎΠ±ΡΠ·Π°Ρ‚Π΅Π»ΡŒΠ½Ρ‹ΠΉ Π°Ρ€Π³ΡƒΠΌΠ΅Π½Ρ‚ offset для смСщСния Ρ‚ΠΎΡ‡Π΅ΠΊ ΠΏΠΎ ΠΏΠ΅Ρ€ΠΈΠΌΠ΅Ρ‚Ρ€Ρƒ ΠΊΡ€ΡƒΠ³Π°. НапримСр, ΠΊΡ€Π°ΠΉΠ½ΠΈΠΉ ΠΊΡ€ΡƒΠ³ Ρ‚ΠΎΡ‡Π΅ΠΊ Π² Рисунок 4 ΠΌΠΎΠΆΠ΅Ρ‚ Π±Ρ‹Ρ‚ΡŒ сгСнСрирован с использованиСм Ρ‚ΠΎΡ‡Π΅ΠΊ(12, 200) . Π‘Π»Π΅Π΄ΡƒΡŽΡ‰ΠΈΠΉ Π²Π½ΡƒΡ‚Ρ€ΡŒ с использованиСм ΠΎΡ‡ΠΊΠΎΠ² (12, 175, 15) . Π‘Π»Π΅Π΄ΡƒΡŽΡ‰ΠΈΠΉ Π·Π° этим ΠΈΡΠΏΠΎΠ»ΡŒΠ·ΡƒΠ΅Ρ‚ ΠΎΡ‡ΠΊΠΎΠ² (12, 150, 30) ΠΈ Ρ‚Π°ΠΊ Π΄Π°Π»Π΅Π΅. Моя заставка CodePen Π±Ρ‹Π»Π° построСна с использованиСм этого.

      Ρ„ΡƒΠ½ΠΊΡ†ΠΈΠΎΠ½Π°Π»ΡŒΠ½Ρ‹Ρ… Ρ‚ΠΎΡ‡Π΅ΠΊ (количСство, радиус, смСщСниС = 0) {
      постоянный ΡƒΠ³ΠΎΠ» = 360/счСт;
      const vertexIndices = Π΄ΠΈΠ°ΠΏΠ°Π·ΠΎΠ½ (количСство);
    
      Π²Π΅Ρ€Π½ΡƒΡ‚ΡŒ Π²Π΅Ρ€ΡˆΠΈΠ½Π½Ρ‹Π΅ индСксы.ΠΊΠ°Ρ€Ρ‚Π° (индСкс => {
        Π²ΠΎΠ·Π²Ρ€Π°Ρ‰Π΅Π½ΠΈΠ΅ {
          Ρ‚Π΅Ρ‚Π°: смСщСниС + градусы ΠΊ Ρ€Π°Π΄ΠΈΠ°Π½Π°ΠΌ (смСщСниС + ΡƒΠ³ΠΎΠ» * индСкс),
          Π³: радиус,
        };
      });
    }
    
    
    Π΄ΠΈΠ°ΠΏΠ°Π·ΠΎΠ½ Ρ„ΡƒΠ½ΠΊΡ†ΠΈΠΉ (количСство) {
      Π²Π΅Ρ€Π½ΡƒΡ‚ΡŒ Array.from(Array(count).keys());
    }
    
    функция DegreesToRadians (angleInDegrees) {
      return (Math.PI * angleInDegrees) / 180;
    }  

    Π“Π΅Π½Π΅Ρ€Π°Ρ‚ΠΎΡ€ ΠΌΠ½ΠΎΠ³ΠΎΡƒΠ³ΠΎΠ»ΡŒΠ½ΠΈΠΊΠΎΠ²

    ΠŸΡ€Π°Π²ΠΈΠ»ΡŒΠ½Ρ‹ΠΉ ΠΌΠ½ΠΎΠ³ΠΎΡƒΠ³ΠΎΠ»ΡŒΠ½ΠΈΠΊ β€” это Ρ€Π°Π²Π½ΠΎΡƒΠ³ΠΎΠ»ΡŒΠ½Ρ‹ΠΉ ΠΌΠ½ΠΎΠ³ΠΎΡƒΠ³ΠΎΠ»ΡŒΠ½ΠΈΠΊ, Ρ‚. Π΅. всС Π΅Π³ΠΎ ΡƒΠ³Π»Ρ‹ Ρ€Π°Π²Π½Ρ‹ ΠΈ всС Π΅Π³ΠΎ стороны ΠΈΠΌΠ΅ΡŽΡ‚ ΠΎΠ΄ΠΈΠ½Π°ΠΊΠΎΠ²ΡƒΡŽ Π΄Π»ΠΈΠ½Ρƒ. Π­Ρ‚ΠΎ ΠΎΠ·Π½Π°Ρ‡Π°Π΅Ρ‚, Ρ‡Ρ‚ΠΎ всС Π²Π΅Ρ€ΡˆΠΈΠ½Ρ‹ ΠΏΡ€Π°Π²ΠΈΠ»ΡŒΠ½ΠΎΠ³ΠΎ ΠΌΠ½ΠΎΠ³ΠΎΡƒΠ³ΠΎΠ»ΡŒΠ½ΠΈΠΊΠ° ΡΠ²Π»ΡΡŽΡ‚ΡΡ Ρ‚ΠΎΡ‡ΠΊΠ°ΠΌΠΈ, Ρ€Π°Π²Π½ΠΎΠΌΠ΅Ρ€Π½ΠΎ располоТСнными Π½Π° окруТности.И ΠΊΠ°ΠΊ ΠΆΠ΅ ΡƒΠ΄ΠΎΠ±Π½ΠΎ, Ρ‡Ρ‚ΠΎ ΠΌΡ‹ Ρ‚ΠΎΠ»ΡŒΠΊΠΎ Ρ‡Ρ‚ΠΎ создали Ρ„ΡƒΠ½ΠΊΡ†ΠΈΡŽ, которая Π³Π΅Π½Π΅Ρ€ΠΈΡ€ΡƒΠ΅Ρ‚ ΠΈΠΌΠ΅Π½Π½ΠΎ это!

    360Β°30Β°60Β°90Β°120Β°150Β°180Β°210Β°240Β°270Β°300Β°330Β° Рисунок 5: РисованиС ΠΌΠ½ΠΎΠ³ΠΎΡƒΠ³ΠΎΠ»ΡŒΠ½ΠΈΠΊΠ° ΠΏΡƒΡ‚Π΅ΠΌ соСдинСния полярных ΠΊΠΎΠΎΡ€Π΄ΠΈΠ½Π°Ρ‚

    Для создания ΠΏΠΎΠ»ΠΈΠ³ΠΎΠ½Π° SVG ΠΌΡ‹ создадим список Ρ‚ΠΎΡ‡ΠΊΠΈ с ΠΏΠΎΠΌΠΎΡ‰ΡŒΡŽ Ρ„ΡƒΠ½ΠΊΡ†ΠΈΠΈ Ρ‚ΠΎΡ‡Π΅ΠΊ , Π° Π·Π°Ρ‚Π΅ΠΌ просто соСдинитС Ρ‚ΠΎΡ‡ΠΊΠΈ. Π‘ SVG Ρƒ нас Π΅ΡΡ‚ΡŒ Π΄Π²Π° Π²Π°Ρ€ΠΈΠ°Π½Ρ‚Π° для этого: ΠΈΠ»ΠΈ . Π’ ΠΏΡ€ΠΈΠ²Π΅Π΄Π΅Π½Π½ΠΎΠΌ Π½ΠΈΠΆΠ΅ ΠΏΡ€ΠΈΠΌΠ΅Ρ€Π΅ создаСтся Π°Ρ‚Ρ€ΠΈΠ±ΡƒΡ‚ Ρ‚ΠΎΡ‡Π΅ΠΊ для элСмСнта .

     
    function polygon (noOfSides, радиус описанной окруТности, Π²Ρ€Π°Ρ‰Π΅Π½ΠΈΠ΅) {
      Ρ‚ΠΎΡ‡ΠΊΠΈ Π²ΠΎΠ·Π²Ρ€Π°Ρ‚Π° (noOfSides, ΠΎΠΊΡ€ΡƒΠΆΠ½ΠΎΡΡ‚ΡŒ, Π²Ρ€Π°Ρ‰Π΅Π½ΠΈΠ΅)
        .map(Π² Π΄Π΅ΠΊΠ°Ρ€Ρ‚ΠΎΠ²ΠΎ)
        .ΠΏΡ€ΠΈΡΠΎΠ΅Π΄ΠΈΠ½ΠΈΡ‚ΡŒΡΡ(' ');
    }
    
    функция toCartesian({r, theta}) {
      return [r * Math.cos(Ρ‚Π΅Ρ‚Π°), r * Math.sin(Ρ‚Π΅Ρ‚Π°)];
    }  

    ΠšΠ°ΠΆΠ΅Ρ‚ΡΡ простым, Π½ΠΎ с Π½ΠΈΠΌ ΠΌΠΎΠΆΠ½ΠΎ ΡΠ΄Π΅Π»Π°Ρ‚ΡŒ ΠΌΠ½ΠΎΠ³ΠΎ интСрСсного. Π’ΠΎΡ‚ ΠΏΠ°Ρ€Π° ΠΏΡ€ΠΈΠΌΠ΅Ρ€ΠΎΠ²:

    Π”Ρ€Π°Π³ΠΎΡ†Π΅Π½Π½Ρ‹Π΅ ΠΊΠ°ΠΌΠ½ΠΈ, сгСнСрированныС с использованиСм полярных ΠΊΠΎΠΎΡ€Π΄ΠΈΠ½Π°Ρ‚

    ΠžΡ‚Π½ΠΎΡΠΈΡ‚Π΅Π»ΡŒΠ½Ρ‹Π΅ полярныС ΠΊΠΎΠΎΡ€Π΄ΠΈΠ½Π°Ρ‚Ρ‹

    Когда Π²Ρ‹ опрСдСляСтС Ρ‚ΠΎΡ‡ΠΊΡƒ ΠΊΠ°ΠΊ (r, ΞΈ) , ΠΏΠΎ ΡƒΠΌΠΎΠ»Ρ‡Π°Π½ΠΈΡŽ это ΠΎΡ‚Π½ΠΎΡΠΈΡ‚Π΅Π»ΡŒΠ½ΠΎ Π½Π°Ρ‡Π°Π»Π° ΠΊΠΎΠΎΡ€Π΄ΠΈΠ½Π°Ρ‚ (0, 0) .ΠœΡ‹ ΠΌΠΎΠΆΠ΅ΠΌ ΠΎΠΏΡ€Π΅Π΄Π΅Π»ΠΈΡ‚ΡŒ Ρ‚ΠΎΡ‡ΠΊΠΈ ΠΎΡ‚Π½ΠΎΡΠΈΡ‚Π΅Π»ΡŒΠ½ΠΎ Π΄Ρ€ΡƒΠ³ΠΈΡ… Ρ‚ΠΎΡ‡Π΅ΠΊ, сдвинув Π½Π°Ρ‡Π°Π»ΠΎ ΠΊΠΎΠΎΡ€Π΄ΠΈΠ½Π°Ρ‚. Π­Ρ‚ΠΎ часто ΠΈΡΠΏΠΎΠ»ΡŒΠ·ΡƒΠ΅Ρ‚ΡΡ для опрСдСлСния полоТСния ΠΌΠ°Ρ€ΠΊΠ΅Ρ€ΠΎΠ² ΠΊΡ€ΠΈΠ²ΠΎΠΉ ΠΎΡ‚Π½ΠΎΡΠΈΡ‚Π΅Π»ΡŒΠ½ΠΎ Π²Π΅Ρ€ΡˆΠΈΠ½Ρ‹.

      const x = cx + r * Math.cos(theta);
    const y = cy + r * Math.sin(theta);  
    60 градусов = 3cxcyPOC Рисунок 6: ΠžΡ‚Π½ΠΎΡΠΈΡ‚Π΅Π»ΡŒΠ½Ρ‹Π΅ полярныС ΠΊΠΎΠΎΡ€Π΄ΠΈΠ½Π°Ρ‚Ρ‹

    ΠœΡ‹ ΠΌΠΎΠΆΠ΅ΠΌ ΠΈΠ·ΠΌΠ΅Π½ΠΈΡ‚ΡŒ Π³Π΅Π½Π΅Ρ€Π°Ρ‚ΠΎΡ€ ΠΏΠΎΠ»ΠΈΠ³ΠΎΠ½ΠΎΠ², Ρ‡Ρ‚ΠΎΠ±Ρ‹ ΠΌΡ‹ ΠΌΠΎΠ³Π»ΠΈ Ρ€ΠΈΡΠΎΠ²Π°Ρ‚ΡŒ ΠΌΠ½ΠΎΠ³ΠΎΡƒΠ³ΠΎΠ»ΡŒΠ½ΠΈΠΊ с Ρ†Π΅Π½Ρ‚Ρ€ΠΎΠΌ Π² любом мСстС Π½Π° холстС SVG.

      function polygon(noOfSides, ΠΎΠΊΡ€ΡƒΠΆΠ½ΠΎΡΡ‚ΡŒ, Π²Ρ€Π°Ρ‰Π΅Π½ΠΈΠ΅, [cx = 0, cy = 0]) {
      Ρ‚ΠΎΡ‡ΠΊΠΈ Π²ΠΎΠ·Π²Ρ€Π°Ρ‚Π° (noOfSides, ΠΎΠΊΡ€ΡƒΠΆΠ½ΠΎΡΡ‚ΡŒ, Π²Ρ€Π°Ρ‰Π΅Π½ΠΈΠ΅)
        .map(pt => toCartesian(pt, [cx, cy]))
        .ΠΏΡ€ΠΈΡΠΎΠ΅Π΄ΠΈΠ½ΠΈΡ‚ΡŒΡΡ(' ');
    }
    
    function toCartesian({r, theta}, [cx, cy]) {
      return [cx + r * Math.cos(Ρ‚Π΅Ρ‚Π°), cy + r * Math.sin(Ρ‚Π΅Ρ‚Π°)];
    }  

    Π’Ρ€Π°Ρ‰Π΅Π½ΠΈΠ΅

    Π”Ρ€ΡƒΠ³ΠΈΠΌ распространСнным ΠΏΡ€ΠΈΠΌΠ΅Π½Π΅Π½ΠΈΠ΅ΠΌ полярных ΠΊΠΎΠΎΡ€Π΄ΠΈΠ½Π°Ρ‚ являСтся Π²Ρ€Π°Ρ‰Π΅Π½ΠΈΠ΅ ΠΎΠ±ΡŠΠ΅ΠΊΡ‚ΠΎΠ² Π²ΠΎΠΊΡ€ΡƒΠ³ Ρ‚ΠΎΡ‡ΠΊΠΈ. Π—Π΄Π΅ΡΡŒ (cx, cy) β€” это Ρ‚ΠΎΡ‡ΠΊΠ°, Π²ΠΎΠΊΡ€ΡƒΠ³ ΠΊΠΎΡ‚ΠΎΡ€ΠΎΠΉ Π²Ρ‹ Ρ…ΠΎΡ‚ΠΈΡ‚Π΅ Π²Ρ€Π°Ρ‰Π°Ρ‚ΡŒΡΡ.

      x = cx + r * Math.cos(Ρ‚Π΅Ρ‚Π°);
    y = cy + r * Math.sin(theta);
    
    
    window.setInterval(() => {
      Ρ‚Π΅Ρ‚Π°++;
    }, 1000/60);  

    ΠŸΠΎΠ»ΡΡ€Π½Ρ‹Π΅ ΠΊΡ€ΠΈΠ²Ρ‹Π΅

    ΠœΡ‹ Π½Π°Ρ‡Π°Π»ΠΈ с рассмотрСния ΠΎΡ‚Π΄Π΅Π»ΡŒΠ½Ρ‹Ρ… Ρ‚ΠΎΡ‡Π΅ΠΊ.Π—Π°Ρ‚Π΅ΠΌ ΠΌΡ‹ сгруппировали нСсколько Ρ‚ΠΎΡ‡Π΅ΠΊ Π² Π½Π°Π±ΠΎΡ€ для опрСдСлСния Ρ„ΠΎΡ€ΠΌ. Для этого ΠΌΡ‹ использовали Ρ„ΡƒΠ½ΠΊΡ†ΠΈΡŽ Π³Π΅Π½Π΅Ρ€Π°Ρ‚ΠΎΡ€Π° ΠΏΠΎΠ»ΠΈΠ³ΠΎΠ½ΠΎΠ² для вычислСния мСстополоТСния ΠΊΠ°ΠΆΠ΄ΠΎΠΉ Π²Π΅Ρ€ΡˆΠΈΠ½Ρ‹ Ρ„ΠΈΠ³ΡƒΡ€Ρ‹. ΠœΡ‹ ΠΌΠΎΠΆΠ΅ΠΌ Π½Π°ΠΏΠΈΡΠ°Ρ‚ΡŒ Π°Π½Π°Π»ΠΎΠ³ΠΈΡ‡Π½Ρ‹Π΅ Ρ„ΡƒΠ½ΠΊΡ†ΠΈΠΈ, ΠΈΡΠΏΠΎΠ»ΡŒΠ·ΡƒΡ Π΄Ρ€ΡƒΠ³ΠΈΠ΅ матСматичСскиС уравнСния. Π­Ρ‚ΠΎ позволяСт Π½Π°ΠΌ ΡΠΎΠ·Π΄Π°Π²Π°Ρ‚ΡŒ Π±ΠΎΠ»Π΅Π΅ слоТныС Ρ„ΠΎΡ€ΠΌΡ‹ ΠΈ ΠΊΡ€ΠΈΠ²Ρ‹Π΅.

    Π”Π²ΡƒΠΌΠ΅Ρ€Π½Ρ‹Π΅ ΠΊΡ€ΠΈΠ²Ρ‹Π΅ ΠΎΠΏΠΈΡΡ‹Π²Π°ΡŽΡ‚ΡΡ уравнСниями Ρ‚ΠΈΠΏΠ° y = f(x) . НапримСр, ΡƒΡ€Π°Π²Π½Π΅Π½ΠΈΠ΅ окруТности x 2 + y 2 = r 2 . ΠœΡ‹ ΠΌΠΎΠΆΠ΅ΠΌ ΡΠ³Π΅Π½Π΅Ρ€ΠΈΡ€ΠΎΠ²Π°Ρ‚ΡŒ Π½Π°Π±ΠΎΡ€ Ρ‚ΠΎΡ‡Π΅ΠΊ, Π½Π°Π·Ρ‹Π²Π°Π΅ΠΌΡ‹ΠΉ локусом, ΠΏΡƒΡ‚Π΅ΠΌ повторСния x ΠΈ вычислСния ΡΠΎΠΎΡ‚Π²Π΅Ρ‚ΡΡ‚Π²ΡƒΡŽΡ‰Π΅Π³ΠΎ значСния y ΠΈΠ»ΠΈ Π½Π°ΠΎΠ±ΠΎΡ€ΠΎΡ‚.Π‘Π»Π΅Π΄ΠΎΠ²Π°Ρ‚Π΅Π»ΡŒΠ½ΠΎ, каТдая Ρ‚ΠΎΡ‡ΠΊΠ° Π±ΡƒΠ΄Π΅Ρ‚ ΠΈΠΌΠ΅Ρ‚ΡŒ Π²ΠΈΠ΄ (x, f(x)) ΠΈΠ»ΠΈ (g(y), y) .

    Π‘ полярными ΠΊΠΎΠΎΡ€Π΄ΠΈΠ½Π°Ρ‚Π°ΠΌΠΈ ΠΌΡ‹ ΠΌΠΎΠΆΠ΅ΠΌ Π°Π½Π°Π»ΠΎΠ³ΠΈΡ‡Π½Ρ‹ΠΌ ΠΎΠ±Ρ€Π°Π·ΠΎΠΌ Ρ€ΠΈΡΠΎΠ²Π°Ρ‚ΡŒ полярныС ΠΊΡ€ΠΈΠ²Ρ‹Π΅. НапримСр, полярноС ΡƒΡ€Π°Π²Π½Π΅Π½ΠΈΠ΅ окруТности r = 2 * cos(0) . Π’ΠΎΡ‡ΠΊΠΈ Π½Π° полярной ΠΊΡ€ΠΈΠ²ΠΎΠΉ ΠΈΠΌΠ΅ΡŽΡ‚ Π²ΠΈΠ΄ (r(0), 0) .

     
    
    
    const r = fn (Ρ‚Π΅Ρ‚Π°);
    
    const x = cx + r * Math.cos(theta);
    const y = cy + r * Math.sin(theta);  

    Eukleides

    ВсС Π΄ΠΈΠ°Π³Ρ€Π°ΠΌΠΌΡ‹ Π² этом постС Π±Ρ‹Π»ΠΈ созданы с использованиСм языка eukleides.Π­Ρ‚ΠΎ фантастичСский инструмСнт для создания гСомСтричСских рисунков. ΠŸΡ€ΠΎΡΡ‚ΠΎ посмотритС Π½Π° этот Π΄Π΅ΠΊΠ»Π°Ρ€Π°Ρ‚ΠΈΠ²Π½Ρ‹ΠΉ API 😍

      c = ΠΊΡ€ΡƒΠ³(Ρ‚ΠΎΡ‡ΠΊΠ°(3, 0), 3)
    P = Ρ‚ΠΎΡ‡ΠΊΠ° (c, 170Β°)
    М = Ρ‚ΠΎΡ‡ΠΊΠ° (0, 0)
    N = Ρ‚ΠΎΡ‡ΠΊΠ° (6, 0)
    
    Π½ΠΈΡ‡ΡŒΡ (M.N.P)
    ΠΌΠ΅Ρ‚ΠΊΠ° M, P, N справа, 0,6  

    Π’Π²Π΅Π΄Π΅Π½ΠΈΠ΅ Π² полярныС ΠΊΠΎΠΎΡ€Π΄ΠΈΠ½Π°Ρ‚Ρ‹

    Β 

    Π’ Π½Π΅ΠΊΠΎΡ‚ΠΎΡ€ΠΎΠΌ смыслС ΠΌΠΎΠΆΠ΅Ρ‚ ΠΏΠΎΠΊΠ°Π·Π°Ρ‚ΡŒΡΡ странным, Ρ‡Ρ‚ΠΎ ΠΏΠ΅Ρ€Π²Ρ‹ΠΉ способ, ΠΊΠΎΡ‚ΠΎΡ€Ρ‹ΠΌ нас ΡƒΡ‡Π°Ρ‚ ΠΏΡ€Π΅Π΄ΡΡ‚Π°Π²Π»ΡΡ‚ΡŒ ΠΏΠΎΠ»ΠΎΠΆΠ΅Π½ΠΈΠ΅ ΠΎΠ±ΡŠΠ΅ΠΊΡ‚ΠΎΠ² Π² ΠΌΠ°Ρ‚Π΅ΠΌΠ°Ρ‚ΠΈΠΊΠ΅, β€” это использованиС Π΄Π΅ΠΊΠ°Ρ€Ρ‚ΠΎΠ²Ρ‹Ρ… ΠΊΠΎΠΎΡ€Π΄ΠΈΠ½Π°Ρ‚, Ρ‚ΠΎΠ³Π΄Π° ΠΊΠ°ΠΊ этот ΠΌΠ΅Ρ‚ΠΎΠ΄ опрСдСлСния мСстополоТСния Π½Π΅ самый СстСствСнный ΠΈΠ»ΠΈ самый ΡƒΠ΄ΠΎΠ±Π½Ρ‹ΠΉ.Для Π½Π°Ρ‡Π°Π»Π° Π²Π°ΠΌ Π½ΡƒΠΆΠ½ΠΎ ΠΈΡΠΏΠΎΠ»ΡŒΠ·ΠΎΠ²Π°Ρ‚ΡŒ ΠΊΠ°ΠΊ ΠΏΠΎΠ»ΠΎΠΆΠΈΡ‚Π΅Π»ΡŒΠ½Ρ‹Π΅, Ρ‚Π°ΠΊ ΠΈ ΠΎΡ‚Ρ€ΠΈΡ†Π°Ρ‚Π΅Π»ΡŒΠ½Ρ‹Π΅ числа для описания всСх Ρ‚ΠΎΡ‡Π΅ΠΊ Π½Π° плоскости, ΠΈ Π²Ρ‹ Π΄ΠΎΠ»ΠΆΠ½Ρ‹ ΡΠΎΠ·Π΄Π°Ρ‚ΡŒ сСтку (оси скваТин), Ρ‡Ρ‚ΠΎΠ±Ρ‹ ΠΈΡΠΏΠΎΠ»ΡŒΠ·ΠΎΠ²Π°Ρ‚ΡŒ Π΅Π΅ Π² качСствС ссылка.

    Когда Π²Ρ‹ спроситС Ρ€Π΅Π±Π΅Π½ΠΊΠ°, Π³Π΄Π΅ ΠΎΠ½ оставил свой мяч, ΠΎΠ½ ΠΎΡ‚Π²Π΅Ρ‚ΠΈΡ‚: Β«Π²ΠΎΠ½ Ρ‚Π°ΠΌΒ» ΠΈ ΠΏΠΎΠΊΠ°ΠΆΠ΅Ρ‚ ΠΏΠ°Π»ΡŒΡ†Π΅ΠΌ. Он описываСт (хотя ΠΈ ΠΎΡ‡Π΅Π½ΡŒ ΠΏΡ€ΠΈΠ±Π»ΠΈΠ·ΠΈΡ‚Π΅Π»ΡŒΠ½ΠΎ) расстояниС Β«Π²ΠΎΠ½ Ρ‚Π°ΠΌΒ» ΠΈ Π½Π°ΠΏΡ€Π°Π²Π»Π΅Π½ΠΈΠ΅ Β«Π²ΠΎΠ½ Ρ‚Π°ΠΌΒ» ΠΈΠ»ΠΈ ΠΊΠΈΠ²ΠΎΠΊ Π³ΠΎΠ»ΠΎΠ²ΠΎΠΉ). Когда Π²Ρ‹ ΡΠΏΡ€Π°ΡˆΠΈΠ²Π°Π΅Ρ‚Π΅ ΠΊΠΎΠ³ΠΎ-Ρ‚ΠΎ, Π³Π΄Π΅ находится ΠΈΡ… Π³ΠΎΡ€ΠΎΠ΄, ΠΎΠ½ΠΈ часто говорят Ρ‡Ρ‚ΠΎ-Ρ‚ΠΎ Π²Ρ€ΠΎΠ΄Π΅ Β«ΠΏΡ€ΠΈΠΌΠ΅Ρ€Π½ΠΎ Π² 30$ милях ΠΊ сСвСру ΠΎΡ‚ Π›ΠΎΠ½Π΄ΠΎΠ½Π°Β».ΠžΠΏΡΡ‚ΡŒ ΠΆΠ΅, расстояниС ΠΈ Π½Π°ΠΏΡ€Π°Π²Π»Π΅Π½ΠΈΠ΅. НС Ρ‚Π°ΠΊ ΡƒΠΆ часто ΠΊΡ‚ΠΎ-Ρ‚ΠΎ Π΄Π°Π΅Ρ‚ ΡˆΠΈΡ€ΠΎΡ‚Ρƒ ΠΈ Π΄ΠΎΠ»Π³ΠΎΡ‚Ρƒ своСго Π³ΠΎΡ€ΠΎΠ΄Π°!

    Β 


    Π’Π°ΠΊΠΈΠΌ ΠΎΠ±Ρ€Π°Π·ΠΎΠΌ, использованиС расстояния ΠΈ направлСния Π² качСствС срСдства описания полоТСния Π³ΠΎΡ€Π°Π·Π΄ΠΎ Π±ΠΎΠ»Π΅Π΅ СстСствСнно, Ρ‡Π΅ΠΌ использованиС Π΄Π²ΡƒΡ… расстояний Π½Π° сСткС. Π­Ρ‚ΠΎ срСдство опрСдСлСния мСстополоТСния ΠΈΡΠΏΠΎΠ»ΡŒΠ·ΡƒΠ΅Ρ‚ΡΡ Π² полярных ΠΊΠΎΠΎΡ€Π΄ΠΈΠ½Π°Ρ‚Π°Ρ… ΠΈ ​​пСлСнгах.

    ΠŸΠΎΠ»ΡΡ€Π½Ρ‹Π΅ ΠΊΠΎΠΎΡ€Π΄ΠΈΠ½Π°Ρ‚Ρ‹ Ρ‚ΠΎΡ‡ΠΊΠΈ ΠΎΠΏΠΈΡΡ‹Π²Π°ΡŽΡ‚ Π΅Π΅ ΠΏΠΎΠ»ΠΎΠΆΠ΅Π½ΠΈΠ΅ Π² Ρ‚Π΅Ρ€ΠΌΠΈΠ½Π°Ρ… расстояния ΠΎΡ‚ фиксированной Ρ‚ΠΎΡ‡ΠΊΠΈ (Π½Π°Ρ‡Π°Π»Π° ΠΊΠΎΠΎΡ€Π΄ΠΈΠ½Π°Ρ‚) ΠΈ ΡƒΠ³Π»Π°, ΠΈΠ·ΠΌΠ΅Ρ€Π΅Π½Π½ΠΎΠ³ΠΎ ΠΎΡ‚ фиксированного направлСния, ΠΊΠΎΡ‚ΠΎΡ€ΠΎΠ΅, Ρ‡Ρ‚ΠΎ интСрСсно, Π½Π΅ являСтся «сСвСром» (ΠΈΠ»ΠΈ Π²Π²Π΅Ρ€Ρ… Π½Π° страницС), Π° «восток» (Π²ΠΏΡ€Π°Π²ΠΎ).Π’ΠΎ Π΅ΡΡ‚ΡŒ Π² Π½Π°ΠΏΡ€Π°Π²Π»Π΅Π½ΠΈΠΈ $Ox$ ΠΏΠΎ Π΄Π΅ΠΊΠ°Ρ€Ρ‚ΠΎΠ²ΠΎΠΉ оси.

    Π˜Ρ‚Π°ΠΊ:

    На плоскости ΠΌΡ‹ Π²Ρ‹Π±ΠΈΡ€Π°Π΅ΠΌ Ρ„ΠΈΠΊΡΠΈΡ€ΠΎΠ²Π°Π½Π½ΡƒΡŽ Ρ‚ΠΎΡ‡ΠΊΡƒ $O$, ΠΈΠ·Π²Π΅ΡΡ‚Π½ΡƒΡŽ ΠΊΠ°ΠΊ «ΠΏΠΎΠ»ΡŽΡ».

    Π—Π°Ρ‚Π΅ΠΌ ΠΌΡ‹ Π²Ρ‹Π±ΠΈΡ€Π°Π΅ΠΌ ось $Ox$ Ρ‡Π΅Ρ€Π΅Π· полюс ΠΈ Π½Π°Π·Ρ‹Π²Π°Π΅ΠΌ Π΅Π΅ «ΠΏΠΎΠ»ΡΡ€Π½ΠΎΠΉ осью».

    Π’Π΅ΠΏΠ΅Ρ€ΡŒ Π½Π°ΠΌ Π½ΡƒΠΆΠ΅Π½ способ описания этих Ρ‚ΠΎΡ‡Π΅ΠΊ Ρ‚Π°ΠΊΠΈΠΌ ΠΎΠ±Ρ€Π°Π·ΠΎΠΌ, Ρ‡Ρ‚ΠΎΠ±Ρ‹ ΠΎΠ½ Π±Ρ‹Π» эффСктивным ΠΈ понятным для всСх.c$… ΠΊ ΡƒΠ³Π»Ρƒ ΠΈ всС Π΅Ρ‰Π΅ Π² ΠΊΠΎΠ½Π΅Ρ‡Π½ΠΎΠΌ ΠΈΡ‚ΠΎΠ³Π΅ ΡƒΠΊΠ°Π·Ρ‹Π²Π°ΡŽΡ‚ Π² Ρ‚ΠΎΠΌ ΠΆΠ΅ Π½Π°ΠΏΡ€Π°Π²Π»Π΅Π½ΠΈΠΈ! Π’ ΠΏΡ€ΠΈΠ²Π΅Π΄Π΅Π½Π½ΠΎΠΌ Π²Ρ‹ΡˆΠ΅ ΠΏΡ€ΠΈΠΌΠ΅Ρ€Π΅ ΠΎΠ±Ρ‰ΠΈΠ΅ ΠΊΠΎΠΎΡ€Π΄ΠΈΠ½Π°Ρ‚Ρ‹ для $A$ Π±ΡƒΠ΄ΡƒΡ‚ $(90,2n\pi + \frac{\pi}{2})$, Π³Π΄Π΅ $n$ β€” Ρ†Π΅Π»ΠΎΠ΅ число.

    Β 

    Π­Ρ‚ΠΎ Ρ‚Π°ΠΊΠΆΠ΅ ΠΎΠ·Π½Π°Ρ‡Π°Π΅Ρ‚, Ρ‡Ρ‚ΠΎ полярныС ΠΊΠΎΠΎΡ€Π΄ΠΈΠ½Π°Ρ‚Ρ‹ полюса $O$ Ρ€Π°Π²Π½Ρ‹ $(0,\theta)$, Π³Π΄Π΅ $\theta$ ΠΌΠΎΠΆΠ΅Ρ‚ Π±Ρ‹Ρ‚ΡŒ Π»ΡŽΠ±Ρ‹ΠΌ ΡƒΠ³Π»ΠΎΠΌ.

    Β 

    Бвязь ΠΌΠ΅ΠΆΠ΄Ρƒ полярными ΠΈ Π΄Π΅ΠΊΠ°Ρ€Ρ‚ΠΎΠ²Ρ‹ΠΌΠΈ ΠΊΠΎΠΎΡ€Π΄ΠΈΠ½Π°Ρ‚Π°ΠΌΠΈ

    ΠŸΡ€Π΅Π΄ΡΡ‚Π°Π²ΡŒΡ‚Π΅ сСбС Ρ‚ΠΎΡ‡ΠΊΡƒ $P$ с полярными ΠΊΠΎΠΎΡ€Π΄ΠΈΠ½Π°Ρ‚Π°ΠΌΠΈ $(r,\theta)$. ΠŸΠΎΠΏΡ€ΠΎΠ±ΡƒΠ΅ΠΌ ΠΈΡΠΏΠΎΠ»ΡŒΠ·ΠΎΠ²Π°Ρ‚ΡŒ эту ΠΈΠ½Ρ„ΠΎΡ€ΠΌΠ°Ρ†ΠΈΡŽ для получСния Π΄Π΅ΠΊΠ°Ρ€Ρ‚ΠΎΠ²Ρ‹Ρ… ΠΊΠΎΠΎΡ€Π΄ΠΈΠ½Π°Ρ‚ $P$. ΠœΡ‹ ΠΌΠΎΠΆΠ΅ΠΌ ΠΎΠΏΡƒΡΡ‚ΠΈΡ‚ΡŒ пСрпСндикуляр ΠΈΠ· Ρ‚ΠΎΡ‡ΠΊΠΈ $P$ Π½Π° $Ox$, ΠΏΠ΅Ρ€Π΅ΡΠ΅ΠΊΠ°ΡŽΡ‰ΠΈΠΉ $Ox$ Π² Ρ‚ΠΎΡ‡ΠΊΠ΅ $Q$.Π”Π»ΠΈΠ½Ρ‹ $OQ$ ΠΈ $OP$ ΠΏΡ€Π΅Π΄ΡΡ‚Π°Π²Π»ΡΡŽΡ‚ собой ΠΊΠΎΠΎΡ€Π΄ΠΈΠ½Π°Ρ‚Ρ‹ $x$ ΠΈ $y$ Π² Π΄Π΅ΠΊΠ°Ρ€Ρ‚ΠΎΠ²ΠΎΠΉ Ρ„ΠΎΡ€ΠΌΠ΅, поэтому Π½Π°ΠΌ просто Π½ΡƒΠΆΠ½ΠΎ Π½Π°ΠΉΡ‚ΠΈ эти Π΄Π²Π° расстояния.

    Β 

    $$\begin{eqnarray} PQ &=& r \sin \theta \\ OQ &=& r \cos \theta \end{eqnarray}$$

    Π‘Π»Π΅Π΄ΠΎΠ²Π°Ρ‚Π΅Π»ΡŒΠ½ΠΎ, Π΄Π΅ΠΊΠ°Ρ€Ρ‚ΠΎΠ²Ρ‹ ΠΊΠΎΠΎΡ€Π΄ΠΈΠ½Π°Ρ‚Ρ‹ $P$ Ρ€Π°Π²Π½Ρ‹ $(r \ sin \theta, r \cos \theta)$

    Π’Π΅ΠΏΠ΅Ρ€ΡŒ ΠΏΠΎΠΉΠ΄Π΅ΠΌ Π΄Ρ€ΡƒΠ³ΠΈΠΌ ΠΏΡƒΡ‚Π΅ΠΌ:

    НачнСм с Π΄Π΅ΠΊΠ°Ρ€Ρ‚ΠΎΠ²ΠΎΠΉ систСмы ΠΊΠΎΠΎΡ€Π΄ΠΈΠ½Π°Ρ‚.

    ΠœΡ‹ возьмСм Π΄Π΅ΠΊΠ°Ρ€Ρ‚ΠΎΠ²Ρ‹ ΠΊΠΎΠΎΡ€Π΄ΠΈΠ½Π°Ρ‚Ρ‹ $P$ ΠΊΠ°ΠΊ $(x,y)$.Π²)$!!

    Π˜ΡΠΏΠΎΠ»ΡŒΠ·ΡƒΡ Π·Π½Π°ΠΊΠΈ $\sin\theta$ ΠΈ $\cos\theta$, Π²Ρ‹ ΠΌΠΎΠΆΠ΅Ρ‚Π΅ Π±Ρ‹Ρ‚ΡŒ ΡƒΠ²Π΅Ρ€Π΅Π½Ρ‹, Ρ‡Ρ‚ΠΎ ΡƒΠ³ΠΎΠ» находится Π² ΠΏΡ€Π°Π²ΠΈΠ»ΡŒΠ½ΠΎΠΌ ΠΊΠ²Π°Π΄Ρ€Π°Π½Ρ‚Π΅.

    Π˜Ρ‚Π°ΠΊ, Π΄Π°Π²Π°ΠΉΡ‚Π΅ Π·Π°ΠΊΠΎΠ½Ρ‡ΠΈΠΌ с использованиСм этой систСмы ΠΊΠΎΠΎΡ€Π΄ΠΈΠ½Π°Ρ‚. Π‘Ρ‹Π»ΠΎ Π±Ρ‹ Π½Π΅ΠΏΠ»ΠΎΡ…ΠΎ ΠΏΠΎΠΏΡ€ΠΎΠ±ΠΎΠ²Π°Ρ‚ΡŒ Π½Π΅ΠΊΠΎΡ‚ΠΎΡ€Ρ‹Π΅ уравнСния ΠΈ ΠΏΠΎΡΠΌΠΎΡ‚Ρ€Π΅Ρ‚ΡŒ Π½Π° ΠΈΡ… Π³Ρ€Π°Ρ„ΠΈΠΊΠΈ (полярныС Π΄ΠΈΠ°Π³Ρ€Π°ΠΌΠΌΡ‹).

    Рассмотрим нСсколько ΠΏΡ€ΠΈΠΌΠ΅Ρ€ΠΎΠ²:

    Рассмотрим Π³Ρ€Π°Ρ„ΠΈΠΊ:

    $r= \theta$

    Он ΠΈΠΌΠ΅Π΅Ρ‚ Ρ„ΠΎΡ€ΠΌΡƒ спирали (каТдая Ρ‚ΠΎΡ‡ΠΊΠ° смСщаСтся ΠΎΡ‚ Ρ†Π΅Π½Ρ‚Ρ€Π° ΠΏΠΎ ΠΌΠ΅Ρ€Π΅ увСличСния ΡƒΠ³Π»Π°).

    На ΠΏΡ€ΠΈΠ²Π΅Π΄Π΅Π½Π½ΠΎΠΉ Π½ΠΈΠΆΠ΅ Π΄ΠΈΠ°Π³Ρ€Π°ΠΌΠΌΠ΅ ΠΏΠΎΠΊΠ°Π·Π°Π½Ρ‹ Π³Ρ€Π°Ρ„ΠΈΠΊΠΈ $r= a\theta$ для Ρ€Π°Π·Π»ΠΈΡ‡Π½Ρ‹Ρ… Π·Π½Π°Ρ‡Π΅Π½ΠΈΠΉ $a$. ΠœΠΎΠΆΠ΅Ρ‚Π΅ Π»ΠΈ Π²Ρ‹ ΠΎΠΏΡ€Π΅Π΄Π΅Π»ΠΈΡ‚ΡŒ, Ρ‡Ρ‚ΠΎ это Π·Π° Π³Ρ€Π°Ρ„ΠΈΠΊΠΈ?

    Β 

    Β 

    Β 

    Β 

    Π’Π΅ΠΏΠ΅Ρ€ΡŒ твоя ΠΎΡ‡Π΅Ρ€Π΅Π΄ΡŒ.ГрафичСский ΠΊΠ°Π»ΡŒΠΊΡƒΠ»ΡΡ‚ΠΎΡ€ ΠΈΠ»ΠΈ графичСский ΠΏΠ°ΠΊΠ΅Ρ‚ Π±Ρ‹Π»ΠΈ Π±Ρ‹ ΠΎΡ‡Π΅Π½ΡŒ ΠΏΠΎΠ»Π΅Π·Π½Ρ‹!

    Как Π±ΡƒΠ΄Π΅Ρ‚ Π²Ρ‹Π³Π»ΡΠ΄Π΅Ρ‚ΡŒ сСрия Π³Ρ€Π°Ρ„ΠΈΠΊΠΎΠ²

    $r=1, r=2, r=3, $…?

    Как насчСт $r = 2a(1 + \cos\theta )$ для Ρ€Π°Π·Π½Ρ‹Ρ… Π·Π½Π°Ρ‡Π΅Π½ΠΈΠΉ $a$? ΠšΡΡ‚Π°Ρ‚ΠΈ, эти Π³Ρ€Π°Ρ„ΠΈΠΊΠΈ Π½Π°Π·Ρ‹Π²Π°ΡŽΡ‚ΡΡ ΠΊΠ°Ρ€Π΄ΠΈΠΎΠΈΠ΄Π°ΠΌΠΈ.

    ΠžΡ‚Π²Π΅Ρ‚Ρ‹:

    $$\begin{eqnarray} \mbox{D }\rightarrow(60,0)\\ \mbox{E }\rightarrow(30, 270)\\ \mbox{C }\rightarrow(120 , 225)\\ \mbox{A}\стрСлка Π²ΠΏΡ€Π°Π²ΠΎ(90, 90)\\ \mbox{F}\стрСлка Π²ΠΏΡ€Π°Π²ΠΎ(60,60)\\ \mbox{B}\стрСлка Π²ΠΏΡ€Π°Π²ΠΎ(120, 180) \end{eqnarray} $$ И $$\begin{eqnarray} (60,0)\rightarrow(60,0)\\ (30, 270)\rightarrow(30, \frac{3\pi}{2})\\ (120, 225)\стрСлка Π²ΠΏΡ€Π°Π²ΠΎ(120, \frac{5\pi}{4})\\ (90, 90)\стрСлка Π²ΠΏΡ€Π°Π²ΠΎ(90, \frac{\pi}{2})\\ (60,60)\rightarrow(60,\frac{\pi}{3})\\ (120, 180)\rightarrow(120,\pi) \end{eqnarray}$$

    Β 

    Β 

    Π‘ΠΎΠΌΠ½ΠΈΡ‚Π΅ Π΅Π³ΠΎ, Ρ‡Ρ‚ΠΎΠ±Ρ‹ получился Π²Π΅Π΅Ρ€ Π²ΠΎΠΊΡ€ΡƒΠ³ Ρ‚ΠΎΡ‡ΠΊΠΈ, ΠΈΠ»ΠΈ .. . .

    . . . . ΠΊΠ°ΠΊ ΠΈΡΠΏΠΎΠ»ΡŒΠ·ΠΎΠ²Π°Ρ‚ΡŒ Π³Ρ€Π°Ρ„ΠΈΠΊ xy, Ρ‡Ρ‚ΠΎΠ±Ρ‹ ΠΏΠΎΠΌΠΎΡ‡ΡŒ Π²ΠΈΠ·ΡƒΠ°Π»ΠΈΠ·ΠΈΡ€ΠΎΠ²Π°Ρ‚ΡŒ полярный Π³Ρ€Π°Ρ„ΠΈΠΊ

    Когда Π²Ρ‹ ΠΏΡ‹Ρ‚Π°Π΅Ρ‚Π΅ΡΡŒ ΠΏΡ€Π΅Π΄ΡΡ‚Π°Π²ΠΈΡ‚ΡŒ сСбС, ΠΊΠ°ΠΊ Π±ΡƒΠ΄Π΅Ρ‚ Π²Ρ‹Π³Π»ΡΠ΄Π΅Ρ‚ΡŒ полярный Π³Ρ€Π°Ρ„ΠΈΠΊ Ρ„ΡƒΠ½ΠΊΡ†ΠΈΠΈ, ΠΈΠ½ΠΎΠ³Π΄Π° ΠΌΠΎΠΆΠ΅Ρ‚ Π±Ρ‹Ρ‚ΡŒ ΠΏΠΎΠ»Π΅Π·Π½ΠΎ сначала Π²Π·Π³Π»ΡΠ½ΡƒΡ‚ΡŒ Π½Π° Π΄Π΅ΠΊΠ°Ρ€Ρ‚ΠΎΠ² (xy) Π³Ρ€Π°Ρ„ΠΈΠΊ для этой Ρ„ΡƒΠ½ΠΊΡ†ΠΈΠΈ, ΠΈΡΠΏΠΎΠ»ΡŒΠ·ΡƒΡ значСния ΠΎΡ‚ $0$ Π΄ΠΎ $2\pi$ (ΠΎΡ‚ $0$ Π΄ΠΎ $360$). градусов), Π° Π·Π°Ρ‚Π΅ΠΌ ΠΏΡ€Π΅Π΄ΡΡ‚Π°Π²ΡŒΡ‚Π΅, Ρ‡Ρ‚ΠΎ Π³Ρ€Π°Ρ„ΠΈΠΊ прСвратился Π² Π²Π΅Π΅Ρ€.

    Β 

    ΠŸΡ€Π΅Π΄ΡΡ‚Π°Π²ΡŒΡ‚Π΅, Ρ‡Ρ‚ΠΎ ось X втянута Π² Ρ‚ΠΎΡ‡ΠΊΡƒ, Π° значСния Ρ„ΡƒΠ½ΠΊΡ†ΠΈΠΉ разбросаны Π²ΠΎΠΊΡ€ΡƒΠ³ Π½Π΅Π΅ Π²Π΅Π΅Ρ€ΠΎΠΌ.

    Β 

    Β 

    НапримСр: $y = 5 \sin 2x$ выглядит ΠΊΠ°ΠΊ Π΄Π΅ΠΊΠ°Ρ€Ρ‚ΠΎΠ²Π° Π΄ΠΈΠ°Π³Ρ€Π°ΠΌΠΌΠ°.

    Β 

    Β 

    Β 

    Β 

    Β 

    Β 

    Но ΠΊΠ°ΠΊ полярный Π³Ρ€Π°Ρ„ΠΈΠΊ $r = 5\sin 2\theta$:

    На ΠΈΠ½Ρ‚Π΅Ρ€Π²Π°Π»Π΅ ΠΎΡ‚ $0$ Π΄ΠΎ $2\pi$ Π³Ρ€Π°Ρ„ $\sin 2x$ ΠΈΠΌΠ΅Π΅Ρ‚ $4$ областСй

    Π’ области $1$ функция возрастаСт Π΄ΠΎ максимального значСния $5$, Π° Π·Π°Ρ‚Π΅ΠΌ симмСтрично ΠΏΠ°Π΄Π°Π΅Ρ‚ Π΄ΠΎ нуля.

    Β 

    Π’ области $2$ функция ΠΏΠ°Π΄Π°Π΅Ρ‚ Π΄ΠΎ минимального значСния $-5$, послС Ρ‡Π΅Π³ΠΎ возвращаСтся ΠΊ Π½ΡƒΠ»ΡŽ.

    Β 

    Β 

    ΠžΠ±Ρ€Π°Ρ‚ΠΈΡ‚Π΅ Π²Π½ΠΈΠΌΠ°Π½ΠΈΠ΅ Π½Π° ΠΏΠΎΠ»ΠΎΠΆΠ΅Π½ΠΈΠ΅ области $2$ Π½Π° полярной Π΄ΠΈΠ°Π³Ρ€Π°ΠΌΠΌΠ΅: ΠΊΠΎΠ³Π΄Π° $\theta$ ΠΏΠ΅Ρ€Π΅ΠΌΠ΅Ρ‰Π°Π΅Ρ‚ Π²Ρ‚ΠΎΡ€ΠΎΠΉ ΠΊΠ²Π°Π΄Ρ€Π°Π½Ρ‚ ΠΎΡ‚ $\pi/2 $ Π΄ΠΎ $\pi$, всС значСния r ΠΎΡ‚Ρ€ΠΈΡ†Π°Ρ‚Π΅Π»ΡŒΠ½Ρ‹, проСцируя ΠΊΠ°ΠΆΠ΄ΡƒΡŽ Ρ‚ΠΎΡ‡ΠΊΡƒ Π³Ρ€Π°Ρ„ΠΈΠΊΠ° Π½Π°Π·Π°Π΄, Π² Ρ‡Π΅Ρ‚Π²Π΅Ρ€Ρ‚Ρ‹ΠΉ ΠΊΠ²Π°Π΄Ρ€Π°Π½Ρ‚. .

    Β 

    Β 

    ΠžΠ±Π»Π°ΡΡ‚ΡŒ $3$ такая ΠΆΠ΅ простая, ΠΊΠ°ΠΊ ΠΈ ΠΎΠ±Π»Π°ΡΡ‚ΡŒ $1$, Π° ΠΎΠ±Π»Π°ΡΡ‚ΡŒ $4$, ΠΊΠ°ΠΊ ΠΈ ΠΎΠ±Π»Π°ΡΡ‚ΡŒ $2$, Ρ‚Π°ΠΊΠΆΠ΅ ΠΈΠΌΠ΅Π΅Ρ‚ ΠΎΡ‚Ρ€ΠΈΡ†Π°Ρ‚Π΅Π»ΡŒΠ½Ρ‹Π΅ значСния $r$ ΠΈ поэтому находится Π²ΠΎ Π²Ρ‚ΠΎΡ€ΠΎΠΌ ΠΊΠ²Π°Π΄Ρ€Π°Π½Ρ‚Π΅.

    Π’Π°Ρˆ ΠΊΠΎΠΌΠΌΠ΅Π½Ρ‚Π°Ρ€ΠΈΠΉ Π±ΡƒΠ΄Π΅Ρ‚ ΠΏΠ΅Ρ€Π²Ρ‹ΠΌ

      Π”ΠΎΠ±Π°Π²ΠΈΡ‚ΡŒ ΠΊΠΎΠΌΠΌΠ΅Π½Ρ‚Π°Ρ€ΠΈΠΉ

      Π’Π°Ρˆ адрСс email Π½Π΅ Π±ΡƒΠ΄Π΅Ρ‚ ΠΎΠΏΡƒΠ±Π»ΠΈΠΊΠΎΠ²Π°Π½. ΠžΠ±ΡΠ·Π°Ρ‚Π΅Π»ΡŒΠ½Ρ‹Π΅ поля ΠΏΠΎΠΌΠ΅Ρ‡Π΅Π½Ρ‹ *